Naked Science Forum

Non Life Sciences => Physics, Astronomy & Cosmology => Topic started by: thedoc on 07/08/2015 13:50:01

Title: What was before the big bang?
Post by: thedoc on 07/08/2015 13:50:01
C Bruce Rodgers asked the Naked Scientists:
   
Theologians have an explanation for the pre-bang state of the universe, but is there any scientific speculation on the source of the source?  Even a cosmic yo-yo started from something.

What do you think?
Title: Re: What was before the big bang?
Post by: Mordeth on 08/08/2015 21:48:41
You are asking about a "time before time".  All fundamental physical laws broke down at the Big Bang, and the Universe "started over".  There is no event prior to the Big Bang that is observable, therefore you are free to speculate.
Title: Re: What was before the big bang?
Post by: PmbPhy on 09/08/2015 06:24:09
Quote from: thedoc
Theologians have an explanation for the pre-bang state of the universe, but is there any scientific speculation on the source of the source?
What is their explanation and what do you mean by "source of the source"?

Regarding the question What was before the big bang?.Depending on what one calls the start of the big bang one could say it was either the inflationary epoch of the Pre-Big Bang epoch. But we don't know with any great certainty.

The Pre-Big Bang Scenario uses string theory so I can't tell you much about it. You can read about it at: The Pre-Big Bang Scenario in String Cosmology by M. Gasperini, G. Veneziano:
http://arxiv.org/abs/hep-th/0207130

The authors are well-known in this field.
Title: Re: What was before the big bang?
Post by: PmbPhy on 09/08/2015 06:26:00
Quote from: Mordeth
You are asking about a "time before time". 
It's not 100% certain that time started when the big bang did.
Title: Re: What was before the big bang?
Post by: Mordeth on 09/08/2015 13:50:06
Quote from: Mordeth
You are asking about a "time before time". 
It's not 100% certain that time started when the big bang did.

Hi Pmb

Not many things are 100% certain.  However, at the singularity of the Big Bang, all known laws of physics are understood to have broken down.  Even conservation of matter.  By definition, nothing before the Big Bang is observable, or even knowable.  The Universe effectively reset itself, wiping out all traces of any former incarnation it might have had, including time.  This is because reaching back in time leads us to a singularity. If our past light cone can be focused, then theories of singularities can be used to show that this was the beginning of time, for our universe. There is nothing meaningful one can say regarding events before the Big Bang.  Only idle speculation.
Title: Re: What was before the big bang?
Post by: Mordeth on 09/08/2015 13:57:50
There was no big bang, the universe has always been here. I could post a pdf of where my theory is now if you are interested, but im having the day off, but your website is restricted to the size of pdf and the file is now 300k.
once you know about the fundamdental force carrier, it falls into place. I think the theory is finished now, im posting worldwide shortly, but it was you that pointed me in the write direction. Quantum mechanics needs i minor tweek not that i understand it that well, general relativity needs a minor tweek.  [:)]
I am getting paranoid re the lack of response to my posts

Is your theory consistent with the second law of thermodynamics?  An infinite universe would likely be one of uniform temperature.  Do you understand the logical process of tracing backwards the fact of expansion?
Title: Re: What was before the big bang?
Post by: PmbPhy on 09/08/2015 18:06:16
See also http://www.scientificamerican.com/article/string-theory-predicts-a-time-before-the-big-bang/
Quote
Was the big bang really the beginning of time? or did the universe exist before then? Such a question seemed almost blasphemous only a decade ago. Most cosmologists insisted that it simply made no sense—that to contemplate a time before the big bang was like asking for directions to a place north of the North Pole. But developments in theoretical physics, especially the rise of string theory, have changed their perspective. The pre-bang universe has become the latest frontier of cosmology.

The new willingness to consider what might have happened before the bang is the latest swing of an intellectual pendulum that has rocked back and forth for millennia. In one form or another, the issue of the ultimate beginning has engaged philosophers and theologians in nearly every culture. It is entwined with a grand set of concerns, one famously encapsulated in an 1897 painting by Paul Gauguin: D'ou venons-nous? Que sommes-nous? Ou allons-nous? “Where do we come from? What are we? Where are we going?” The piece depicts the cycle of birth, life and death—origin, identity and destiny for each individual—and these personal concerns connect directly to cosmic ones. We can trace our lineage back through the generations, back through our animal ancestors, to early forms of life and protolife, to the elements synthesized in the primordial universe, to the amorphous energy deposited in space before that. Does our family tree extend forever backward? Or do its roots terminate? Is the cosmos as impermanent as we are?
Title: Re: What was before the big bang?
Post by: PmbPhy on 09/08/2015 18:08:19
Quote from: Aquarius
There was no big bang, the universe has always been here.
Those are two separate issues, neither of which belong here. They belong in the New Theories section. When people come to this particular forum they're looking to find out what mainstream physicists hold to be true. They're not asking for answers from people who claim that mainstream physics is wrong.
Title: Re: What was before the big bang?
Post by: Mordeth on 09/08/2015 20:22:04
See also http://www.scientificamerican.com/article/string-theory-predicts-a-time-before-the-big-bang/
Quote
Was the big bang really the beginning of time? or did the universe exist before then? Such a question seemed almost blasphemous only a decade ago. Most cosmologists insisted that it simply made no sense—that to contemplate a time before the big bang was like asking for directions to a place north of the North Pole. But developments in theoretical physics, especially the rise of string theory, have changed their perspective. The pre-bang universe has become the latest frontier of cosmology.

The new willingness to consider what might have happened before the bang is the latest swing of an intellectual pendulum that has rocked back and forth for millennia. In one form or another, the issue of the ultimate beginning has engaged philosophers and theologians in nearly every culture. It is entwined with a grand set of concerns, one famously encapsulated in an 1897 painting by Paul Gauguin: D'ou venons-nous? Que sommes-nous? Ou allons-nous? “Where do we come from? What are we? Where are we going?” The piece depicts the cycle of birth, life and death—origin, identity and destiny for each individual—and these personal concerns connect directly to cosmic ones. We can trace our lineage back through the generations, back through our animal ancestors, to early forms of life and protolife, to the elements synthesized in the primordial universe, to the amorphous energy deposited in space before that. Does our family tree extend forever backward? Or do its roots terminate? Is the cosmos as impermanent as we are?

Hi Pmb,

Thanks for the link.  As you know, any "theory" that attempts to describe the conditions that existed prior to the Big bang is untestable and not falsifiable.  Therefore, it is pure speculation, and does not nor cannot follow the scientific method.  The Big Bang, by definition, was a singularity.  There is no event prior to The Big Bang that is observable, nor can there ever be if the theory is accurate (which I believe it is).  We can go even further with this and state that anything that may have happened prior to The Big Bang is not even relevant.  The state of the Universe today was the result of the conditions OF the Big Bang, not before.  Period. That is to say that due to the singularity of the Big Bang, if there was anything prior it was FOREVER essentially erased and the Universe literally started over, whereby all physical laws were broken down.   Everything we observe today is a consequence of the Big Bang.  Not before (assuming before even has meaning in this context). Any and all attempts to avoid this conclusion are not scientifically valid, if The Big Bang is correct.  Time had a beginning, and it is called The Big Bang.

Humans intrinsically want to know what caused what and what came from where.  Our origin, and the origin of everything is as deep a subject as one can contemplate.  All that we can do today is say that the Big Bang accurately predicts and describes the Universe as we see it now.  It has not been falsified and it is possible to do so.  If you agree that the Big Bang theory is accurate, then you are forced to also agree that no scientific discussion can occur that produces testable conclusions or predictions regarding what happened before.  Therefore, your guess is as good as mine.  Better that we say: "We do not know, and likely CANNOT know".

Title: Re: What was before the big bang?
Post by: Bill S on 10/08/2015 18:35:34
Mordeth, your last post contains a number of dogmatic statements, the upshot of which seems little better than saying something like "God created it, so there's nothing to talk about".

My own feeling is that if there is something in nature we don't understand it is worth looking at to see if there is any insight to be had. You say:

Quote
The Big Bang, by definition, was a singularity.

That might be a good place to start, but you would need to say what you understand by a singularity.
Title: Re: What was before the big bang?
Post by: PmbPhy on 10/08/2015 23:02:06
Quote from: Mordeth
As you know, any "theory" that attempts to describe the conditions that existed prior to the Big bang is untestable and not falsifiable.
Where did you get that idea from? You're thinking that there is only one model of the Big Bang and that's not true.  One cannot rule out cyclic cosmological models, in which there is no Big Bang. Everything that you've said in this thread has assumed that there was a singularity and we don't know that there was such a thing.

In any case just because something is not falsifiable doesn't mean that it doesn't belong in science. The falsifiability criteria was proposed by Karl Popper but not all physicists/philosophers think that it should be adhered to. See:
https://en.wikipedia.org/wiki/Falsifiability#Criticisms

Please note that people have the wrong idea about the Big Bang. As Peebles explains in his text on physical cosmology
Quote
If there were an instant, at a "big bang," when the universe started expanding, it is not in cosmology as now accepted, because nobody has thought of a way to adduce objective physical evidence that such an event really happened.

Quote from: Mordeth
  Therefore, it is pure speculation, and does not nor cannot follow the scientific method.
That's quite wrong. Falsification is not part of the scientific method. See the article What is science? by American Physical Society, Am. J. Phys. 67 (8), August 1999
I placed this online at http://home.comcast.net/~peter.m.brown/ref/what_is_science.pdf
Quote
Science is the systematic enterprise of gathering knowledge about the world and organizing and condensing that knowledge into testable laws and theories. The success and credibility of science is anchored in the willingness of scientists to:

(1) expose their ideas and results to independent testing and replication by other scientists; this
requires the complete and open exchange of data, procedures and materials;

(2) abandon or modify accepted conclusions when confronted with more complete or reliable experimental evidence.

Adherence to these principles provides a mechanism for self-correction that is the foundation of the credibility of science
Nowhere in that does it speak of falsification.


Quote from: Mordeth
  The Big Bang, by definition, was a singularity.
You're wrong. It certainly isn't how the Big Bang was/is defined and there's no supporting evidence that there was a singularity

Quote from: Mordeth
There is no event prior to The Big Bang that is observable, ...
Yet. Physical cosmology isn't that old. While cosmology as a science dates back about 100 years our real understanding of the universe is in its infancy. We've only just begun to start applying modern methodology to the study of it.  Those answers might not be forth coming for hundreds of years as far as we know.

Quote from: Mordeth
We can go even further with this and state that anything that may have happened prior to The Big Bang is not even relevant.  The state of the Universe today was the result of the conditions OF the Big Bang, not before.  Period.
Please keep in mind that just because you're not interested in something it doesn't mean that the rest of the world isn't. Some scientists mind be very interested in Pre-Big Bang Scenarios as possible theories and couple that with the notion that there might be other universes out there in parallel to ours. And if we can build an Einstein-Rosen Bridge (aka wormhole) we can have access to those universes.

You're simply ruling out a lot of ideas just because you haven't thought of them. Suppose it's possible to create wormholes. The it'd be possible to go back in time. If we could do that and the universe didn't start out at a singularity then it might be possible to tunnel into the universe prior to the Big Bang with a micro wormhole and use it as a probe for information. Is that possible? I don't know either way. But that doesn't mean that we should not consider such things.

Anyway, all of your claims are based on one false assumption, i.e. that there was a singularity. But that's merely an extrapolation. According to the way you've been reasoning about there's no way to prove such a thing.
Title: Re: What was before the big bang?
Post by: Mordeth on 14/08/2015 21:51:24
Quote from: Mordeth
As you know, any "theory" that attempts to describe the conditions that existed prior to the Big bang is untestable and not falsifiable.
Where did you get that idea from? You're thinking that there is only one model of the Big Bang and that's not true.  One cannot rule out cyclic cosmological models, in which there is no Big Bang. Everything that you've said in this thread has assumed that there was a singularity and we don't know that there was such a thing.
Hi Pmb,

I am aware that there are many models of the Big Bang theory.  I think there are something like 70+ models of "Inflation" alone.   As you know, The Big Bang model itself was built on General Relativity.  And the results suggest a singularity of infinite density at the so called "beginning".  Now I realize that this supposed singularity is different than the singularity proposed to exist inside a black hole.  For the Big Bang, it would probably be more accurate to say that the math and modern physics simply break down and everything results in infinities.  Even more accurate to say all that we know is that it was "hot and dense".  The term singularity here is simply another way to say WE DON'T KNOW and that the physics break down.  Which has been my entire point.  If we could focus our past light cone, it would be possible to prove the singularity.

Do you understand the significance of time = 0 when referring to how the equations of Relativity lead to a singularity at the Big Bang?   Time did not exist prior to the Big Bang, at least not the way we measure or understand it. 

Quote from: PmbPhy
In any case just because something is not falsifiable doesn't mean that it doesn't belong in science. The falsifiability criteria was proposed by Karl Popper but not all physicists/philosophers think that it should be adhered to. See:
https://en.wikipedia.org/wiki/Falsifiability#Criticisms
This is ludicrous.   Falsifiability simply implies testability.  I am aware that there are string theorists and other scientists who challeng Popper, mainly because they believe falsifiability is too blunt an instrument (and they enjoy whatever funding they are receiving for pursuing an untestable theory).  Fact is, testability is the hallmark of good science and the scientific method.  This is not the pseudoscience board.    Also, there are others who defend falsifiability, and do not think that simple "elegance" is enough.

See:
http://www.nature.com/news/scientific-method-defend-the-integrity-of-physics-1.16535

How could you ever change or modify or discard a theory if you don't feel that testability and falsification principles are important? 

https://en.wikipedia.org/wiki/Scientific_method

"Properties of scientific inquiry
Scientific knowledge is closely tied to empirical findings, and can remain subject to falsification if new experimental observation incompatible with it is found. That is, no theory can ever be considered final, since new problematic evidence might be discovered. If such evidence is found, a new theory may be proposed, or (more commonly) it is found that modifications to the previous theory are sufficient to explain the new evidence. The strength of a theory can be argued to be related to how long it has persisted without major alteration to its core principles."





Quote from: Mordeth
  The Big Bang, by definition, was a singularity.
Quote from: Pmb
You're wrong. It certainly isn't how the Big Bang was/is defined and there's no supporting evidence that there was a singularity

Well, you should update Wikipedia, and just about every other source on planet Earth that refers to the Big Bang "beginning" as having a singularity (which simply means WE DON'T KNOW):

https://en.wikipedia.org/wiki/Big_Bang

"The Big Bang theory is the prevailing cosmological model for the universe from the earliest known periods through its subsequent large-scale evolution.[1][2][3] It states that the universe expanded from a very high density state,[4][5] and offers a comprehensive explanation for a broad range of observed phenomena, including the abundance of light elements, the cosmic microwave background, large scale structure, and Hubble's Law.[6] If the known laws of physics are extrapolated beyond where they are valid, there is a singularity."  (emphasis mine)

Quote from: Mordeth
There is no event prior to The Big Bang that is observable, ...
Quote from: PmbPhy
Yet. Physical cosmology isn't that old. While cosmology as a science dates back about 100 years our real understanding of the universe is in its infancy. We've only just begun to start applying modern methodology to the study of it.  Those answers might not be forth coming for hundreds of years as far as we know.

I am not sure what the point of this statement is Pmb.   It is common sense that many things are not known "yet".   The OP did not ask about what is "yet" to be known.  What we know is that we know absolutely NOTHING relating to anything that could or might have happened prior to 10^-43 seconds (Planck time).  This is the earlier period of time that our equations are at all useful to describe anything.  Nothing is known from t=0 to Planck time.  And anything prior to t=0 is BY DEFINITION not observable. Now take the velocity of light and multiply by Planck time and you get Planck length (10^-35meter).  No distance less than this can be defined.  It is thought of as a singularity in SPACE and in TIME.  Physicists have made many efforts to avoid this "singularity" at the "beginning", and quite frankly I don't blame them.  The fact is though, we simply do not know what happened prior to t=0 or even t=10^-43 seconds and it is unlikely we ever will.  The calculations that result from General Relativity being applied to the so called "origin" of the universe create a finite beginning of both time and space.  I don't even know how you can argue against this.

Quote from: Mordeth
We can go even further with this and state that anything that may have happened prior to The Big Bang is not even relevant.  The state of the Universe today was the result of the conditions OF the Big Bang, not before.  Period.
Quote from: PmbPhy

Please keep in mind that just because you're not interested in something it doesn't mean that the rest of the world isn't. Some scientists mind be very interested in Pre-Big Bang Scenarios as possible theories and couple that with the notion that there might be other universes out there in parallel to ours. And if we can build an Einstein-Rosen Bridge (aka wormhole) we can have access to those universes.

Other universes?  What other universes?   Which forum are we in again?


Quote from: PmbPhy


You're simply ruling out a lot of ideas just because you haven't thought of them. Suppose it's possible to create wormholes. The it'd be possible to go back in time. If we could do that and the universe didn't start out at a singularity then it might be possible to tunnel into the universe prior to the Big Bang with a micro wormhole and use it as a probe for information. Is that possible? I don't know either way. But that doesn't mean that we should not consider such things.

I have certainly thought of them.  Please do not tell me what I have thought of or not thought of.  But conjecturing over tunnelling to other universes when there is not even evidence of other universes is speculation of the highest order, and belongs in the new theory section, not here.
Title: Re: What was before the big bang?
Post by: Mordeth on 14/08/2015 22:03:57
Mordeth, your last post contains a number of dogmatic statements, the upshot of which seems little better than saying something like "God created it, so there's nothing to talk about".

My own feeling is that if there is something in nature we don't understand it is worth looking at to see if there is any insight to be had. You say:

Quote
The Big Bang, by definition, was a singularity.

That might be a good place to start, but you would need to say what you understand by a singularity.

Hi Bill,

In the context of the Big Bang theory, a singularity is the point where density and temperature become infinite (or at least beyond the scope of our equations) and general relativity breaks down.  We can no longer explain what happened.  "Hot and dense" is a another way to describe it, but I feel is a gross understatement of how we understand it . Time literally "began" at this point, at least how we define it.  This is, the "Big Bang".   

See here:
https://en.wikipedia.org/wiki/Lambda-CDM_model

"The model includes a single originating event, the "Big Bang" or initial singularity, which was not an explosion but the abrupt appearance of expanding space-time containing radiation at temperatures of around 1015 K."
Title: Re: What was before the big bang?
Post by: PmbPhy on 15/08/2015 02:23:58
Quote from: Wikipedia
As you know, The Big Bang model itself was built on General Relativity.  And the results suggest a singularity of infinite density at the so called "beginning".
That's not true at all. While there could be a solution where there's a singularity there need not be one. As I've always held since a professor pointed this out to me in the late 80s, the singularity is merely an extrapolation. See: https://en.wikipedia.org/wiki/Big_Bang
Quote
If the known laws of physics are extrapolated beyond where they are valid, there is a singularity.
...
Extrapolation of the expansion of the universe backwards in time using general relativity yields an infinite density and temperature at a finite time in the past. This singularity signals the breakdown of general relativity and thus, all the laws of physics. How closely we can extrapolate towards the singularity is debated[/u]—certainly no closer than the end of the Planck epoch. This singularity is sometimes called "the Big Bang", but the term can also refer to the early hot, dense phase itself,[22][notes 1] which can be considered the "birth" of our universe.
First of all we don't really know if there was a big bang at all. There no big-bang cosmological models that are consistent with current observations. I.e. they don't extrapolate back to a singularity but rebound. One such no big-bang is referred to as the bounce model. See: https://en.wikipedia.org/wiki/Big_Bounce

Quote from: Mordeth
The term singularity here is simply another way to say WE DON'T KNOW and that the physics break down.  Which has been my entire point.  If we could focus our past light cone, it would be possible to prove the singularity.
That's not what the term singularity means. The proper definition is found here:
https://en.wikipedia.org/wiki/Initial_singularity
Quote
The initial singularity was the gravitational singularity of infinite density thought to have contained all of the mass and spacetime of the Universe before quantum fluctuations caused it to rapidly expand in the Big Bang and subsequent inflation, creating the present-day Universe.

Quote from: Mordeth
Time did not exist prior to the Big Bang, at least not the way we measure or understand it. 
I've already explained why that's not necessarily correct. Even if the pre-big bang scenario can't be falsified it doesn't mean that it didn't occur. And as my GR prof at MIT told me - the universe might be infinitely old.[/i[

Quote from: Mordeth
In any case just because something is not falsifiable doesn't mean that it doesn't belong in science. The falsifiability criteria was proposed by Karl Popper but not all physicists/philosophers think that it should be adhered to. See:
https://en.wikipedia.org/wiki/Falsifiability#Criticisms
This is ludicrous.
[/quote]
Up to this point you appeared to me to be just another member with a few misconceptions about the big bang theory and the philosophy of science. However when you start using terms like "ludicrous" then you're violating forum rules. In particular: http://www.thenakedscientists.com/forum/index.php?topic=8535.0
Quote
Keep it friendly 
 
Do not use insulting, aggressive, or provocative language.

If you feel another forum user is using insulting language, seek to calm things down, or if that fails, report the matter to the moderators.  Under no circumstances should you seek to trade insults, or make accusatory remarks to that, or any other, forum user.

Show respect to other forum users.  In particular, there are times when forum users might post about delicate personal issues.  Please refrain from trivialising or making inappropriate remarks, or remarks that might embarrass the poster.

The term ludicrous is a pejorative. See:
http://www.merriam-webster.com/dictionary/ludicrous
Quote
1:  amusing or laughable through obvious absurdity, incongruity, exaggeration, or eccentricity

2:  meriting derisive laughter or scorn as absurdly inept, false, or foolish
all of which mean that the term is pejorative.

Physics does use concepts which aren't falsifiable such as the many worlds interpretation of quantum mechanics and the cosmological principle which is one of the fundamental postulates of the big bang.

If a theory is not falsifiable all it means is that we can't construct a test which will prove that it's wrong. However the theory may become more and more acceptable and that's how some postulates which are not falsifiable remain important, the cosmological principle being one of the most obvious ones that I can think of off hand.

But this thread is not about that subject and there's plenty of literature on the internet if you wish to learn more about it. For example; there's a talk about this on pbs at: http://www.pbs.org/wgbh/nova/blogs/physics/2015/02/falsifiability/

In any case I don't know string theory and as such I don't know whether it is or isn't falsifiable. There's an article on this subject that I haven't read at http://arxiv.org/abs/gr-qc/9911104

What you haven't noticed yet is that the Big Bang theory itself isn't falsifiable because one of the postulates that it's based on, i.e. the cosmological principle isn't falsifiable. That postulate states that the distribution of matter in the universe is both uniform and isotropic. However this can't be falsified because we'll never be able to determine whether it's valid or not. We can only observer matter out to the particle horizon.

In any case if you've assumed that I don't think that falsifiability should be part of science then you've assumed incorrectly.

Quote
https://en.wikipedia.org/wiki/Scientific_method
There's never a need to explain the scientific method to a professional physicist with 30 years experience. You need to learn a lot more about the role of falsifiability and why philosophers of science criticize its usage in science.

Quote from: Mordeth
Please do not tell me what I have thought of or not thought of. 
Alright. That's enough. You've proven to be too rude for me to want to converse with again. You're far too rude for my taste.
Title: Re: What was before the big bang?
Post by: Bill S on 15/08/2015 23:20:06
Quote from: Mordeth
In the context of the Big Bang theory, a singularity is the point where density and temperature become infinite (or at least beyond the scope of our equations) and general relativity breaks down.

There is a vast difference between “become infinite” and “beyond the scope of our equations”.

Something can become beyond the scope of our equations; but something “finite” cannot become “infinite”. 

Quote
Time literally "began" at this point, at least how we define it.

Here again you are, perhaps, talking about two different things.  Are “time”, and “time as we define it” necessarily the same thing?

One thing we do know about time is that it is what we use to measure change.  If there was no time before the BB, how could the universe change from not being to being.

I’m not saying that there was time before the BB, just looking at/for ideas.
Title: Re: What was before the big bang?
Post by: evan_au on 17/08/2015 00:07:24
Quote from: Bill S
Something can become beyond the scope of our equations; but something “finite” cannot become “infinite”.

In quantum physics, infinities pop up in a number of inconvenient places, including some where the answers are known to be finite. One example is in the Casimir Effect (https://en.wikipedia.org/wiki/Casimir_effect#Vacuum_energy):
Quote from: Wikipedia
Summing over all possible oscillators at all points in space gives an infinite quantity. Since only differences in energy are physically measurable... this infinity may be considered a feature of the mathematics rather than of the physics. This argument is the underpinning of the theory of renormalization.

In the vicinity of a gravitational field, physicists have not (yet) discovered the right mathematical incantations that will make the infinities go away, producing the familiar finite answer that we experience in the laboratory.
Title: Re: What was before the big bang?
Post by: PmbPhy on 17/08/2015 00:23:57
Quote from: Bill S
Something can become beyond the scope of our equations; but something “finite” cannot become “infinite”.
I disagree. Before the Big Bang there was no matter in the universe since there was no universe. If the universe has a flat geometry it means that space is flat and therefore infinite. If that's the case, and assuming that cosmological principle is true (universe is homogeneous and isotropic) then there's an infinite number of atoms.
Title: Re: What was before the big bang?
Post by: Mordeth on 17/08/2015 15:24:27
Quote from: Mordeth
In the context of the Big Bang theory, a singularity is the point where density and temperature become infinite (or at least beyond the scope of our equations) and general relativity breaks down.

Quote from: Bill S
There is a vast difference between “become infinite” and “beyond the scope of our equations”.

Something can become beyond the scope of our equations; but something “finite” cannot become “infinite”.


Hi Bill.  The math describing these events is very complex.  General Relativity predicts singularities, and these can be defined different ways depending on a number of factors.  In simple terms, a singularity is considered "geodesic incompleteness".  This is a fancy way to say infinite curvature.  At a singularity, the path of light cannot be extended any further in proper time and spacetime simply comes to an end.  Even a change in coordinates produces the same result.  A "hole" in spacetime.   Normally, a free falling particle follows a geodesic, which is simply a straight line through spacetime. At a singularity, the geodesic stops and the particle, for lack of a better term, vanishes.  This is where General Relativity breaks down, which essentially and unfortunately for us makes it an incomplete theory.  It cannot describe or predict what happens at the singularity.

By the way, a simple way to make something infinite from something finite is as such:  Take a 1 meter length of rope.  It is finite in length. Now connect the ends of the rope.  You have just created an infinite loop.

Quote from: Mordeth
Time literally "began" at this point, at least how we define it.

Quote from: Bill S
Here again you are, perhaps, talking about two different things.  Are “time”, and “time as we define it” necessarily the same thing?

One thing we do know about time is that it is what we use to measure change.  If there was no time before the BB, how could the universe change from not being to being.

I’m not saying that there was time before the BB, just looking at/for ideas.

The age of the universe is presently measured to be 13.798 billion years, plus or minus 37 million years.    This is the beginning of time Bill, as_we_define it. According to the International Astonomical Union, the age is defined as the duration of the Lambda-CDM expansion.  Put another way, it is the elapsed time since the Big Bang.   This is the beginning of time by our own definition.  I don't know how else to explain it. 

From Planck time (10^-43 seconds) until now, we feel we have a OK understanding of how things came to be.  Research the Epochs.  If we could directly observe the cosmic neutrino backround, we could know more of the events from Planck time to the "dark ages". 

However, from t=0 to Planck time, we know nothing.  This is the "singularity".   It simply means we don't know and likely can't know IN THIS CONTEXT.  If we could focus our past light cone we could maybe prove an actual singularity. 

Before t=0, we know less than nothing.  We can't even properly speculate, as no event from before t=0 is observable or even can be observable.    Quantum fluctuations of "space foam"?  Cyclic Big Bang universes?  Loop quantum gravity? How about from nothing at all?  Your guess is as good as mine.   The problem is that anytime you answer the question of what came before, you will also face the same dilemna:   Well Bill, what came before that?    It will always be TURTLES Bill, Turtles! All the way down.  And "from nothing" models don't define where the energy came from.  So it is the same two answers every time. 

The Big Bang model does not attempt to describe events prior to 10^-43 seconds.  It does not know what happened from t=0 to t=10^-43 seconds (singularity, hot and dense, etc), and anything before t=0 is pure conjecture.  As I have said here better to simply say we DO NOT KNOW.  This does not imply we should not still ask the question.
Title: Re: What was before the big bang?
Post by: PmbPhy on 17/08/2015 15:53:30
Quote from: Mordeth
General Relativity predicts singularities, ...
Bill - I've already explained to him, that's an extrapolation of the use of GR back to the Big Bang. I.e. only if the known laws of physics are extrapolated beyond where they are valid is there a singularity. See: https://en.wikipedia.org/wiki/Big_Bang

As Ohanian and Rufini explain in Gravitation and Spacetime - Second Edition (2013) page 466
Quote
The absence of a singularity at the first moment of the Big Bang would seem to be in contradiction with singularity theorems that have been proved for the solutions of Einstein's equations. But these theorems deal only with the classical regime; that can be circumvented in the very early universe, where matter is in the forum of quantum fields and where geometry is quantized. Furthermore, the presence of a positive cosmological constant can lead to a violation of the energy condition for the Hawking-Penrose theorem even in the classical regime. As we will see, in the inflationary epoch of the early universe, the cosmological constant was large and positive, and singularity theorems were inapplicable. .... Thus, the question of whether there is an actual singularity at the first moment of the Big Bang still remains open.
An analogy has been made with biology. See Did The Universe Really Begin With a Singularity?
http://profmattstrassler.com/2014/03/21/did-the-universe-begin-with-a-singularity/
Quote
Did The Universe Really Begin With a Singularity?

Posted on March 21, 2014 | 307 Comments
 
Did the universe begin with a singularity?  A point in space and/or a moment in time where everything in the universe was crushed together, infinitely hot and infinitely densely packed?

Doesn’t the Big Bang Theory say so?

Well, let me ask you a question. Did you begin with a singularity?

Let’s see. Some decades ago, you were smaller. And then before that, you were even smaller. At some point you could fit inside your mother’s body, and if we follow time backwards, you were even much smaller than that.

If we follow your growth curve back, it would be very natural — if we didn’t know anything about biology, cells, and human reproduction — to assume that initially you were infinitesimally small… that you were created from a single point!

But that would be wrong. The mistake is obvious — it doesn’t make sense to assume that the period of rapid growth that you went through as a tiny embryo was the simple continuation of a process that extends on and on into the past, back until you were infinitely small.  Instead, there was a point where something changed… the growth began not from a point but from a single object of definite size: a fertilized egg.

The notion that the Universe started with a Big Bang, and that this Big Bang started from a singularity — a point in space and/or a moment in time where the universe was infinitely hot and dense — is not that different, really, from assuming humans begin their lives as infinitely small eggs. It’s about over-extrapolating into the past.
Title: Re: What was before the big bang?
Post by: Bill S on 17/08/2015 18:41:17
Quote from: Pete
I disagree. Before the Big Bang there was no matter in the universe since there was no universe.


Pete, this is a rhetorical tautology.  Of course it must be true that if there is no universe, there is no matter in it.  However this is different from saying if there is no universe, there is no matter; because that assumes that we know what conditions were before the BB, which we don’t. 

 
Quote from: Pete
If the universe has a flat geometry it means that space is flat and therefore infinite.
 

We have been here before; and, unless you are Lewis Carroll’s Bellman, repetition does not guarantee veracity.  [:)]

I accept that this is true, but only within the mathematical usage of “infinite” which does not even take account of Cantor’s “Absolute infinity”; which, you will recall, even he thought might be equated with “God”.

How much sense does it make to say: There is no overarching infinity, but infinities are infinite, so there must be an “Absolute infinity” which doesn’t exist, anyway?   

Quote from: Pete
If that's the case, and assuming that cosmological principle is true (universe is homogeneous and isotropic) then there's an infinite number of atoms.

Even if we assume that the multiplicity of assumptions involved here is acceptable; and I think we have to if we are to make any progress in cosmology; then you have still missed the point of the nature of infinity. 

If the number of atoms is infinite, it has always been infinite.  If you are arguing that this quantity of atoms is infinite, then it must always have been infinite.  The alternative is that it started as a finite quantity, in which case it is still increasing. To reach infinity would require infinite time which, if there were such a thing, has certainly not passed since the BB. 

BTW, Mordeth, I'm not ignoring you; just out of time.
Title: Re: What was before the big bang?
Post by: Bill S on 17/08/2015 21:22:54
Quote from: Mordeth
The age of the universe is presently measured to be 13.798 billion years, plus or minus 37 million years.    This is the beginning of time Bill, as_we_define it. According to the International Astonomical Union, the age is defined as the duration of the Lambda-CDM expansion.  Put another way, it is the elapsed time since the Big Bang.   This is the beginning of time by our own definition.  I don't know how else to explain it.

Mordeth, I don't know what your scientific background is, but you certainly possess one quality common to many scientists and mathematicians: the ability to answer a question with a perfectly valid (and often detailed) answer to a different question.  [:)]

Your answer doesn't touch on the question as to how nothing could even begin change into something without time in which to initiate change.
Title: Re: What was before the big bang?
Post by: Mordeth on 18/08/2015 01:03:14
Quote from: Mordeth
The age of the universe is presently measured to be 13.798 billion years, plus or minus 37 million years.    This is the beginning of time Bill, as_we_define it. According to the International Astonomical Union, the age is defined as the duration of the Lambda-CDM expansion.  Put another way, it is the elapsed time since the Big Bang.   This is the beginning of time by our own definition.  I don't know how else to explain it.

Mordeth, I don't know what your scientific background is, but you certainly possess one quality common to many scientists and mathematicians: the ability to answer a question with a perfectly valid (and often detailed) answer to a different question.  [:)]

Your answer doesn't touch on the question as to how nothing could even begin change into something without time in which to initiate change.

Hi Bill,

I don't know.  Nor does science. I hope that answers your question directly.  What I say below is only an extension of the words: "I don't know".

In Big Bang cosmology, t=0 is not properly defined or understood.  All we understand are causes and events from Planck time forward (t>10^-43 seconds).  The models derived from General Relativity do not even describe the Big Bang as an event, so by definition it needs no cause.  An event in GR can only occur in space-time at t>0.  In fact, space-time itself only exists for t>0.  Understand the significance of this? There is no explanation, and likely won't ever be for causes at t<0 that impact events at t>0, because we cannot define t=0 in this scenario (The Big Bang). GR breaks down, and unfortunately it is all we have.   t<0 is a boundary for modern science and t=0 is not understood. A quantum theory of gravity might help describe what happened before Planck time though, when all the forces were likely united, but before that we likely will never know.  And the answer today is we simply don't know.  So for us, time began at the Big Bang.

I don't know how something can change without time in which to initiate the change (we could talk about quantum entanglement if you prefer. or the Casimir effect, or Hawking radiation), nor can I (or anyone) explain why or how our observable universe was initially created.  If I did properly, I might win myself a Nobel prize.  All we can do here in this context is explain when.  And that was 13.798 billion years ago, give or take a few 10s of million of years.  And we can explain what happened and how and why since "then".  So by our own definitions, this is when time began for our space-time.

Is your question a philosophical or metaphysical one?  I guess I am confused as to your intent.   Are you trying to convince me of something or are you honestly trying to understand something?

Best regards.
Title: Re: What was before the big bang?
Post by: Mordeth on 18/08/2015 01:17:39
Hi Pmb,

I sincerely apologize for my puerile use of the word ludicrous in response to an earlier post of yours. I should not have used this word and I retract it.  You are a committed member of this forum and clearly devoted to science.  I was out of line.
Title: Re: What was before the big bang?
Post by: PmbPhy on 18/08/2015 03:41:32
Hi Pmb,

I sincerely apologize for my puerile use of the word ludicrous in response to an earlier post of yours. I should not have used this word and I retract it.  You are a committed member of this forum and clearly devoted to science.  I was out of line.
Apology accepted. [:)]  And thank you for that clearly sincere apology. I admire people who are able to correct themselves when they recognize that they've made a mistake as you did here. My compliments.

By the way. A good friend of mine is an authority in general relativity. I've discussed the existence of an initial singularity with him and he noted that there is no simple answer to the question on singularities in the solution to Einstein's field equations. In cosmology, everything depends on the class of models one adopts. He mentioned that he questioned the usual assumption that one should start by assuming that the sources are homogeneous and isotropic and wrote to Penrose about it. I guess they know each other. He noted that if this assumption is dropped and we insert some hierarchical source into the field equations, we don't have any idea what GR would predict.

He told me that others have questioned the assumption that the torsion can be set = 0. If we drop that assumption and include torsion, there are non-singular solutions to the field equations. These are only two examples, but I hope they are enough to show that answers to such questions depend on the class of models one accepts.
Title: Re: What was before the big bang?
Post by: PmbPhy on 18/08/2015 04:05:36
Quote from: Bill S
Pete, this is a rhetorical tautology.  Of course it must be true that if there is no universe, there is no matter in it.  However this is different from saying if there is no universe, there is no matter; because that assumes that we know what conditions were before the BB, which we don’t. 
I agree. However it is quite conceivable. And in that case it provides a example of something infinite coming from something finite. A good example is the existence of, say, electrons. The early universe was too hot to have particles such as electrons or atoms. Eventually it cooled off enough for it to happen. If the universe has zero spatial curvature, which is quite possible and is actually looking that way, then, as will all models, its size started off with zero and in an instant grew to infinite. Then when it cooled off enough an infinite number of electrons, atoms, protons etc were created.

Quote from: Bill S
We have been here before; and, unless you are Lewis Carroll’s Bellman, repetition does not guarantee veracity.  [:)]
Come on, Bill. I can hardly be expected to recall what I've said to you in the past or what you remember. And you're not the only one reading this. Its for those reasons I state it, especially since I have to in order to make clear what it is that I'm explaining.

Quote from: Bill S
I accept that this is true, but only within the mathematical usage of “infinite” which does not even take account of Cantor’s “Absolute infinity”; which, you will recall, even he thought might be equated with “God”.
Who's Cantor and what does God have to do with any of this?

Quote from: Bill S
Even if we assume that the multiplicity of assumptions involved here is acceptable; and I think we have to if we are to make any progress in cosmology; then you have still missed the point of the nature of infinity. 
I hardly ever miss a point being made or that has been made.

Quote from: Bill S
If the number of atoms is infinite, it has always been infinite.
Cosmology and particle theory is inconsistent with your assertion. What is your justification for this speculation of yours?

Quote from: Bill S
If you are arguing that this quantity of atoms is infinite, then it must always have been infinite.  The alternative is that it started as a finite quantity, in which case it is still increasing. To reach infinity would require infinite time which, if there were such a thing, has certainly not passed since the BB. 
From what you just said here it's clear to me that you don't understand the physics of the early universe. The first atoms, i.e. hydrogen, didn't form until the universe had cooled down enough which was 377,000 years after the Big Bang. This epoch of the early universe is called recombination.

If I were you I'd get a book and read about it. You an download one at http://bookos-z1.org/  An Introduction to Modern Cosmology - 2nd Ed. by Andrew Liddle would be a good text for you. In fact I myself am studying it. While I know all the matter in it I haven't memorized it nor have I worked with the equations all that much. That's why I'm using it. Ohanian and Rufini is good too but more mathy.

First start with these:
https://en.wikipedia.org/wiki/Chronology_of_the_universe
http://www.physicsoftheuniverse.com/topics_bigbang_timeline.html
Title: Re: What was before the big bang?
Post by: Bill S on 19/08/2015 16:38:30
Quote from: Pete
. A good example is the existence of, say, electrons. The early universe was too hot to have particles such as electrons or atoms. Eventually it cooled off enough for it to happen. If the universe has zero spatial curvature, which is quite possible and is actually looking that way, then, as will all models, its size started off with zero and in an instant grew to infinite. Then when it cooled off enough an infinite number of electrons, atoms, protons etc were created.

Zero to infinite is a different thing from finite to infinite.  Neither zero nor infinity is a finite number.  What do you have to do to zero to make it infinite?  How long would that take at any given rate?

On an earlier visit to the suggestion that electrons provided an example of finite objects that could become infinite, we never reached a conclusion.  For clarity, can we establish if we are saying that an electron is a finite object that somehow evolves an infinite field?  Or are we looking at a pre-existing infinite field, of which each electron is a specific excitation?

Quote from: Pete
Who's Cantor and what does God have to do with any of this?

I will not insult your education be assuming that this is anything other than the verbal equivalent of peremptory wave of the hand, intended to dismiss the subject.

Quote from: Pete
I hardly ever miss a point being made or that has been made.

Whilst I will not argue with that, it does not necessarily lead to a logical sequitur that would say: “Therefore I can’t possibly have missed this point”.

Quote from: Pete
 
Quote from: Bill S
If the number of atoms is infinite, it has always been infinite.
Cosmology and particle theory is inconsistent with your assertion. What is your justification for this speculation of yours?

 Most of the scientists I have talked to agree that a finite object would require infinite time in order to become infinite.  I appreciate that scientific veracity is not a matter of popular vote, but the opinions of experts must carry some weight, and as a rational being I tend towards those views that make sense to me.

Quote from: Pete
From what you just said here it's clear to me that you don't understand the physics of the early universe.


Thank you for highlighting my ignorance, which I neither deny nor excuse.  The point I was trying to make, however, has nothing to do with the time at which hydrogen atoms formed, or with Big Bang nucleosynthesis in general.  It addresses, rather, such issues as: If the Universe is infinite, has it always been infinite?

Thanks for the links, I will certainly follow them when time permits.
Title: Re: What was before the big bang?
Post by: PmbPhy on 19/08/2015 16:53:20
Quote from: Bill S
I will not insult your education be assuming that this is anything other than the verbal equivalent of peremptory wave of the hand, intended to dismiss the subject.
I don't know what you're implying by that. I merely ask a simple question because I didn't know the answer. Please answer it.
Title: Re: What was before the big bang?
Post by: Bill S on 19/08/2015 19:09:43
Quote from: Pete
I don't know what you're implying by that. I merely ask a simple question because I didn't know the answer. Please answer it.

I didn't want to risk insulting you by assuming that you really didn't know who Cantor was.  Perhaps the quickest explanation is a link.

 https://en.wikipedia.org/wiki/Absolute_Infinite
Title: Re: What was before the big bang?
Post by: PmbPhy on 20/08/2015 17:20:37
Bill - Cantor is wrong. He wasn't a cosmologist and had no idea of the concepts that modern physics would bring to the subject. I'll give an example below.

However, the subject of infinity is something I've gone over with you countless times in the past and as such I've said all that I can say on the subject. Let's just agree to disagree, shall we? If you've forgotten what I've said then let me know and I'll repeat it for you. Otherwise I have nothing else to add. What I've explained is quite well known and standard stuff in cosmology. Let me give you one last comment and that will be it.

A property of inflation is explained in appendix A of Inflationary Universe by Alan Guth. The author explains the hypothesis that the total energy of the universe is zero and always has been. If you wish to read it then it's online at my website at:
http://home.comcast.net/~peter.m.brown/ref/guth_grav_energy.pdf
and at Wikipedia at: https://en.wikipedia.org/wiki/Zero-energy_universe

Suppose the universe started out with a Big Bang and at that moment it went from non-existence to being infinite in size. The total positive mass-energy was exactly balanced by negative gravitational potential energy. The positive mass-energy was in the form of an infinite number of elementary particles, the mass-energy also being infinite.
Title: Re: What was before the big bang?
Post by: Bill S on 20/08/2015 17:48:07
Quote
Suppose the universe started out with a Big Bang and at that moment it went from non-existence to being infinite in size. The total positive mass-energy was exactly balanced by negative gravitational potential energy. The positive mass-energy was in the form of an infinite number of elementary particles, the mass-energy also being infinite.

I do not argue, and never have argued, with the concept of instantaneous change from zero to infinite.  I see no way in which it could be achieved, and intend following your link in the hope of finding something convincing. Unless/until that happens, I will continue neither to argue for or against.

What I do argue against is the claim that something finite can become infinite in a finite length of time.   As always, I am willing to be convinced.
Title: Re: What was before the big bang?
Post by: Mordeth on 20/08/2015 18:42:17
Hi Bill,

Please see reply #23 for a question I posted to you.

Title: Re: What was before the big bang?
Post by: PmbPhy on 20/08/2015 19:48:33
Quote from: Bill S
I do not argue, and never have argued, with the concept of instantaneous change from zero to infinite.  I see no way in which it could be achieved, and intend following your link in the hope of finding something convincing. Unless/until that happens, I will continue neither to argue for or against.

What I do argue against is the claim that something finite can become infinite in a finite length of time.   As always, I am willing to be convinced.
You wrote both:

I do not argue, and never have argued, with the concept of instantaneous change from zero to infinite.

What I do argue against is the claim that something finite can become infinite in a finite length of time.

Why do you not object to something becoming infinite in an instant but not in a finite length of time? In any case that subject is off topic for this thread and I don't want to discuss it again myself.
Title: Re: What was before the big bang?
Post by: Bill S on 20/08/2015 21:01:48
Quote from: Pete
Why do you not object to something becoming infinite in an instant but not in a finite length of time? In any case that subject is off topic for this thread and I don't want to discuss it again myself.

I respect your wish not to discuss this, but I also try to answer questions.

"something becoming infinite in an instant" is a misinterpretation of what I said.

Zero (which is not something) to infinite, is quite different from something finite to infinity. 

Something that happens instantly, arguably, does not involve the passage of/through time, so infinite time may not be involved.
Title: Re: What was before the big bang?
Post by: PmbPhy on 21/08/2015 00:45:42
Quote from: Bill
I respect your wish not to discuss this, ...
Thank you my friend.

Quote from: Bill
"something becoming infinite in an instant" is a misinterpretation of what I said.
What you said was, and I quote - instantaneous change from zero to infinite which is precisely what it means to become infinite in an instant. So how is that a misinterpretation of what you said?

Quote from: Bill
Zero (which is not something) to infinite, is quite different from something finite to infinity. 
That's where you're quite wrong since the quantity zero is a finite quantity and is in the set of all finite numbers. If you thought otherwise then you have a bit of a problem with your knowledge of math. No offense intended of course.
Title: Re: What was before the big bang?
Post by: Bill S on 21/08/2015 03:12:27
Quote from: Mordeth
I don't know.  Nor does science. I hope that answers your question directly.  What I say below is only an extension of the words: "I don't know".

Are you saying:  “The Universe came from nothing, but we have no idea how something could possibly come from nothing.”?

I hope you will agree that this is very different from saying: “The Universe clearly exists, but we don’t know how or why.”

Quote from: Mordeth
The models derived from General Relativity do not even describe the Big Bang as an event, so by definition it needs no cause.

I would strongly disagree with that statement.  Had you said:  The models derived from General Relativity do not even describe the Big Bang as an event, so we will not find a cause in GR; I would not have objected.  I would argue that the Universe needs a cause, even if we cannot agree as to what it might have been.

Quote from: Mordeth
There is no explanation, and likely won't ever be for causes at t<0 that impact events at t>0,

That may well be true, but unless you can explain how something can emerge from nothing, and how nothing can become something without the passage of time, logic says there must have been something before the BB.  Whether we ever can, or ever will know what that something might have been, is a completely different matter.

Quote from: Mordeth
Is your question a philosophical or metaphysical one?  I guess I am confused as to your intent.   Are you trying to convince me of something or are you honestly trying to understand something?

Arguing that something must have preceded the Universe, otherwise we would not be here now, is a matter of logic.  To speculate as to what that something might have been would probably take us into philosophy or metaphysics.
Title: Re: What was before the big bang?
Post by: Bill S on 21/08/2015 03:28:19
Quote
That's where you're quite wrong since the quantity zero is a finite quantity and is in the set of all finite numbers.

Pete, you are still misinterpreting me.  What I said was " Zero (which is not something)".  I didn't say if it was finite or infinite.  In the context of our discussion it referred to the "nothing" from which the Universe is said (by some) to have emerged. 

I think you are right, this discussion might be best abandoned; but first  could we revisit an unanswered question?

Quote from: Bill S
On an earlier visit to the suggestion that electrons provided an example of finite objects that could become infinite, we never reached a conclusion.  For clarity, can we establish if we are saying that an electron is a finite object that somehow evolves an infinite field?  Or are we looking at a pre-existing infinite field, of which each electron is a specific excitation?
Title: Re: What was before the big bang?
Post by: chiralSPO on 21/08/2015 05:02:16
could we revisit an unanswered question?

Quote from: Bill S
On an earlier visit to the suggestion that electrons provided an example of finite objects that could become infinite, we never reached a conclusion.  For clarity, can we establish if we are saying that an electron is a finite object that somehow evolves an infinite field?  Or are we looking at a pre-existing infinite field, of which each electron is a specific excitation?
I like this reasoning.

Imagine a system in a state that contains zero sandwiches, but unlimited (infinite) amount of bread and cheese (in the correct proportion). Somehow, over some time interval, it all converts into an unlimited (infinite) amount of sandwiches. Apparently converting from zero (which is a finite number) to unlimited (infinite).

I think the "trick" is that is has to take an infinite amount of time for the conversion to take place. What would it mean "thinking relativistically" for it to happen simultaneously? Imagine an infinite array of proto-sandwiches converting into an infinite array of sandwiches all at the same time. There is no shared frame of reference anywhere within that array, so any definition of "now!" is meaningless. Therefore, I think that from the perspective of one (proto) sandwich, the transformation of all its fellows couldn't possibly be simultaneous, but would have to occur over an infinite amount of time.

And this is essentially what we observe: We can still see the big bang, we will always be able to see it--in some sense you could say it will always be happening, just further and further away, in a receding front (the edge of the observable universe) moving away at the speed of light (not accounting for expansion issues).
Title: Re: What was before the big bang?
Post by: PmbPhy on 21/08/2015 10:03:45
Quote from: Bill S
On an earlier visit to the suggestion that electrons provided an example of finite objects that could become infinite, we never reached a conclusion.  For clarity, can we establish if we are saying that an electron is a finite object that somehow evolves an infinite field?  Or are we looking at a pre-existing infinite field, of which each electron is a specific excitation?
An electron is a finite object.
Title: Re: What was before the big bang?
Post by: jeffreyH on 21/08/2015 11:13:36
could we revisit an unanswered question?

Quote from: Bill S
On an earlier visit to the suggestion that electrons provided an example of finite objects that could become infinite, we never reached a conclusion.  For clarity, can we establish if we are saying that an electron is a finite object that somehow evolves an infinite field?  Or are we looking at a pre-existing infinite field, of which each electron is a specific excitation?
I like this reasoning.

Imagine a system in a state that contains zero sandwiches, but unlimited (infinite) amount of bread and cheese (in the correct proportion). Somehow, over some time interval, it all converts into an unlimited (infinite) amount of sandwiches. Apparently converting from zero (which is a finite number) to unlimited (infinite).

I think the "trick" is that is has to take an infinite amount of time for the conversion to take place. What would it mean "thinking relativistically" for it to happen simultaneously? Imagine an infinite array of proto-sandwiches converting into an infinite array of sandwiches all at the same time. There is no shared frame of reference anywhere within that array, so any definition of "now!" is meaningless. Therefore, I think that from the perspective of one (proto) sandwich, the transformation of all its fellows couldn't possibly be simultaneous, but would have to occur over an infinite amount of time.

And this is essentially what we observe: We can still see the big bang, we will always be able to see it--in some sense you could say it will always be happening, just further and further away, in a receding front (the edge of the observable universe) moving away at the speed of light (not accounting for expansion issues).

Your viewpoint is very interesting. This is a difficult concept but I think you have summed it up excellently.
Title: Re: What was before the big bang?
Post by: chiralSPO on 21/08/2015 15:10:29
Your viewpoint is very interesting. This is a difficult concept but I think you have summed it up excellently.

Thank you. I hope so.Though, as you point out, this is quite difficult to conceptualize, so this may not necessarily be the best way to think about it. Hopefully the discussion can continue on this matter (no pun intended)...
Title: Re: What was before the big bang?
Post by: Mordeth on 21/08/2015 16:14:49
Quote from: Mordeth
I don't know.  Nor does science. I hope that answers your question directly.  What I say below is only an extension of the words: "I don't know".
Quote from: Bill S
Are you saying:  “The Universe came from nothing, but we have no idea how something could possibly come from nothing.”?
No, I am not saying that.  I am saying that I don't know either way, nor does science.

Quote from: Bill S
I hope you will agree that this is very different from saying: “The Universe clearly exists, but we don’t know how or why.”
Hi Bill.  This is what I am saying:  "The Universe clearly exists,  but we don't know how or why."  We can only describe events from Planck time forward.  Everything else is a guess.  I believe I am now repeating myself.



Quote from: Mordeth
The models derived from General Relativity do not even describe the Big Bang as an event, so by definition it needs no cause.
Quote from: Bill S
I would strongly disagree with that statement.  Had you said:  The models derived from General Relativity do not even describe the Big Bang as an event, so we will not find a cause in GR; I would not have objected.  I would argue that the Universe needs a cause, even if we cannot agree as to what it might have been.
Clearly the statement you quoted from me above was written in the context of General Relativity.  Why would it be necessary to write "General Relativity" twice in the same sentence?  I feel you are parsing my words in order to refute them.  If you have another, better scientific framework in which to draw conclusions in this context, then please submit it for review. 

Quote from: Mordeth
There is no explanation, and likely won't ever be for causes at t<0 that impact events at t>0,
Quote from: Bill S
That may well be true, but unless you can explain how something can emerge from nothing, and how nothing can become something without the passage of time, logic says there must have been something before the BB.  Whether we ever can, or ever will know what that something might have been, is a completely different matter.
No, "logic" does not say there must have been something before the Big Bang.  Whose "logic" are you referring to anyhow? 

Quote from: Mordeth
Is your question a philosophical or metaphysical one?  I guess I am confused as to your intent.   Are you trying to convince me of something or are you honestly trying to understand something?

Quote from: Bill S
Arguing that something must have preceded the Universe, otherwise we would not be here now, is a matter of logic.  To speculate as to what that something might have been would probably take us into philosophy or metaphysics.

This issue has been debated for millenia.  From Aristotle to Olber to present day. I always liked Olber's paradox myself (yes I know it can be responded to).  What preceded our universe is one of the most fundamental questions that one can ask.  I do enjoy a great deal both discussing it and thinking about it, as well as debating it.  However, no answers are available, and no answers may ever be available.  There is a point whereby both our science AND our logic fails us.  We can no more prove these things than we can even deduce them.   When you ask what came before our observable universe (which we DO NOT KNOW), you are then forced to ask, well what came before that, and before that, and before that.  As I have told you, it ends up with TURTLES, the whole way down.

Consider this, Bill:

We shall define the Universe as all that exists, ever has existed or ever will exist.

Therefore, either:

1)  The Universe has always existed and is infinite in time.

Or

2)  The Universe is finite in time and had a beginning.

If 1) is true, the Universe cannot have a cause, by definition. (as it is infinite in time)
If 2) is true, the Universe cannot have a cause, by definition.  (as it is all that exists so there is nothing to cause it)

So, we are foced to accept the fact that the Universe exists without a cause. Very hard to digest, I know. 

Now, let us go farther and apply this to what we can observe, measure and calculate.

a) It is possible that the Universe is finite in time and "our" Big Bang is the beggining of everything (The Universe).  In this scenario, there was no cause (see #2 above) and all of time truly started here.

b)  It is possible that the Universe is finite in time but "our" Big Bang was preceded by another event, or some chain of other events.  These events are not observable though, and are disconnected from our space-time (maybe forever).  For us then, time started here.   For the Universe though, time started before our Big Bang.  If we could see back before our Big Bang, and if we could follow every prior event to its cause (in a Universe of finite time), we would logically eventually arrive at the first event, and this event would have no cause. (see #2 above)

c) It is possible that there was no Big Bang, and that the significant evidence suggesting there was can be explained some other way.  Even so, the Universe (all that exists) either began with no cause (#2) or is infinite with no cause (#1).

d) It is possible that the Universe is infinite in time, and that our Big Bang is just one event in an infinite number of events (which we likely can never observe or deduce).  There is still no cause (#1).

Even proposals of other "universes" or "dimensions" must follow the above, as they fall into the definition of the Universe (all that exists).  In any scenario you choose, you are forced to agree that something happened without any time for it to happen (the beginning), or that it has always existed (infinite in time).  Either way, you end up with no originating cause, which is precisely where your line of thinking breaks down.   To ask for any cause for either scenario is nonsensical and a false question, as it is unanswerable without invoking the metaphysical.  To ask why would be equally nonsensical.  To ask when could possibly be answered in a Universe of finite time  and to ask where is not covered here.
Title: Re: What was before the big bang?
Post by: jeffreyH on 21/08/2015 17:04:57
Mordeth, I can see you have put a lot of thought into this. I can't see any fault in your reasoning.
Title: Re: What was before the big bang?
Post by: PmbPhy on 21/08/2015 18:17:02
Mordeth, I can see you have put a lot of thought into this. I can't see any fault in your reasoning.
I haven't read what you wrote in detail since I don't want to think about this anymore but what I do recall from what little I know, after we straightened out our disagreement, he's right on track in my humble opinion.
Title: Re: What was before the big bang?
Post by: Bill S on 21/08/2015 20:53:57
Mordeth, Jeffrey beat me to commenting on the amount of thought you have obviously put into this.  I have a lot of respect for people who think through there ideas, whether I (or scientists) agree with them, or not.  For that reason I would certainly not try to trip you up by paraphrasing what you post.  For what it is worth, I agree with much of what you say.

Quote from: Mordeth
This is what I am saying:  "The Universe clearly exists, but we don't know how or why."  We can only describe events from Planck time forward.


Let’s chalk that up as a major point on which we agree!

Quote from: Mordeth
  Everything else is a guess.

That’s an “almost agree”.  The sticking point for me is that, given that there is something now, I cannot see how there could ever have been absolutely nothing.  Had there been, there would still be nothing now; which, clearly is not the case.  What I am saying is that something must be eternal, but I agree that any attempt to say what that something might be would be just a guess.

Quote from: Mordeth
Whose "logic" are you referring to anyhow? 

This may be a bit off topic, but I think it is worth mentioning that logic is not subjective.  If we say “ your logic is different from mine”, what we are really saying is that your interpretation is different from mine; in which case, one or both of us is almost certainly misusing the precepts of logic. 

I still maintain that unless you can show, logically, how something can emerge from nothing, it is logical to claim that something must always have existed.

Quote from: Mordeth
Are you trying to convince me of something or are you honestly trying to understand something?

Certainly the latter, but possibly also the former; because I believe the best way to understand something is to try to explain it to someone else.  Practically all my notes are written as though I were explaining points to someone.  The advantage of real discussion is that the “someone” argues back.[:)]

Quote from: Mordeth
When you ask what came before our observable universe (which we DO NOT KNOW), you are then forced to ask, well what came before that, and before that, and before that.  As I have told you, it ends up with TURTLES, the whole way down.

I disagree.  If the turtle below our Universe is infinite/eternal, there are no other turtles, nor is there a “whole way down”.

Quote from: Mordeth
We shall define the Universe as all that exists, ever has existed or ever will exist.

Therefore, either:

1)  The Universe has always existed and is infinite in time.

Or

2)  The Universe is finite in time and had a beginning.

If your definition of the Universe is correct, then option 1 must be correct, unless you can show how something can come from nothing.  The nearest I have seen anyone come to that was JP.  He argued that because we are basing our reasoning on what appears to be the case in the observable Universe, we could not argue that there could not be conditions outside the Universe in which something could come from nothing.  The discussion, which ended in a succession of PMs, finished with agreement that “conditions” would have to be considered as something.

Duty calls, I’m afraid, so I’ll pick up your other points later.
Title: Re: What was before the big bang?
Post by: Bill S on 21/08/2015 22:40:45
Quote from: Mordeth
Quote from: Bill S
Had you said:  The models derived from General Relativity do not even describe the Big Bang as an event, so we will not find a cause in GR; I would not have objected.
Clearly the statement you quoted from me above was written in the context of General Relativity.  Why would it be necessary to write "General Relativity" twice in the same sentence?

The reason I used GR twice in the same sentence was to stress that GR can establish the truth of something only within GR.  The same thing might not be true, for example, in QM.

Quote from: Mordeth
What preceded our universe is one of the most fundamental questions that one can ask.

True, but do you agree that there is a difference between asking: “What came before our Universe”
and “Did anything come before our Universe”?

Quote from: Mordeth
If 2) is true, the Universe cannot have a cause, by definition.  (as it is all that exists so there is nothing to cause it)

Once again this begs the question: “Can something come from nothing?”

Quote from: Mordeth
So, we are foced to accept the fact that the Universe exists without a cause.

I disagree, we are forced to accept that we cannot identify the cause, but insisting that there is no cause is speculation.
-----------------------
Your choices a-d raise some interesting possibilities. My comments are my personal thoughts for which I make no claims to veracity.  I present them simply for discussion.

a) This is the something from nothing scenario again.


b) I’m fine with most of this, except that you take it back to an original finite event which had no cause.  This is “turtles the whole way down”, but with a turtle miraculously appearing somewhere in the column to start things off.  That doesn’t strengthen the something from nothing argument, it just pushes it back in time.

c) “It is possible that there was no Big Bang, and that the significant evidence suggesting there was can be explained some other way.  Even so, the Universe (all that exists) either began with no cause (#2) or is infinite with no cause (#1).”

I agree with the first sentence, with the proviso that you would have to come up with a good theory if you wanted to convince the majority of cosmologists that they should abandon the BB.

After that you return to the something from nothing, which we have already considered, then plunge into a tautology!  If the Universe is infinite it had no beginning, therefore no cause.  That is implicit in the definition “infinite”.

d) “It is possible that the Universe is infinite in time, and that our Big Bang is just one event in an infinite number of events (which we likely can never observe or deduce).  There is still no cause”

I know what you mean here, and, as it stands, I agree.  However as the “resident infinity crackpot” I have to suggest that you have problems that arise from an apparent misunderstanding of infinity/eternity.  For example: “infinite in time” suggests that eternity is a length of time, which it is not.  Eternity and time are completely different things and cannot realistically be mixed.  Similarly, we might talk of “in an infinite number of events”.  This suggests that infinity is a number, which it is not.

If this discussion is to continue, I would like to suggest that we consider using John Gribbin’s terminology:

Cosmos = everything that exists, or can exist.
Universe = our (in principle) observable portion of spacetime and its contents.
universe = any other universe that may, or may not, exist.
Title: Re: What was before the big bang?
Post by: Mordeth on 22/08/2015 02:13:39
Mordeth, Jeffrey beat me to commenting on the amount of thought you have obviously put into this.  I have a lot of respect for people who think through there ideas, whether I (or scientists) agree with them, or not.  For that reason I would certainly not try to trip you up by paraphrasing what you post.  For what it is worth, I agree with much of what you say.
Hi Bill,

It is apparent that you too have put much thought into this. I appreciate the discussion.

Quote from: Mordeth
Whose "logic" are you referring to anyhow? 

Quote from: Bill S
This may be a bit off topic, but I think it is worth mentioning that logic is not subjective.  If we say “ your logic is different from mine”, what we are really saying is that your interpretation is different from mine; in which case, one or both of us is almost certainly misusing the precepts of logic. 
Roughly speaking, logic is the study of the principles of correct reasoning. The rules are objective, but many forms are not. I think that Ayn Rand was correct when she referred to logic as the art of non-contradictory identification.  Know much art that is not subjective?   Please don't take this wrong, but any formal training in logic would make it clear that logic and philosophy take many forms, some of which are in opposition to each other.  Existentialism, which opposes rationalism, is based on subjective reasoning.  In fact it can be argued that codifying logic itself is subjective.  If you understood logical positivism, you would understand that one cannot ask questions that do not translate to observations.  The source of knowledge is derived from facts.   So an adherent of logical positivism would simply reject your question as meaningless, and would be logically correct, within that framework. Now,  Heisenberg, basically the creator of quantum mechanics, disagreed and had much to say on the subject.   There is signicant other critique of positivism.  This is well off track now so I will stop. The point is that the logical framework in which you ask and answer questions is as important as the scientific framework.  This was the point of my question.

Quote from: Bill S
I still maintain that unless you can show, logically, how something can emerge from nothing, it is logical to claim that something must always have existed.
Can you logically show me how something can have always existed? 



Quote from: Mordeth
When you ask what came before our observable universe (which we DO NOT KNOW), you are then forced to ask, well what came before that, and before that, and before that.  As I have told you, it ends up with TURTLES, the whole way down.
Quote from: Bill S
I disagree.  If the turtle below our Universe is infinite/eternal, there are no other turtles, nor is there a “whole way down”.
I said turtles, not turtle.  The turtle reference is a common joke in Cosmology.   It refers to the infinite regress problem Bill.  The Earth sits on the back of a turtle, says old lady.  Well what does the turtle stand on?  Another turtle of course, she answers.  And what does this turtle stand on?  It is TURTLES, THE WHOLE WAY DOWN! her final response.
Quote from: Bill S
If your definition of the Universe is correct, then option 1 must be correct, unless you can show how something can come from nothing.  The nearest I have seen anyone come to that was JP.  He argued that because we are basing our reasoning on what appears to be the case in the observable Universe, we could not argue that there could not be conditions outside the Universe in which something could come from nothing.  The discussion, which ended in a succession of PMs, finished with agreement that “conditions” would have to be considered as something.
Bill,  we already have a fundamental condition we cannot explain.  It is the alleged singularity at the origin of the Big Bang.  Our physics do not apply to it.  Nor does our logic.  And I have explained this many times.  Physics itself, math itself, breaks down.   So does our logic.

Title: Re: What was before the big bang?
Post by: Bill S on 22/08/2015 02:59:52
Quote from: Pete
Quote from: Bill S
On an earlier visit to the suggestion that electrons provided an example of finite objects that could become infinite, we never reached a conclusion.  For clarity, can we establish if we are saying that an electron is a finite object that somehow evolves an infinite field?  Or are we looking at a pre-existing infinite field, of which each electron is a specific excitation?
  An electron is a finite object.

I have never questioned that.  If this is the best answer you can find to the questions, I think we should call it a day.  We are wasting your time and mine.
Title: Re: What was before the big bang?
Post by: Mordeth on 22/08/2015 03:12:14
Quote from: Mordeth
Quote from: Bill S
Had you said:  The models derived from General Relativity do not even describe the Big Bang as an event, so we will not find a cause in GR; I would not have objected.
Clearly the statement you quoted from me above was written in the context of General Relativity.  Why would it be necessary to write "General Relativity" twice in the same sentence?

The reason I used GR twice in the same sentence was to stress that GR can establish the truth of something only within GR.  The same thing might not be true, for example, in QM.
I used the term once in the sentence, as well as throughout my post.  It is enough for every person on Earth I am guessing, except you my friend.  I will no longer comment on this subject.   

Quote from: Mordeth
What preceded our universe is one of the most fundamental questions that one can ask.
Quote from: Bill S
True, but do you agree that there is a difference between asking: “What came before our Universe”
and “Did anything come before our Universe”?
Yes, but they are related and the second question will lead to the first.

Quote from: Bill S

Once again this begs the question: “Can something come from nothing?”
Bill, this question is very deep and fundamental.  I am afraid I do not have time to give you my thoughts on this right now.  It would also significantly derail this thread.   The very definition of nothing would first have to be agreed upon, and even then the question could rightly be dismissed as non-answerable in the logical form of positivism.
Quote from: Mordeth
So, we are foced to accept the fact that the Universe exists without a cause.

Quote from: Bill S
I disagree, we are forced to accept that we cannot identify the cause, but insisting that there is no cause is speculation.
I honestly think you either did not fully read my thread or did not understand it.  There is no flaw in the logic.   You can only argue my initial definition Bill.   In both cases, there is no cause.  Please read it again.



Quote from: Bill S
b) I’m fine with most of this, except that you take it back to an original finite event which had no cause.  This is “turtles the whole way down”, but with a turtle miraculously appearing somewhere in the column to start things off.  That doesn’t strengthen the something from nothing argument, it just pushes it back in time.
Bill, with all due respect, you missed the point. 
Quote from: Bill S
After that you return to the something from nothing, which we have already considered, then plunge into a tautology!  If the Universe is infinite it had no beginning, therefore no cause.  That is implicit in the definition “infinite”.

d) “It is possible that the Universe is infinite in time, and that our Big Bang is just one event in an infinite number of events (which we likely can never observe or deduce).  There is still no cause”

I know what you mean here, and, as it stands, I agree.  However as the “resident infinity crackpot” I have to suggest that you have problems that arise from an apparent misunderstanding of infinity/eternity.  For example: “infinite in time” suggests that eternity is a length of time, which it is not.  Eternity and time are completely different things and cannot realistically be mixed.  Similarly, we might talk of “in an infinite number of events”.  This suggests that infinity is a number, which it is not.
Bill, you entirely missed the point of my post.   For an argument to be logically valid,  it must be impossible for the premise to be true and the conclusion false.   Validity itself has does not care if the premise or the conclusion are false themselves, except that a true premise cannot have a false conclusion.  That is, you could have a false premise and a false conclusion, but still be logically valid.  It would not be sound though.   Logical systems themelves though must be sound, consistent and complete.  My argument was, and is therefore logical.  You can only invalidate my argument by proving my premise to be false which leads to a false conclusion.  But if you accept my premise, you must accept my conclusion.  This is deductive reasoning.  Please re-read each word of my post.  And I understand infinity as well as the next guy...meaning not at all.  Do you have some evidence of infinity outside of mathematics?
Title: Re: What was before the big bang?
Post by: Bill S on 22/08/2015 15:26:26
Mordeth, I'm not trying to cover all outstanding points in this post; partly through lack of time, and partly because I think that trying to cover too much at a time can lead to confusion.

Back in the late 1950s I was in the audience at a Grammar School, 6th Form, debate. The topic was: “Half a loaf is better than no bread; true or false?”  The conclusion was that it was false.  This (briefly) was based on the assertion that a loaf is a mass of bread, and you can’t have half a mass.  I leave you to identify the absurdity.

Whilst I acknowledge that dissertatio gratia dissertationis can be a valuable exercise, but I am much more interested in what makes sense than in what might be semantically “correct”.

Quote from: Mordeth
Quote from: Bill
True, but do you agree that there is a difference between asking: “What came before our Universe”
and “Did anything come before our Universe”?
  Yes, but they are related and the second question will lead to the first.

True, but I think it’s quite important to take the steps in sequence and not to assume that the more fundamental question has been answered if it has not.

Quote from: Mordeth
  The very definition of nothing would first have to be agreed upon,

“Define nothing!” = “standard cop-out”.  You are capable of better than that, Mordeth. [:)]

Quote from: Mordeth
a) It is possible that the Universe is finite in time and "our" Big Bang is the beggining of everything (The Universe).  In this scenario, there was no cause (see #2 above) and all of time truly started here.

I read this and I understand it.  I have no objection to the first sentence, but the statement “there was no cause” assumes knowledge outside that which you have identified as time.  You would need to justify that assumption.


Title: Re: What was before the big bang?
Post by: PmbPhy on 22/08/2015 17:23:23
Quote from: Bill S
“Define nothing!” = “standard cop-out”.  You are capable of better than that, Mordeth. [:)]
I have to admit that I don't know what he means by this myself.
Title: Re: What was before the big bang?
Post by: timey on 22/08/2015 19:24:22
IF we can define 'nothing' as an absence of 'everything', can an infinite progression of 'everything' be considered logical?

And IF so... can it be considered logical to define 'nothing' as an infinite state?

An infinite state of nothing progressing into an infinite state of everything...
Title: Re: What was before the big bang?
Post by: Bill S on 22/08/2015 21:38:26
Quote from: Timey
IF we can define 'nothing' as an absence of 'everything', can an infinite progression of 'everything' be considered logical?

And IF so... can it be considered logical to define 'nothing' as an infinite state?

An infinite state of nothing progressing into an infinite state of everything...

Semantics, Timey, just semantics. 

How do you define anything, without defining it in terms of something?

If you try to define nothing in terms of something, you no longer have nothing to define.

Mordeth, never say I didn't offer you something to have fun with. [:)]
Title: Re: What was before the big bang?
Post by: timey on 22/08/2015 21:58:29
How do you define anything, without defining it in terms of something?

Well, if it is possible to define 'everything', then to define a state of nothing as an absence of everything answers your question.

If you try to define nothing in terms of something, you no longer have nothing to define.

This just becomes a matter of defining 'absence'. :)

In any case I was just popping back to edit my post with this:

(Edit: or perhaps I could rephrase that as:  ...An infinite state of nothing infinitely progressing into a state of everything.)
Title: Re: What was before the big bang?
Post by: Bill S on 22/08/2015 22:21:36
Quote from: Chiral
Apparently converting from zero (which is a finite number) to unlimited (infinite).

It seems I’m outnumbered, and by experts! 

However, I understand that there is no absolute agreement among mathematicians as to whether zero should be considered as a natural number.  If you define "finite" as having the cardinality of a natural number, and do not consider zero as a natural number, you are, according to this school of thought, justified in considering zero as non-finite.
Title: Re: What was before the big bang?
Post by: Bill S on 22/08/2015 22:27:55
Quote from: Timey
Well, if it is possible to define 'everything', then to define a state of nothing as an absence of everything answers your question.

I like that, Timey, but it might be wise to leave out "a state of", or someone is bound to argue that "a state" is something.
Title: Re: What was before the big bang?
Post by: timey on 22/08/2015 23:11:13
Quote from: Chiral
Apparently converting from zero (which is a finite number) to unlimited (infinite).

It seems I’m outnumbered, and by experts! 

However, I understand that there is no absolute agreement among mathematicians as to whether zero should be considered as a natural number.  If you define "finite" as having the cardinality of a natural number, and do not consider zero as a natural number, you are, according to this school of thought, justified in considering zero as non-finite.

Is that you attempting to wow me with mathematics Bill? :D

Just kidding... I agree that 0 is a bit strange...conceptually, it can be considered as both something and nothing.

Which...(I'm now adding, having read post above), does kind of detract from your comment concerning 'a state' of nothing, as nothing is already considered a state of something.

You could now argue that the state of nothing must also be absenced as part of everything to achieve a true state of nothing...but I think it was 'you' that mentioned semantics :) (chuckle)
Title: Re: What was before the big bang?
Post by: Bill S on 22/08/2015 23:21:12
Quote from: Timey
Is that you attempting to wow me with mathematics Bill? :D

Far be it from me to attempt such a thing.  As a non-mathematician I enter that arena with trepidation, but I like to try to respond, even belatedly, as in this case.
Title: Re: What was before the big bang?
Post by: Bill S on 22/08/2015 23:26:29
Quote from: Timey
...An infinite state of nothing infinitely progressing into a state of everything.

If infinity is not a number, and eternity is not a length of time, and if change requires time, how can you have any progression in infinity/eternity?
Title: Re: What was before the big bang?
Post by: Mordeth on 22/08/2015 23:37:39
Quote from: Mordeth
  The very definition of nothing would first have to be agreed upon,
Quote from: Bill S

“Define nothing!” = “standard cop-out”.  You are capable of better than that, Mordeth. [:)]
Bill,  do you believe that one can ask questions that have no translation to observations?  For example, where is infinity located? Or maybe you can ask me if the number 322 is a Marxist?     The fact is, there are meaningless questions that exist.  Perhaps it makes a person feel smart to ask them, but a true purveyor of knowledge will reject these questions as meaningless.   Your chain of reasoning, Bill, depends on the assumption of your own answer and is therefore invalid. 

And so now we derail the thread.  I will likely be contacted by the local magistrate of this forum. 

"Nothing" is simply a word, and in the context of existence itself becomes a meaningless, undefined concept. The question of non-being is self-contradictory.  Why is there something?   Do you think there is an object we can call nothing that is nothing?  If nothing is this object of nothingness, then it is not nothing, but something.   Can nothing exist?  Then it is something.  Is non-existence a concept in your mind?   A concept is something, not nothing.  Is nothing a non-existent state and therefore a state and therefore something?  Is nothing the void?   The void is something.   Can you displace nothing?  Can it be measured?   What are the attributes of nothing?  Say them, and we shall label it something.   The very act of defining nothing, makes it something.  To contemplate non-existence implies existence, so nothing is a non-existent idea in our minds.  As an idea, it is something.    How about we say <THIS> is something and <THAT> is nothing.  Well, <THAT> nothing is now something.

See where this goes?  Do you have a sufficient, satisfactory answer to why there is something rather than nothing?  I tell you that there is none.  The question itself is absurd.   The question of nothing is itself a fallacious question, as you cannot appeal to nothingness without appealing to something.    These questions are meaningless, and need not be answered.   

Quote from: Mordeth
a) It is possible that the Universe is finite in time and "our" Big Bang is the beggining of everything (The Universe).  In this scenario, there was no cause (see #2 above) and all of time truly started here.
Quote from: bill S
I read this and I understand it.  I have no objection to the first sentence, but the statement “there was no cause” assumes knowledge outside that which you have identified as time.  You would need to justify that assumption.
Bill, I defined the Universe as all that exists.  If time is finite and the Universe began at some point in the past, by definition there was no cause - as the Universe is all that exists, so what could cause it?  So no, you do not understand it.  The assumption of no cause is justified by the definition of the Universe.  The logic is valid, therefore if you accept the premise you must accept the conclusion, or it is YOU who are committing the fallacy.
Title: Re: What was before the big bang?
Post by: timey on 22/08/2015 23:46:41
Quote from: Timey
Is that you attempting to wow me with mathematics Bill? :D

Far be it from me to attempt such a thing.  As a non-mathematician I enter that arena with trepidation, but I like to try to respond, even belatedly, as in this case.

Awww Bill...that was indeed, under the circumstances of a 0, just a joke. :).  Myself, I struggle with the maths but can understand what's occurring when the concepts behind them are explained.  I guess some people just don't have a flair for the notation.  You have my sympathy, and hey...nice to speak with you!

Aha, but I see there is more... :)

Quote from: Timey
...An infinite state of nothing infinitely progressing into a state of everything.

If infinity is not a number, and eternity is not a length of time, and if change requires time, how can you have any progression in infinity/eternity?

Ok then clever clogs, how's about:

An infinite state of nothing progressed into a state of everything infinitely...

???
Title: Re: What was before the big bang?
Post by: Bill S on 23/08/2015 00:58:17
Quote from: Mordeth
"Nothing" is simply a word, and in the context of existence itself becomes a meaningless, undefined concept. The question of non-being is self-contradictory.  Why is there something?   Do you think there is an object we can call nothing that is nothing?  If nothing is this object of nothingness, then it is not nothing, but something.   Can nothing exist?  Then it is something.  Is non-existence a concept in your mind?   A concept is something, not nothing.  Is nothing a non-existent state and therefore a state and therefore something?  Is nothing the void?   The void is something.   Can you displace nothing?  Can it be measured?   What are the attributes of nothing?  Say them, and we shall label it something.   The very act of defining nothing, makes it something.  To contemplate non-existence implies existence, so nothing is a non-existent idea in our minds.  As an idea, it is something.    How about we say <THIS> is something and <THAT> is nothing.  Well, <THAT> nothing is now something.

See where this goes?  Do you have a sufficient, satisfactory answer to why there is something rather than nothing?  I tell you that there is none.  The question itself is absurd.   The question of nothing is itself a fallacious question, as you cannot appeal to nothingness without appealing to something.    These questions are meaningless, and need not be answered.

Thanks, Mordeth, I will certainly not try to better your explanation as to why there could never have been "nothing".  It doesn't exist.
Title: Re: What was before the big bang?
Post by: Bill S on 23/08/2015 01:05:58
Quote from: Timey
An infinite state of nothing progressed into a state of everything infinitely...

Sorry. You still have progress in infinity.  Maybe you could get Mordeth to formulate it for you, his perplexing verbosity has an exuberance that is quite enviable.  [:)]
Title: Re: What was before the big bang?
Post by: timey on 23/08/2015 01:28:09
Thanks, Mordeth, I will certainly not try to better your explanation as to why there could never have been "nothing".  It doesn't exist.

But surely now we have a quandary...if nothing does not exist we have no means of defining everything, therefore this is calling into question the existence of everything.  I'm quite sure my logic is not faulty!

"Nothing" is simply a word,

True, but it is also a concept...

Quote from: Timey
An infinite state of nothing progressed into a state of everything infinitely...

Sorry. You still have progress in infinity.

Ah yes my main man, how very true :), but now we find ourselves at the crux of the OP's question...
Can we say that what comes before the word 'progressed' is before the Big Bang or initial point of creation, and that the word 'progressed' is the Big Bang or initial point of creation and also the point of the initiation of the beginning of the phenomenon of time itself? 
Title: Re: What was before the big bang?
Post by: Mordeth on 23/08/2015 02:28:55
"Nothing" is simply a word,  <the word concept snipped by Timey, among other things>
Quote from: timey
True, but it is also a concept...

Hi timey,  did you intentionally snip the second part of the sentence of mine that you quoted in which I specifically stated it was a concept and then correct me and say it was a concept?   Surely there is some fundamental universal principle  that has been violated here.   Good stuff my friend!  [:)]  I shall henceforth compare this outrage to the work of another poster here who is of the opinion that terms must be stated twice in the same sentence in order to carry meaning to the reader. 

All said above in good fun. 

 
Title: Re: What was before the big bang?
Post by: timey on 23/08/2015 02:47:28
Please Mordeth accept my most abject apologies, I actually 'hate' it when people do that to me... :).  Your point is taken!

Are we arguing the same toss of the coin though?  What do you reckon to the finale?  The creation concept - "An infinite state of nothing progressed into a state of everything infinitely" - in relation to what came before the Big Bang, or the moment of creation (as I prefer) ?

Can we say that what comes before the word 'progressed' is before the Big Bang or initial point of creation, and that the word 'progressed' is the Big Bang or initial point of creation and also the point of the initiation of the beginning of the phenomenon of time itself?
Title: Re: What was before the big bang?
Post by: jeffreyH on 23/08/2015 11:07:34
Due to the fact that GR requires mass to increase, hence relativistic mass, then an infinite amount of energy is required to reach light speed for particles with rest mass. This can be applied to the escape velocity required to move away from a black hole. We also have infinite energy required to escape at the event horizon. The mathematics breaks down because of this. This al relates to movement normal or perpendicular to the imaginary surface of the event horizon. If we consider an object moving towards a black hole with a velocity that is an appreciable percentage of light speed then the effect of the tidal forces give us another problem. As they will tend to accelerate the object. Something has to prevent light speed violation outside the horizon. If the speed of the particle is approaching the speed of gravity then this may be the answer to preventing the mathematics from breaking down. The gravitation is then less effective as the speed of the particle approaches the speed of gravitation. In which case you can think of the singularity in different terms. You could also explain the coordinate nature of time and distance. Thus the big bang might not be such a mystery.
Title: Re: What was before the big bang?
Post by: timey on 23/08/2015 11:19:14
These all being points of high interest to me, I find that I 'can' easily follow the path of your logic Jeff and look forward to hearing more on this...
Title: Re: What was before the big bang?
Post by: jeffreyH on 23/08/2015 12:21:26
These all being points of high interest to me, I find that I 'can' easily follow the path of your logic Jeff and look forward to hearing more on this...

For this to be feasible you need a force carrier, the graviton. The density of force carriers would have to operate to reduce the increase in acceleration. Effectively slowing down the passage of time as well.
Title: Re: What was before the big bang?
Post by: timey on 23/08/2015 12:24:18
Yes, that 'is' an interesting notion indeed Jeff! :)
Title: Re: What was before the big bang?
Post by: timey on 23/08/2015 12:24:49
Presumably no one wishes to enter a discussion concerning my questions regarding the statement:

"An infinite state of nothing progressed into a state of everything infinitely."

Can we say that what comes before the word 'progressed' is before the Big Bang or initial point of creation, and that the word 'progressed' is the Big Bang or initial point of creation and also the point of the initiation of the beginning of the phenomenon of time itself?

This being because this will, following the path of logic, lead to a discussion regarding the part after the word 'progressed' in relation to the second law of thermodynamics and 'perhaps' call into question our current perception of the nature of the Big Bang?
Title: Re: What was before the big bang?
Post by: Mordeth on 23/08/2015 15:25:09
Please Mordeth accept my most abject apologies, I actually 'hate' it when people do that to me... :).  Your point is taken!
No problem timey.   I appreciate the discussion. 

Quote from: timey
Are we arguing the same toss of the coin though?  What do you reckon to the finale?  The creation concept - "An infinite state of nothing progressed into a state of everything infinitely" - in relation to what came before the Big Bang, or the moment of creation (as I prefer) ?
I truly wish I knew.  I easily have 200 pages of notes on this subject alone.  Many of the discrete models, like loop quantum gravity,  assume that spacetime is not fundamental, but emerges from something else.  The problem is that these discrete models tend to violate Lorentz invariance.   The Fermi observations suggest spacetime is continous, as GR predicts.  This is one of the reasons we have no quantum theory of gravity. Without a quantum theory of gravity, we may never know. 


Can we say that what comes before the word 'progressed' is before the Big Bang or initial point of creation, and that the word 'progressed' is the Big Bang or initial point of creation and also the point of the initiation of the beginning of the phenomenon of time itself?
Events have only progressed from Planck time forward.  Fundamentally, GR and most of physics only concerns itself wih events and the relationship between these events.  So we measure these relationships and develop theories to describe them.  Like the curvature of spacetime.

 GR does not describe the initial conditions as an event.  Events only occur in spacetime.  In fact, events and their relationships are what define spacetime. 


Title: Re: What was before the big bang?
Post by: Mordeth on 23/08/2015 15:58:36
Hi jeffreyH,

Many individuals misunderstand the term escape velocity.  Escape velocity only describes the initial speed at which an object can escape forever.  Objects under constant acceleration can "escape"  at virtually any speed if under propulsion.  The problem with the event horizon trapping objects forever is not necessarily related to the escape velocity equaling  c.  The problem is that once you cross the event horizon, your light cone is forever enclosed within. There is no geodesic that points outside the event horizon.   Space has become curved on itself and the future lies only at the singularity.  Once you cross the event horizon, no matter what speed you take, or which direction you  point, you can never leave.  The future direction of time has become radially bent to the singularity.  Put another way, the singularity exists in all directions you take after crossing an event horizon, regardless of any arbitrarily large speed.   All you will do is get to the singularity faster.
Title: Re: What was before the big bang?
Post by: jeffreyH on 23/08/2015 16:27:11
Hi jeffreyH,

Many individuals misunderstand the term escape velocity.  Escape velocity only describes the initial speed at which an object can escape forever.  Objects under constant acceleration can "escape"  at virtually any speed if under propulsion.  The problem with the event horizon trapping objects forever is not necessarily related to the escape velocity equaling  c.  The problem is that once you cross the event horizon, your light cone is forever enclosed within. There is no geodesic that points outside the event horizon.   Space has become curved on itself and the future lies only at the singularity.  Once you cross the event horizon, no matter what speed you take, or which direction you  point, you can never leave.  The future direction of time has become radially bent to the singularity.  Put another way, the singularity exists in all directions you take after crossing an event horizon, regardless of any arbitrarily large speed.   All you will do is get to the singularity faster.

I already know all that. I am interested in the kinetic energy necessary for escape to infinity.
Title: Re: What was before the big bang?
Post by: timey on 23/08/2015 16:45:41
Please Mordeth accept my most abject apologies, I actually 'hate' it when people do that to me... :).  Your point is taken!
No problem timey.   I appreciate the discussion. 

Can we say that what comes before the word 'progressed' is before the Big Bang or initial point of creation, and that the word 'progressed' is the Big Bang or initial point of creation and also the point of the initiation of the beginning of the phenomenon of time itself?
Events have only progressed from Planck time forward.  Fundamentally, GR and most of physics only concerns itself wih events and the relationship between these events.  So we measure these relationships and develop theories to describe them.  Like the curvature of spacetime.

 GR does not describe the initial conditions as an event.  Events only occur in spacetime.  In fact, events and their relationships are what define spacetime.

I too appreciate the discussion :)

(I have not quoted the whole of your post, Pete asked me not to quote needlessly to make for easier reading, but please know I have taken on board your other comments)

Ok, well the OP's question is allowing us room for a 'little' speculation... Perhaps?

You have alluded to the fact that GR and QM are insufficient to describe everything.  Let's ditch them for the present.

Given that you are happy with the derivation of the statement.  ie: on the basis that an 'absence' can be defined as a description of that which is not there, and 'nothing' can be defined by the existence of 'everything', which is already defined - and by the same token, because 'everything' is defined, 'nothing' cannot not exist - and that we have by definition defined a state of nothing as a concept...so long as you are happy that this now is a statement based in logic...

Could we consider 'an infinite state of nothing' as a pre Big Bang state?

Could we consider 'progressed' as the means of cross over from nothing to everything being a Big Bang or creation type scenario?
Title: Re: What was before the big bang?
Post by: Mordeth on 24/08/2015 03:48:44
Quote from: timey
I too appreciate the discussion :)

(I have not quoted the whole of your post, Pete asked me not to quote needlessly to make for easier reading, but please know I have taken on board your other comments)

Ok, well the OP's question is allowing us room for a 'little' speculation... Perhaps?

You have alluded to the fact that GR and QM are insufficient to describe everything.  Let's ditch them for the present.

Given that you are happy with the derivation of the statement.  ie: on the basis that an 'absence' can be defined as a description of that which is not there, and 'nothing' can be defined by the existence of 'everything', which is already defined - and by the same token, because 'everything' is defined, 'nothing' cannot not exist - and that we have by definition defined a state of nothing as a concept...so long as you are happy that this now is a statement based in logic...

Could we consider 'an infinite state of nothing' as a pre Big Bang state?

Could we consider 'progressed' as the means of cross over from nothing to everything being a Big Bang or creation type scenario?
Hi timey,

I understand where you are going with this, but there is a point where a discussion of physics turns to philosophy, metaphysics and opinion.  There is no quantifiable property that can be empirically measured in the scenario you ask me of.  Therefore,  no answer I give will be in the context of physics.  If spacetime is not a fundamental property, that is, if it is more than the 4 dimensional manifold that GR predicts, then we need to invoke strings, loops and other mathematical constructs to describe what it emerged from.  I am ok with that.  But beyond that, we enter a realm that numbers and math and physics do not and cannot describe.  So as I said somewhere around page 1,  a person is free to speculate.

The philosopher in me wants to believe that our Universe emerged from something.  The logician inside me denies it.  The scientist in me does not know and refuses to guess.   An infinite state of nothing, containing only the potential to  be something, and progressing to a state of something I can measure suits me, I suppose.

Do you think, in the context of philosophy and not mathematics, that the concept of infinity can be any more understood than the concept of nothingness?  Can you even conceive of something without limit?  Could the concepts of infinity and nothingness be the same thing?   Are they then indistinguishable from each other?  Equally paradoxical and otherwise inconceivable?  If infinity exists, how could it then be something other than nothing?
Title: Re: What was before the big bang?
Post by: PmbPhy on 24/08/2015 04:11:20
Quote from: Mordeth
The philosopher in me wants to believe that our Universe emerged from something.  The logician inside me denies it.
The appropriate stance for a scientist is to take the position of I don't know if the universe emerged from something or not. It's incorrect to assert that it didn't emerge because you simply don't know.

Quote from: Mordeth
  The scientist in me does not know and refuses to guess.
Nothing wrong with guesses either. Sometimes a long way down the road they might lead to something. One guess that is popular in cosmology is the cosmological principle which states that distribution of matter in the universe is both homogeneous and isotropic. But we can never know this to be true or false. However when its employed by cosmologists they're making a guess, albeit an educated guess on what we have already observed on the large scale in the observable universe.
Title: Re: What was before the big bang?
Post by: Bill S on 24/08/2015 14:38:01
Quote from: Mordeth
  An infinite state of nothing, containing only the potential to  be something, and progressing to a state of something I can measure suits me, I suppose.

This is another example of the verbal contortions commonly found in "something from nothing" circles.

How can nothing contain anything and still be nothing?

What is potential if not something?

How can nothing progress?
Title: Re: What was before the big bang?
Post by: Bill S on 24/08/2015 14:53:52
Quote from: Pete
The appropriate stance for a scientist is to take the position of I don't know if the universe emerged from something or not.

Would it not be reasonable to add to that something like: ....as no one has shown how something to emerge from nothing, then the claim that the Universe emerged from nothing involves an additional assumption.  Our old friend "Bill Ockham" might not like that.  [:)]
Title: Re: What was before the big bang?
Post by: dlorde on 24/08/2015 17:57:18
Mithani and Vilenkin’s claim (http://arxiv.org/pdf/1204.4658.pdf) that they've shown mathematically that the universe must have a beginning has an amusingly simple response from Leonard Susskind (http://arxiv.org/pdf/1204.4658.pdf), who shows that if we accept a beginning:
Quote from: Susskind
in any kind of inflating cosmology the odds strongly (infinitely) favor the beginning to be so far in the past that it is effectively at minus infinity.

More precisely, given any T the probability is unity that the beginning was more than T time-units ago.
So the beginning of the universe is infinitely far in the past. Perhaps this endearing solution can be added to Mordeth's list of possibilities.
Title: Re: What was before the big bang?
Post by: timey on 24/08/2015 20:55:17
Ah Mordeth, interesting reply! (Post 76)

Yes, I agree, there is a point where all fields shake hands.  However, I do believe that because logic is associated with philosophy and mathematics with physics, that the significance of applied logic has become under-represented in the world of physics.
Logic is a very precise tool that can be applied to any field/situation.  Like mathematics, logic is a process of reduction.  When a matter can be reduced no more, then truth emerges.  Logic is a more precise tool than mathematics because Mathematica it is only as good as the concepts it is attached to.  Logic is comprised only of concepts, and therefore it 'can' be more productive.

Take infinity.  You say you have trouble comprehending it.  Ok, to reduce this we must question one's ability to comprehend.  Quite easily we can deduce that because one cannot comprehend is not a proof that a concept does not exist.  Let us now try divorcing the concept of infinity from the time line aspect.  We can always add it back in later...  So, we find that now we can look more closely at these examples of what the mind may comprehend as infinite. 'Nothing', or 0, and 'everything.'
Looking at everything, let's say that everything were only 1 thing.  We can comprehend that this 1 thing is everything and in that it is only 1 thing and there is nothing else, this 1 thing 'is' infinite.

Now let's us look at nothing.   Everything absence everything = nothing.  Nothing is the symmetrical opposite to everything, if everything is infinite then nothing must also be infinite and we can see that when we add the time line aspect back in, a state of nothing has no time and remains by definition infinite.

Ok, all good so far... We have an infinite state of nothing and an infinite state of everything.  But we must remember that we haven't added the timeline aspect back into an infinite state of everything yet, so hang on, back up a mo... because an infinite state of everything is not as possible for everything as it is for nothing because everything includes the passage of time.

We have to carefully consider how this all fits together.  The second law of thermodynamics must be upheld... After much consideration, blah, blah... and so, henceforth my interpretation of a state of everything progressed infinitely.

"An infinite state of nothing progressed into a state of everything infinitely."

This however, in consideration of the second law of thermodynamics, when following this path of logic becomes quite interesting indeed. (To me anyway :) )

Bill makes a good point about potential.  Is potential something?  What is potential?  Potential is the possibility for change.  Can we logically reason that a state of nothing has the potential to change?  Again the second law is relevant.  If we consider our surroundings our experience tells us that all states have the potential to change.  Why would a state of nothing differ.  What potential change can be logically reasoned for a state of nothing?  The only potential change for a state of nothing is for it to become a state of everything.  For change to happen there must be a time aspect for it to happen within.  A state of nothing is timeless.  So... any potential that a state of nothing may possess can only be found in the possibility that the potential change in a state of nothing involve the initial moment of the phenomenon of time.

Now just to back track the reasoning, we have defined everything and we have defined the phenomenon of time, but we have not defined the rate that time is occurring at.  We know that the rate of time is affected by a gravitational field. (could time actually be caused by a gravitational field, being an entirely logical notion) ...And what was the gravitational field at the point of a state of nothing gaining the potential to a state of everything?  Again, second law becomes interesting!!!
Title: Re: What was before the big bang?
Post by: sciconoclast on 25/08/2015 00:25:24
I agree with some of Pmbhy's comments: "our understanding of the Universe is in its infancy....there  may be other universes.....singularity is merely an extrapolation."

Before scientist could observe other solar systems they had already deduced the process by which ours was formed from the laws of physics and the properties of the planets. I am hoping that similarly once both the understanding of physics and the universe is advanced the process generating our universe can be understood [ working on it ].

Einstein acknowledged his theory could lead to singularities occurring but he thought that an as yet unknown principle would prevent it.

Science once believed our galaxy to be the only one because we could not see beyond it. History often repeats itself. 

I thought I might introduce a relevant but slightly different concept for the pundits to jump on. Is it possible that there is a greater cosmos which does not have a composite time vector ( exist independent of time ) but localized events, such as the shock wave from a local big bang create a local time vector for everything within it. I could be more specific about how this could occur but this is not the place for that.
Title: Re: What was before the big bang?
Post by: Mordeth on 25/08/2015 00:38:17
Hi timey, Bill S and dlorde,

I must think and consider more. I am not well skilled in the area of transcendental thinking, so I substitute science and physics to arrive at explanations.  I am mostly interested in the events of the observable universe and the measurable relationship between those events.  I find that current scientific theory is adequate for my needs. This does not imply whatsoever that current science is adequate as a whole, nor does it suggest that we should not continually advance our somewhat limited understanding of things. 

timey, I particularly enjoyed reading what you just said.  Thank you for taking the time to write it.

Pmb, I also agree with your statements.  I believe I have stated multiple times in this thread that I do not know, and neither does science.  My recent posts were speculative responses to Bill and timey.
Title: Re: What was before the big bang?
Post by: PmbPhy on 25/08/2015 07:23:48
Quote from: Mordeth
Pmb, I also agree with your statements.  I believe I have stated multiple times in this thread that I do not know, and neither does science.  My recent posts were speculative responses to Bill and timey.
Same here. What I was disagreeing with was the automatic assumption about a singularity merely because GR is being used for all values of "t". I was recently doing a refresh on my cosmology as a result of this thread. I have a text by Andrew Liddle on the subject that I haven't read in years which I just started to read again. On the point of the Big Bang the author writes
Quote
In fact, if the cosmological constant is powerful enough, there need not even be a Big Bang, with the Universe instead beginning in a collapsing phase, followed by a bounce at finite size under the influence of the cosmological constant (though such models are ruled out by observations).
And in such a case there will be no singularity.

You can read the text at: http://bookos-z1.org/book/930602/67a687
Title: Re: What was before the big bang?
Post by: Bill S on 25/08/2015 14:40:49
   
Quote
(though such models are ruled out by observations).


It seems they may have been ruled in as far as computer simulations go.

Quote from: Anil Ananthaswamy (New Scientist2008)


    “Abhay Ashtekar remembers his reaction the first time he saw the universe bounce. ‘I was taken aback,’ he says.  He was watching a simulation of the universe rewind towards the big bang.  Mostly the universe behaved as expected, becoming smaller and denser as the galaxies converged.  But then, instead of reaching the big bang ‘singularity’, the universe bounced and started expanding again.  What on earth was happening?   

    Ashtekar wanted to be sure of what he was seeing, so he asked his colleagues to sit on the result for six months before publishing it in 2006.  And no wonder.  The theory that the recycled universe was based on, called loop quantum cosmology (LQC), had managed to illuminate the very birth of the universe – something even Einstein’s general theory of relativity fails to do. 

    LQC has been tantalising physicists since 2003 with the idea that our universe could conceivably have emerged from the collapse of a previous universe.  Now the theory is poised to make predictions we can actually test.  If they are verified, the big bang will give way to a big bounce and we will finally know the quantum structure of space-time.  Instead of a universe that emerged from a point of infinite density, we have one that recycles, possibly through an eternal series of expansions and contractions, with no beginning and no end.”
   
Title: Re: What was before the big bang?
Post by: timey on 25/08/2015 19:32:08
Hey Mordeth, thanks for the positive comment, really appreciated!

...and Bill, I am familiar with the LQC concept.  It's a fascinating thought that such was the result of a computer simulation!
Title: Re: What was before the big bang?
Post by: Bill S on 25/08/2015 22:24:15
For anyone who might not be familiar with Matt Strassler, this could be an interesting link.

http://profmattstrassler.com/articles-and-posts/relativity-space-astronomy-and-cosmology/history-of-the-universe/
Title: Re: What was before the big bang?
Post by: timey on 26/08/2015 10:23:28
Having read the link above, it disappoints me that Matt has not given more precise details as to why the discovery of March 18th 2014 did not stand up to scrutiny.  To know the reason why something does not work is just as useful as knowing why something does work!

Does anyone have any further info?
Title: Re: What was before the big bang?
Post by: jeffreyH on 26/08/2015 13:07:08
http://arstechnica.com/science/2014/09/gravity-wave-evidence-disappears-into-dust/
Title: Re: What was before the big bang?
Post by: timey on 26/08/2015 19:41:11
Great stuff!  Thanks for that link Jeff... I have read snippets of news concerning, but that was a very concise and up to date rendition and also I hadn't made the connection to relating this to Matt Strassler's comment.  Silly me :) .

But... and this is what interests me... presumably the computer simulation in 2008 that led to the concept of LQC was programmed independently of any CMB expectations?  And that this 'dust everywhere' disappointment is only relevant to the inflationary part of LQC concept, as it is a disappointment to all theories inclusive of an inflationary period?
Title: Re: What was before the big bang?
Post by: jeffreyH on 28/08/2015 17:32:18
Great stuff!  Thanks for that link Jeff... I have read snippets of news concerning, but that was a very concise and up to date rendition and also I hadn't made the connection to relating this to Matt Strassler's comment.  Silly me :) .

But... and this is what interests me... presumably the computer simulation in 2008 that led to the concept of LQC was programmed independently of any CMB expectations?  And that this 'dust everywhere' disappointment is only relevant to the inflationary part of LQC concept, as it is a disappointment to all theories inclusive of an inflationary period?

Not really. The wavelengths involved are very long and it is very hard to detect the effects. Dust would be enough to hide the effects.
Title: Re: What was before the big bang?
Post by: guest39538 on 28/08/2015 18:25:17
What was before the big bang? We could alter the question to say, what was before arbitrary time?


Clearly there can be only one definite answer to the question without any theory or hypotheses related to fallacy. Before the big bang there was space.  Logic and experiment on earth suggests that for anything to ''exist'' or happen, that something needs space to happen in. An expansion of metal by adding energy, the metal expands into space, an inflating balloon expands into space.  Observation shows us this and experiment shows us this.   
The expansion of ''space'' is by definition and logic not accurate in my opinion and based on science observation.  Science does not observe actual space, space is ''transparent'' to sight, we see through space like seeing through a sheet of glass. We observe matter in space and the motion of matter in space, velocities that we can calculate.
Using the big bang as a way to record the ''beginning of time'' is no more than measuring the distance of matter and the distance it has travelled through space from a specific event in space at a specific ''time''.  A specific time that is arbitrary and made by us, there is nothing to suggest that ''space-time'' did not pre-exist the big bang and everything we believe is based on a distance and speed rather than a time itself.
When people suggest there was nothing before the big bang, I see ''nothing'' as darkness, a dark space without content.
The human mind can only go this far back, that is what observation allows us to do, to imagine a beginning always leads to zero, a zero point space.
We are fools if we think that nothing means no space, because without space there can not be something. 
Title: Re: What was before the big bang?
Post by: timey on 28/08/2015 19:15:08
Great stuff!  Thanks for that link Jeff... I have read snippets of news concerning, but that was a very concise and up to date rendition and also I hadn't made the connection to relating this to Matt Strassler's comment.  Silly me :) .

But... and this is what interests me... presumably the computer simulation in 2008 that led to the concept of LQC was programmed independently of any CMB expectations?  And that this 'dust everywhere' disappointment is only relevant to the inflationary part of LQC concept, as it is a disappointment to all theories inclusive of an inflationary period?

Not really. The wavelengths involved are very long and it is very hard to detect the effects. Dust would be enough to hide the effects.

I'm sorry Jeff.  Not really what???  Clearly the link explains in much greater detail the reasons why the dust would be enough to hide the effects of long wavelengths, surely?

I fail to ascertain the 'point' of your post to be honest.  What does this have to do with the computer simulation from 2008 that Loop Quantum Cosmology is based on?

Or, if indeed this is what you intended to mean...why was it 'not really' a disappointment to all inflationary theories that the results of the CMB were dust tainted?

Can you be a little clearer please?
Title: Re: What was before the big bang?
Post by: timey on 28/08/2015 21:31:38
Jeff, I am not an expert but there is a reason why these discussions are called threads.  When partaking of one of these discussions it is expected that one can upkeep the thread of the conversation.  This involves reading what other posters have said and responding to the content of their posts in a relevant fashion... and it involves a poster to keep up with the posts that have transpired in-between the last post they made and the post they wish to make in the present.  Please know also that it is considered rude to reply to a post in a language not understood by the other poster/posters... And, to say so, it goes without saying that it is rather frowned upon to drastically alter one's statement, especially 'after' the fact... also... I have never heard before of someone deleting the evidence of 'ever' having made a post, because 'that' is just wrong...and lastly, this being just a suggestion, but talking to other posters as if they are of a lesser intelligence than you could actually prove counter productive!

On the basis of my observations of you Jeff, particularly regarding the 'how does light speed up when exiting a denser material" thread, I have come to the conclusion that you must not be aware of these factors concerning internet discussions, henceforth... please consider the above as some good sound advice!
Title: Re: What was before the big bang?
Post by: jeffreyH on 29/08/2015 11:48:52
Which post did I delete? I have rarely deleted a reply. I can't remember when I last did that.
Title: Re: What was before the big bang?
Post by: timey on 29/08/2015 12:12:16
Oh dear me Jeff, I can see that its much worse than I thought (chuckle)... Ok, to explain, there arises within these forums this wonderful feature that is in as much as being in possession of a time travelling machine.  By applying the scroll button one can just navigate back up the pages and revisit the past.
I have also experimented with time travelling into what future posts may relate, this manipulation isn't quite so obvious as a functionality of the forum, but is quite easily employed none-the-less!
Title: Re: What was before the big bang?
Post by: jeffreyH on 29/08/2015 14:31:47
You are trying to bring personality and personal motivation into the discussion. I am only interested in the physics. If you can tell me which post I deleted I will check the thread.
Title: Re: What was before the big bang?
Post by: timey on 29/08/2015 15:20:23
Well Jeff, I too am 'just' about the physics, which is why I have been somewhat non-plussed by some of your earlier responses to my posts.
I would rather get back to 'just' the physics whereas we can converse in relevance to the subject matter, as in recent posts on the "how does light speed up when it exits a denser material".

The post in question is within this thread I've mentioned, a post whereas you replied to me in response to post 49 saying "Now you are clearer in meaning and are saying something interesting"... This reply you made disappeared.  Your post at the top of page 4, post no 75 was particularly irrelevant and in relation to your disappearing post, well... if you continue reading page 4 in its entirety you will be up to speed.
Personally I feel, and if you read top of page 5, that this 'could' be a past tense situation, but if you are going to comment on my posts could you respond relevantly?

...and please know that my personality and personal motivation are always going to be an inseparable part of the package.
Title: Re: What was before the big bang?
Post by: timey on 06/09/2015 21:56:00
Well, on the basis that no comments are forthcoming concerning loop quantum cosmology and the relevant computer simulation in relation to the the dust factor of the CMB... how's about a bit more attempted application of logic?

Going back to the statement 'An infinite state of nothing progressed into a state of everything infinitely', we have already addressed the fact that a zero, although classed as nothing, is also classed as something.  Now let us look at the term 'existence'.  Can a state of nothing be defined as existing?  A state of nothing has no time to exist in...  Can something have an existence outside of time?  A state of everything cannot exist outside of time.  By definition, as soon as a state of nothing becomes a state of everything, even if that everything is only one thing, the fact of a change in state signifies a time factor.  By this reasoning we can then deduce that the only thing that can exist outside of time is nothing, and that a state of nothing can exist.
Title: Re: What was before the big bang?
Post by: dlorde on 07/09/2015 10:16:02
...Can a state of nothing be defined as existing?
There's no such thing as a 'state' of nothing. Nothing is the negation of existence and state. As such, it is purely conceptual; it exists as a negating concept only.
Title: Re: What was before the big bang?
Post by: timey on 07/09/2015 10:39:22
Nope, very sorry dlorde but that's not how it works.  Where is your picking apart of my logic in order to prove it wrong?  Or...where is the logical argument of your alternative statement?  There is none.  What you are doing is just making statements and presenting them as 'fact' without showing your reasoning.  To ague against statements presented in logic, one must counter present in logic, otherwise any alternative statement one makes in response is meaningless.
Title: Re: What was before the big bang?
Post by: dlorde on 07/09/2015 10:56:09
Where is your picking apart of my logic in order to prove it wrong?  Or...where is the logical argument of your alternative statement?  There is none.  What you are doing is just making statements and presenting them as 'fact' without showing your reasoning.  To ague against statements presented in logic, one must counter present in logic, otherwise any alternative statement one makes in response is meaningless.
I'm saying your premise is wrong. I stopped where you suggested nothing could be a state. A state is a particular condition of something, a mode or condition of being. But if you can justify nothing as a state, then I'll consider the rest of your argument. I suspect it will come down to another argument about the semantics of the word 'nothing', i.e. that we mean different things by it.
Title: Re: What was before the big bang?
Post by: timey on 07/09/2015 11:20:02
Again dlorde, the logic is not forthcoming... Let me show you your error.

Quote from: timey link=topic=59829.msg466467#msg4


66467 date=1441572960
...Can a state of nothing be defined as existing?
There's no such thing as a 'state' of nothing. Nothing is the negation of existence and state. As such, it is purely conceptual; it exists as a negating concept only.

Here you are contradicting yourself.  You say there is no such thing as a state of nothing and then say that a state of nothing does exist as a type of concept.  It is also a double contradiction of terms because by stating this you are agreeing to my statement of logic.

Where is your picking apart of my logic in order to prove it wrong?  Or...where is the logical argument of your alternative statement?  There is none.  What you are doing is just making statements and presenting them as 'fact' without showing your reasoning.  To ague against statements presented in logic, one must counter present in logic, otherwise any alternative statement one makes in response is meaningless.
I'm saying your premise is wrong. I stopped where you suggested nothing could be a state. A state is a particular condition of something, a mode or condition of being. But if you can justify nothing as a state, then I'll consider the rest of your argument. I suspect it will come down to another argument about the semantics of the word 'nothing', i.e. that we mean different things by it.

Here you are again contradicting yourself.  You say my premiss is wrong.  However you are stating that you stopped reading.  Then we have the double contradiction again, where you ask me to justify my premiss having stated that you stopped reading my presented argument.

Really dlorde, perhaps you might consider that the application of logic is not your forte?
Title: Re: What was before the big bang?
Post by: timey on 07/09/2015 11:49:59
Ok, on consideration there is one avenue of your argument that does hold merit.  Despite the fact that I have defined the statement in hand "An infinite state of nothing progressed into a state of everything infinitely", and  I do advise you to go back and read this if you plan on continuing this discussion, I 'have' neglected to define the term 'state'.

What defines something as a state?

As you have brought this up I will leave it to you to define this question and argue how nothing, which has already been defined as being something, cannot then be considered as a state...
Title: Re: What was before the big bang?
Post by: jeffreyH on 07/09/2015 12:37:48
Ok, on consideration there is one avenue of your argument that does hold merit.  Despite the fact that I have defined the statement in hand "An infinite state of nothing progressed into a state of everything infinitely", and  I do advise you to go back and read this if you plan on continuing this discussion, I 'have' neglected to define the term 'state'.

What defines something as a state?

As you have brought this up I will leave it to you to define this question and argue how nothing, which has already been defined as being something, cannot then be considered as a state...

I am surprised that anyone is still debating with you at all. Your attitude is atrocious.
Title: Re: What was before the big bang?
Post by: timey on 07/09/2015 13:07:06
I am surprised that anyone is still debating with you at all. Your attitude is atrocious.

Jeff, I'm truly surprised and shocked at your conjecture considering your own earlier attitude towards me.

I do not feel that I am being untruthful about dlorde's comments. He is not being logical.  End of story... and I am not the person/people talking to other posters as if they are stupid!  Or trying to lay down 'the law' about what is and what isn't!!!

(Edit...and furthermore I think it pertinent to describe to you my abject disappointment when that little rush that I felt, when I saw that you had posted, in anticipation that you had brought your obvious intelligence to bear upon the conversation in some manner that would be progressive, and therefore interesting to me, fell flat on its face. In fact, although Mordeth posts in context, with intelligence, and obviously has an appreciation of logic... it has only been Bill that has shown any talent in the 'progressive' department really)
Title: Re: What was before the big bang?
Post by: dlorde on 07/09/2015 14:50:47
...You say there is no such thing as a state of nothing and then say that a state of nothing does exist as a type of concept.
You misconstrued my reply. Perhaps it wasn't clear - I was referring to 'nothing' as existing only as a concept, not a 'state of nothing', which is meaningless.

Quote
You say my premiss is wrong.  However you are stating that you stopped reading.  Then we have the double contradiction again, where you ask me to justify my premiss having stated that you stopped reading my presented argument.
The premise I was referring to was the implicit assumption that a 'state of nothing' has meaning. I have suggest that it doesn't because 'state' refers to something. If the premise is invalid, the rest of the argument is moot.

Quote
Really dlorde, perhaps you might consider that the application of logic is not your forte?
I'm not a logician, but I don't need to be at this level. If you can justify or explain the meaning of a 'state of nothing', I will, as I said, consider your argument that depends on it.
Title: Re: What was before the big bang?
Post by: dlorde on 07/09/2015 15:09:12
As you have brought this up I will leave it to you to define this question and argue how nothing, which has already been defined as being something, cannot then be considered as a state...
I see a category error here. 'Nothing' is an abstraction; not part of physical reality, at one level explicitly by meaning, and at another implicitly as a conceptual abstraction (though it can have symbolic representation). The discussion, unless I misunderstand it, has been about physical reality - a different category.

I'm not sure how one could usefully refer to an abstraction having a state, but as far as I can see, it bears no relation to a state of the physically real. You may be able to explain it to me.

Title: Re: What was before the big bang?
Post by: timey on 07/09/2015 15:41:23
Actually dlorde, you are raising points that I have already defined logically.  When playing the game of logic, it is required that if you are questioning my logic, that you identify the definition that I have made, you prove it wrong and then redefine it to support your argument.

...and this is what I will commence in regards to this bit of your comment.

I see a category error here. 'Nothing' is an abstraction; not part of physical reality

For there to be nothing, everything must be physically absent, therefore by definition nothing 'is' a physicality.
Title: Re: What was before the big bang?
Post by: dlorde on 07/09/2015 15:51:52
For there to be nothing, everything must be physically absent, therefore by definition nothing 'is' a physicality.
Rather, an absence or negation of physicality.

But whatever; I've said my piece. Make of it what you will.
Title: Re: What was before the big bang?
Post by: Bill S on 07/09/2015 22:00:13
Quote from: Timey
When playing the game of logic....

Unfortunately, that is what this thread has become.  Great fun if what you are looking for a chance to score points.

Quote
For there to be nothing, everything must be physically absent, therefore by definition nothing 'is' a physicality.

I doubt that there is a "non-sequitur of the year" award, but if there were this would certainly make the podium.  [:)]
Title: Re: What was before the big bang?
Post by: timey on 07/09/2015 22:48:31
Quote from: Timey
When playing the game of logic....


Unfortunately, that is what this thread has become.  Great fun if what you are looking for a chance to score points.

I disagree, what this thread has become is a place were people are getting off point.

Quote
For there to be nothing, everything must be physically absent, therefore by definition nothing 'is' a physicality.

I doubt that there is a "non-sequitur of the year" award, but if there were this would certainly make the podium.  [:)]

All I'm looking for is an 'interesting' conversation Bill.

If you could give a logical explanation as to why you consider the statement as 'non-sequitur' when it clearly follows on from dlorde's post?  Or if you feel the logic is flawed - then a definition of why a physical absence of something constituting nothing is not a physicality, would be nice? :)

For there to be nothing, everything must be physically absent, therefore by definition nothing 'is' a physicality.
Rather, an absence or negation of physicality.

But whatever; I've said my piece. Make of it what you will.

You are, of course entitled to your viewpoint and I respect that.  I am disappointed that you have not furthered your statement though.  As I have indicated before, it is my opinion that you are arguing the same toss of the coin as I am, only using alternative words, which makes me wonder why you are stating to me that I am wrong...
Title: Re: What was before the big bang?
Post by: timey on 08/09/2015 11:04:19
'Nothing is the physical result of everything being absent'

Where is the logic flawed?
Title: Re: What was before the big bang?
Post by: Bill S on 08/09/2015 20:50:12
Quote from: Timey
For there to be nothing, everything must be physically absent, therefore by definition nothing 'is' a physicality.

If everything is physically absent, everything physical must be absent, so in what sense is nothing a physicality?
Title: Re: What was before the big bang?
Post by: timey on 08/09/2015 20:59:07
Quote from: Timey
For there to be nothing, everything must be physically absent, therefore by definition nothing 'is' a physicality.

If everything is physically absent, everything physical must be absent, so in what sense is nothing a physicality?

Because... nothing is the physical result of everything being absent.

Edit: ...this being because everything is 'physically' not there.
Title: Re: What was before the big bang?
Post by: Bill S on 08/09/2015 21:51:51
Quote from: Timey
Because... nothing is the physical result of everything being absent.

Edit: ...this being because everything is 'physically' not there.

We all know where ever decreasing circles take us; so lets try getting back on track.

http://discovermagazine.com/2013/september/13-starting-point

I have not had time to read the article yet, but it looks as though it might be relevant.
Title: Re: What was before the big bang?
Post by: timey on 08/09/2015 23:16:52
That is an interesting link Bill.  I can identify quite strongly with a few of the ideas presented.  The most pertinent with regards to this conversation being that everything started from nothing.

Let's remember that this current consideration, in response to dlorde's suggestion, is required in order to define nothing as a physical reality, so that nothing can be attributed the status of being a state.

I'm actually quite happy to drop the word 'state' from the statement altogether:

"Infinite nothing progressed into everything infinitely"

I'm all for reductionism, but dlorde has a point and it deserves attention.

I don't feel that I am presenting a circular argument.   I leave the room, I am physically absent.  The fact of something being physically not there, is in itself a physicality.  This, I think, is an argument of symmetry.  Both aspects of this symmetry being, I do believe, a physical reality .
Title: Re: What was before the big bang?
Post by: Bill S on 08/09/2015 23:38:33
Quote from: Timey
I leave the room, I am physically absent

No argument with that, but I assume you are neither nothing, nor everything - although I could construct quite a good case for the latter [:)] - so it is not really relevant.
Title: Re: What was before the big bang?
Post by: timey on 08/09/2015 23:51:47
...and here hark at the man who accuses others of point scoring, while in presenting, is also guilty of being off point. :D

Bill, clearly you have taken my analogy as the context in this instance and I was of course referring to the symmetry of nothing in relation to everything.

Have you any comment on symmetry per chance.  That might be interesting!
Title: Re: What was before the big bang?
Post by: jeffreyH on 09/09/2015 00:23:39
In physics, a symmetry of a physical system is a physical or mathematical feature of the system (observed or intrinsic) that is preserved or remains unchanged under some transformation.

https://en.wikipedia.org/wiki/Symmetry_(physics)

How can nothing be a symmetry of everything. It doesn't remain unchanged.
Title: Re: What was before the big bang?
Post by: Bill S on 09/09/2015 00:33:47
Quote from: Timey
and here hark at the man who accuses others of point scoring,

Someone accused someone else of point scoring?  Where would that have been?
Title: Re: What was before the big bang?
Post by: timey on 09/09/2015 00:49:21
Quote from: Timey
and here hark at the man who accuses others of point scoring,

Someone accused someone else of point scoring?  Where would that have been?

Unfortunately, that is what this thread has become.  Great fun if what you are looking for a chance to score points.

That would be you Bill.

What's happened?  You showed such promise with regards to progressive thought structures earlier in the thread.
Title: Re: What was before the big bang?
Post by: timey on 09/09/2015 02:25:00
In physics, a symmetry of a physical system is a physical or mathematical feature of the system (observed or intrinsic) that is preserved or remains unchanged under some transformation.

https://en.wikipedia.org/wiki/Symmetry_(physics)

How can nothing be a symmetry of everything. It doesn't remain unchanged.

Just saw this one Jeff...  Interesting!

But we are talking about a nothing that does become changed.

"What was before the Big Bang?"

So, by definition of nothing transforming into everything, symmetry is achieved. Edit: Nothing remains unchanged regarding nothing as a conceptual reality, in that nothing is still nothing.  It's just that everything has now taken its place.
Title: Re: What was before the big bang?
Post by: Bill S on 09/09/2015 04:08:21
Quote from: Timey
That would be you Bill.

You really should read what people write before you make accusations. 
Title: Re: What was before the big bang?
Post by: timey on 09/09/2015 11:46:39
Quote from: Timey
That would be you Bill.

You really should read what people write before you make accusations.

Ok then.  Let's do just that...

Unfortunately, that is what this thread has become.  Great fun if what you are looking for a chance to score points.

You start out with the word 'unfortunately', this indicating that what is to come is in the negative.  Because you do not follow this word with 'I', you negate yourself of any responsibility for the context.
The context is that the thread has become, to its detriment, a place of opportunity to score points.
Therefore you are suggesting that someone else is responsible for rendering this thread as such.  This can be classed as an accusation,

'My' use of the word 'accuse' was made in lightheartedness.  My intention to show you that the fact of your mention of 'non sequitur' arguments minus any relevant definition of them being so, my winning a place on a 'podium', and your taking my analogies out of context and presenting them as jibes at my person... really is displaying 'the' behaviour' that you have written about, and written about in the context of someone else doing it.

I want to talk about what may or may not have been before the Big Bang Bill.

If your intention is to be derogatory, I can tell you I do not appreciate it... it's boring!
Or if you are wanting to 'jibe' with me, then why don't you come down "On the 'even' lighter side"... and we can do so in 'fun' and 'ridiculousness'.  I certainly have no wish to 'argue' anything with you here other than the subject matter, this being 'what was before the Big Bang'!
Title: Re: What was before the big bang?
Post by: Bill S on 09/09/2015 14:56:25
Quote from: Timey
You start out with the word 'unfortunately', this indicating that what is to come is in the negative.  Because you do not follow this word with 'I', you negate yourself of any responsibility for the context.

I accept responsibility for anything I write; don’t you?

Quote from: Timey
Therefore you are suggesting that someone else is responsible for rendering this thread as such.  This can be classed as an accusation,

That is your interpretation.  Take responsibility for it.

In what context can your interpretation of what you claim to be my suggestion be considered an accusation?

Quote from: Timey
'My' use of the word 'accuse' was made in lightheartedness

Light-hearted, or not, it behoves one to be accurate.

Quote from: Timey
If your intention is to be derogatory, I can tell you I do not appreciate it... it's boring!

My comment was pertinent to the thread, and was not personal.  I invite you to look at as many of my posts, in this and other threads, as you choose.  I think you would have to agree that “ad hominem” derogatory attacks are not my style. 

Quote from: Timey
I want to talk about what may or may not have been before the Big Bang Bill.

Then perhaps we would have more success if we were to avoid “playing the game of logic”, or any other game.

I suggest we draw a line at this point and return to the subject we, apparently, all want to discuss.   
Title: Re: What was before the big bang?
Post by: timey on 09/09/2015 15:02:06
In physics, a symmetry of a physical system is a physical or mathematical feature of the system (observed or intrinsic) that is preserved or remains unchanged under some transformation.

https://en.wikipedia.org/wiki/Symmetry_(physics)

How can nothing be a symmetry of everything. It doesn't remain unchanged.

Just saw this one Jeff...  Interesting!

But we are talking about a nothing that does become changed.

"What was before the Big Bang?"

So, by definition of nothing transforming into everything, symmetry is achieved. Edit: Nothing remains unchanged regarding nothing as a conceptual reality, in that nothing is still nothing.  It's just that everything has now taken its place.

Ok Jeff, to continue... Your post was thought inspiring, I appreciate that...

I have been further thinking and while your argument holds merit, my response is not sufficiently defined and is therefore on 'shaky' ground.

Quote:
"In physics, a symmetry of a physical system is a physical or mathematical feature of the system (observed or intrinsic) that is preserved or remains unchanged under some transformation."
Unquote

Before I start, I want to illuminate that it is a well documented fact that friction arrises between individuals approaching physics from the stand point of conceptual ideas and individuals approaching physics from the basis of mathematics.  May I suggest that we not fall into that trap, and be respectful of each other's contributions be they meaningful or not?

I make a case for everything and nothing to be symmetrical opposites of a physical reality.  It is appreciated that a symmetry of a physical system must remain unchanged under a transformation.  That nothing is 'replaced' with everything doesn't obviously reflect this balance of symmetry, it is true!  And perhaps I may concede that you are correct in stating this suggestion as unrealistic.  But looking more closely, is it possible that a symmetry of sorts can be achieved in the event of the cross over from nothing to everything itself?

Nothing in its 'existence', (unproven), it's existence being 'perhaps' signified by the fact that everything had to start from somewhere, has - by this reasoning - the potential for change.
Nothing can then be thought (experimentally) to be a state of reality and everything, in the event that we are in nothing's reference frame, is the negative of nothing. (I use the word negative tentatively, as I understand this is a mathematical term, one which may not apply to the context I am using it in... so if you will accept my terminology loosely). In everything's reference frame, nothing is the negative of everything.  In the cross over reference frame we see a positive negative, negative positive crossover.  As a concept, can an interchanged crossover be considered as symmetry?  Does this translate into feasible mathematical terms?
Title: Re: What was before the big bang?
Post by: Bill S on 09/09/2015 21:27:36
Quote from: Timey
Before I start, I want to illuminate that it is a well documented fact that friction arrises between individuals approaching physics from the stand point of conceptual ideas and individuals approaching physics from the basis of mathematics.

I would like to add to that that friction can often arise between the groups you mention, on one hand, and the pragmatic bunch, on the other.  The label “crackpot” is frequently attached to pragmatists. 

Quote
Nothing in its 'existence', (unproven), it's existence being 'perhaps' signified by the fact that everything had to start from somewhere, has - by this reasoning - the potential for change.

Surely, existence is a quality of something that exists.  If you argue that “nothing” exists, are you not classifying it as “something that exists”.  Isn’t this a contradiction in terms?

Apparently you assume that “everything had to start from somewhere”.  Wouldn’t it be equally reasonable to assume that everything is eternal?

Quote
in the event that we are in nothing's reference frame,

Are you using “reference frame” in the relativistic sense, or do you give it some other meaning.  In either event, how do you define nothing’s F of R?

Quote
In the cross over reference frame we see a positive negative, negative positive crossover.

Would you see this as a reversible process?
Title: Re: What was before the big bang?
Post by: timey on 09/09/2015 23:12:31
Quote from: Timey
Before I start, I want to illuminate that it is a well documented fact that friction arrises between individuals approaching physics from the stand point of conceptual ideas and individuals approaching physics from the basis of mathematics.

I would like to add to that that friction can often arise between the groups you mention, on one hand, and the pragmatic bunch, on the other.  The label “crackpot” is frequently attached to pragmatists.

Are you inferring some direct reference to this discussion in this consideration, or is that a more general observation?

Quote
Nothing in its 'existence', (unproven), it's existence being 'perhaps' signified by the fact that everything had to start from somewhere, has - by this reasoning - the potential for change.

Surely, existence is a quality of something that exists.  If you argue that “nothing” exists, are you not classifying it as “something that exists”.  Isn’t this a contradiction in terms?

I am arguing a case for nothing existing.  The paradox is that nothing is indicative of nothing existing. (Edit: as in nothing does not exist). My premiss for nothing existing (Edit: as in nothing does exist) is the existence of everything.  If everything is physically absent then nothing is the physical result.

Apparently you assume that “everything had to start from somewhere”.  Wouldn’t it be equally reasonable to assume that everything is eternal?

Well yes, of course, and I do believe that the steady state theory covered that one.  Problem being that we observe progression.  Progression being indicative of a beginning and an end, or a cycle.

Quote
in the event that we are in nothing's reference frame,

Are you using “reference frame” in the relativistic sense, or do you give it some other meaning.  In either event, how do you define nothing’s F of R?

Not sure, I just mean from the perspective of...  Again, not sure, what are F and R referring to here? Edit: (chuckle) :)

Quote
In the cross over reference frame we see a positive negative, negative positive crossover.

Would you see this as a reversible process?

No, not in the slightest, (edit: this would contravene the second law of thermodynamics) which is also a good point in reference to any potential symmetry of the situation.
Title: Re: What was before the big bang?
Post by: Bill S on 10/09/2015 02:54:37
Quote
Are you inferring some direct reference to this discussion in this consideration, or is that a more general observation?

I'm inferring nothing, just making an observation, relevant to your comment, arising from experience in many threads, on various forums.

Quote
Well yes, of course, and I do believe that the steady state theory covered that one.  Problem being that we observe progression.  Progression being indicative of a beginning and an end, or a cycle.

Does that mean you reject the idea of an eternal cosmos, or an eternally inflating multiverse/universe?

Quote
No, not in the slightest, (edit: this would contravene the second law of thermodynamics) which is also a good point in reference to any potential symmetry of the situation.

That's an interesting thought; what would be the entropy of nothing?
Title: Re: What was before the big bang?
Post by: timey on 10/09/2015 12:06:26
Quote
Well yes, of course, and I do believe that the steady state theory covered that one.  Problem being that we observe progression.  Progression being indicative of a beginning and an end, or a cycle.

Does that mean you reject the idea of an eternal cosmos, or an eternally inflating multiverse/universe?

Well... I do not out-rightly reject anything unless there is direct evidence to support doing so...

Eternal cosmos I think is unlikely in the face of what we observe everywhere concerning progression, and the fact of a more general consensus held by the world of physics itself.

I do not reject the idea of a multi-verse, or more accurately 'multi-verses', but feel that it is a pointless consideration in the face of our lack of understanding about this universe.

The current interpretation of observed data suggests very strongly that we are of an inflating universe.  Do I blindly accept this hypothesis?  In the face of the problems encountered by the current theories concerning unification, no I do not, but neither do I entirely discount it.  If the universe has a beginning and an end, then this inflation, for me, remains logical in the big freeze scenario, although this scenario does maintain a fuzzy interpretation of the second law.  Roger Penrose does have a theory that renders this inflation into a cyclic universe.

Roger Penrose's theory aside,  if the universe is a cyclic phenomenon, then this inflation could perhaps be considered illogical.  The Big Crunch or Bounce theory renders the second law fuzzy again and leaves us wondering how everything we see today can originate from a point, and retract to a point.

It's a mystery for sure Bill, but it does make for some damn fine thinking matter, aye ;)

That's an interesting thought; what would be the entropy of nothing?

Ok, so... If we 'can' consider nothing as a physical reality, all parameters of everything would be set to 0.   0 mass, 0 gravity, 0 time, 0 entropy.  At the cross over point, logically speaking 'everything' would be initiated as a progression up the scale from the point of 0.  At this stage of the path this logic is taking, it's the behaviour of bacteria that comes to my mind.
Title: Re: What was before the big bang?
Post by: jeffreyH on 10/09/2015 12:19:04
If there is nothing how can anything be initiated? Cause and effect then break down. There has to be some energy around to initiate a big bang.
Title: Re: What was before the big bang?
Post by: timey on 10/09/2015 12:33:28
If there is nothing how can anything be initiated? Cause and effect then break down. There has to be some energy around to initiate a big bang.

Well of course I'm really not entirely sure Jeff, but this is the mystery that surrounds the fact of the Big Bang theory... How did everything we see today originate from a point?

If we lower our expectations somewhat and ask the moment of the creation of our universe to follow a cyclic pattern of progression from a point of 0, could the famous Dirac equation that pops a particle out of a vacuum be significant?
Title: Re: What was before the big bang?
Post by: Bill S on 10/09/2015 14:47:38
Quote from: Timey
If we lower our expectations somewhat and ask the moment of the creation of our universe to follow a cyclic pattern of progression from a point of 0, could the famous Dirac equation that pops a particle out of a vacuum be significant?

There are a few points I want to pick up, but no time at present, so just the last one.  I think the answer has to be "no" because the vacuum, in this context, can certainly not be classed as nothing.
Title: Re: What was before the big bang?
Post by: timey on 10/09/2015 14:59:19
Cool... :) .  I'll look forward to hearing about these points later on, in the meantime, just a consideration...

What is it that defines a vacuum?
Title: Re: What was before the big bang?
Post by: timey on 10/09/2015 20:18:39
Well...as far as I'm concerned this looking at the moment of creation as being in terms of the microscopic is all getting quite interesting!

A Vacuum State
https://en.m.wikipedia.org/wiki/Vacuum_state

In particular with regards to this statement:

"An outstanding requirement imposed on a potential Theory of Everything is that the energy of the quantum vacuum state must explain the physically observed cosmological constant."

https://en.m.wikipedia.org/wiki/Cosmological_constant

And with regards to this statement in link above:

"Why can't the zero-point energy of the vacuum be interpreted as a cosmological constant? What causes the discrepancies?"

Are we back to my proposed idea of the use of an inadequate time measurement in relation to the mathematical structure of the Planck scale?

However...I have realised, in the event of the moment of creation in microscopic terms, that IF a "perfect vacuum' where to exist, and I expected to find the energy to pop a particle out of this perfect vacuum - that I would be forced to look at the Casimir Effect.  If fluctuations were occurring in a perfect vacuum, ie: the nothing before creation, then time would have had to have started before a particle was created.

Leading me to wonder, due to the relationship between mass and time, if fluctuations such as the Casimir Effect have mass?
Title: Re: What was before the big bang?
Post by: Bill S on 11/09/2015 00:26:29
Obviously “vacuum” has different meanings in different contexts, but here, as your last post indicates, we are dealing with the cosmological version, where it is widely accepted that there is energy, which must be something.

http://hetdex.org/dark_energy/what_is_it/vacuum_energy.php

“Today, physicists explain the cosmological constant as the vacuum energy of space.

In essence, this says that pairs of particles are constantly popping into existence throughout the universe. These "virtual pairs" consist of one particle with a negative charge and one with a positive charge. They exist for only a tiny fraction of a second before they collide and annihilate each other in a tiny burst of energy. This energy may be pushing outward on space itself, causing the universe to accelerate faster.”

Here’s another valuable look at the vacuum – or vacua. 

http://profmattstrassler.com/articles-and-posts/particle-physics-basics/theories-and-vacua/ 

Title: Re: What was before the big bang?
Post by: Bill S on 11/09/2015 00:41:44
Quote from: Timey
Eternal cosmos I think is unlikely in the face of what we observe everywhere concerning progression, and the fact of a more general consensus held by the world of physics itself.

I should have mentioned, in case you missed it elsewhere on this forum, I normally follow John Gribbin’s terminology with regard to the use of universe and cosmos.   

Cosmos = everything that exists, or can exist.
Universe = our (in principle) observable portion of spacetime and its contents.
universe = any other universe that may, or may not, exist.

Would that make any difference to your response?
Title: Re: What was before the big bang?
Post by: timey on 11/09/2015 01:05:46
Ok, well thanks for that Bill.  The first link pretty much reiterated what the links I posted state...

The second is more interesting, especially with regards to this statement:

"So when we talk about empty space, we really mean space that is as empty as possible. In some sense it is empty, because it has no particles in it, not even particles of light (photons).  (Particles are long-lived and simply-behaved ripples in fields.) But in some sense it is not empty, because the electric field, W field, Higgs field etc. are always in it! The vacuum, or rather, “a” vacuum, isn’t specified just by saying “it’s empty space”, because not only do we have to say that there are no particles in it, we also have to specify what the fields in that empty space are actually doing… Terminology: we say that we have to specify “the configuration of the fields” in that vacuum."

The link also goes on to say that there is a train of thought that allows for 2 types of vacuum with regards to the Higgs Field,

If we can concede that in as much as the Big Bang of the Big Bang theory constitutes a 'special' situation... that the 'nothing' that was before the moment of creation also would constitute a special situation - could the possibility of a completely empty vacuum exist?

In the event that we are considering this as feasible, what are the properties of a normal vacuum that we would expect to see emerging first and why?

And given that the emerging fields necessary to initiate the appearance of these fleeting virtual pairs of particles can emerge from this 'special vacuum', what would happen if the positive and negative do not annihilate each other (the fact of them doing so is also not a proven fact of dark energy, just a supposition and not a thoroughly accepted one at that) ... What happens if there are another pair of virtual particles that appear in close vicinity and two positives or two negatives join up?

:), that's a lot if what ifs I realise, but we are rather in that sort of region with the subject matter... anyway, that's my excuse and I'm sticking to it.

Quote from: Timey
Eternal cosmos I think is unlikely in the face of what we observe everywhere concerning progression, and the fact of a more general consensus held by the world of physics itself.

I should have mentioned, in case you missed it elsewhere on this forum, I normally follow John Gribbin’s terminology with regard to the use of universe and cosmos.   

Cosmos = everything that exists, or can exist.
Universe = our (in principle) observable portion of spacetime and its contents.
universe = any other universe that may, or may not, exist.

Would that make any difference to your response?

Yes, your definition of the terminology does change my answer.  Eternal cosmos, on the basis that this developed from a beginning and is in constant progression via a cyclic universe, yes I think this is a more likely scenario.
Title: Re: What was before the big bang?
Post by: Bill S on 11/09/2015 03:39:35
Quote from: Timey
I do not reject the idea of a multi-verse, or more accurately 'multi-verses', but feel that it is a pointless consideration in the face of our lack of understanding about this universe.

Let’s not forget that the multiverse began as the many worlds interpretation of quantum physics.  It is a concept relating to QM and the peculiarity of quons*, rather than to cosmology.

*Having been reproached in another thread for using a non-mainstream term, I feel I should do my best to popularise it. [;)]

https://en.wikipedia.org/wiki/Quon
Title: Re: What was before the big bang?
Post by: timey on 11/09/2015 10:32:49
Yes, you have a very valid point and it's not one that I have forgotten about.

For instance, IF we can look at the moment of creation as occurring in the microscopic region, and IF we can look at the nothing that came before the moment of creation as a reality that we can interpret as a 'perfect vacuum', but we cannot attribute the mechanics of this scenario with the emergence/beginning of time.  That we go on to find that despite the fact that time had to exist in order for quantum fluctuations to occur, we can find no cause for this scenario of emerging fields in this perfect vacuum state to account for the first moment of time...but everything else fits to a tee. Then we might say this signified the existence of our universe being part of a multi-verse.

Even if we could find a cause for time emerging from this perfect vacuum state... the potential that we may define this 'nothing' before creation as a reality of a perfect vacuum, would of course open up the possibility that other fluctuations in this perfect vacuum may have led to other universes that might be the same as ours, or similar, or perhaps even wildly different.  In this instance would these alternative universes be part of a multi-verse or would they be individually universes in their own right?

If we are to use the concept of multiple universes, or a multi-verse as an off-set to discovery about our universe, I think it IS the aspect of time and how it emerges in our universe that will be the telling factor.

*Having been reproached in another thread for using a non-mainstream term, I feel I should do my best to popularise it. [;)]

Quantum objects: Quons, I like it! :)
Title: Re: What was before the big bang?
Post by: PmbPhy on 11/09/2015 16:45:11
Quote from: Timey
I do not reject the idea of a multi-verse, or more accurately 'multi-verses', but feel that it is a pointless consideration in the face of our lack of understanding about this universe.

Let’s not forget that the multiverse began as the many worlds interpretation of quantum physics.  It is a concept relating to QM and the peculiarity of quons*, rather than to cosmology.

*Having been reproached in another thread for using a non-mainstream term, I feel I should do my best to popularise it. [;)]

https://en.wikipedia.org/wiki/Quon
It's appropriate to post the definition of a non-mainstream term so that your reader won't have to ask you what it means and won't have to search for it.
Title: Re: What was before the big bang?
Post by: Bill S on 11/09/2015 21:14:55
Quote from: Pete
It's appropriate to post the definition of a non-mainstream term so that your reader won't have to ask you what it means and won't have to search for it.

I did that first time I used it, then added it to Wiki to make it easy for others to find it. Also, I'm not the only one to use it on TNS.
Title: Re: What was before the big bang?
Post by: Bill S on 11/09/2015 21:46:33
Quote from: Timey
Roger Penrose's theory aside,  if the universe is a cyclic phenomenon, then this inflation could perhaps be considered illogical.  The Big Crunch or Bounce theory renders the second law fuzzy again and leaves us wondering how everything we see today can originate from a point, and retract to a point.

The cyclic idea has been around for some time, and has been used to suggest that the fine tuning of our Universe is the result of "evolution" through countless (infinite?) universes.  Paul Davies poured some cold water on this a few years ago.

He argues that: “… any cyclic universe that allows physical structures and systems to propagate from one cycle to the next will not evade the degenerative influences of the second law of thermodynamics.  There will still be heat death"………. "One way to sidestep this dismal conclusion is to suppose that the physical conditions at the bounce are so extreme that no information about earlier cycles can get through to the next.  All preceding physical objects are destroyed, all influences annihilated.  In effect, the universe is reborn entirely from scratch”.

A universe from scratch?  Where would you find enough scratch? [:)]

Seriously, though, it would rule out evolution.
Title: Re: What was before the big bang?
Post by: timey on 11/09/2015 22:15:57
We'll actually Bill, I have a theory that negates Paul's argument, but I don't really feel that this thread is the place to extrapolate this theory.  If you read from post 103 onwards on my thread "a theory of inverted time dilation", you will see how my theory of a cyclic universe copes with the second law of thermodynamics and why it would have the scratch to start again, although where it would get the scratch from to begin its first cycle is the mystery.

I really don't agree that a cyclic universe would rule out evolution.  If the present cycle of the universe takes x amount of billions of years to transpire from beginning to end, why would evolution be ruled out?
Title: Re: What was before the big bang?
Post by: Bill S on 11/09/2015 22:25:09
Evolution would be ruled out if nothing could pass from one universe to the next, but as you say, PD might not be right.
Title: Re: What was before the big bang?
Post by: timey on 11/09/2015 22:32:24
Ok, so if everything did pass from one cycle of the universe into the next, this argument is null and void?
Title: Re: What was before the big bang?
Post by: timey on 11/09/2015 22:41:32
QM 'is' indicative of the possibility of a cyclic universe.

https://en.m.wikipedia.org/wiki/Cosmological_constant

"Cyclic Model
More recent work has suggested the problem may be indirect evidence of a cyclic universe possibly as allowed by string theory. With every cycle of the universe (Big Bang then eventually a Big Crunch) taking about a trillion (1012) years, "the amount of matter and radiation in the universe is reset, but the cosmological constant is not. Instead, the cosmological constant gradually diminishes over many cycles to the small value observed today."[20] Critics respond that, as the authors acknowledge in their paper, the model "entails ... the same degree of tuning required in any cosmological model".[21]"
Title: Re: What was before the big bang?
Post by: jeffreyH on 11/09/2015 22:57:49
It all depends upon the fate of the universe. If there is a big crunch then ALL the matter that originated from the big bang would need to recombine. If there is any loss of mass in one cycle there will be a cumulative loss of mass over many cycles. Therefore the universe would shrink with each successive bounce until it did not have enough mass to initiate a new cycle. If we look at a multiverse representation there needs to be a way that mass loss from bubble universes can be explained. There are a whole lot more issues to consider. I went through this thought process a few years ago.
Title: Re: What was before the big bang?
Post by: timey on 11/09/2015 23:05:15
It all depends upon the fate of the universe. If there is a big crunch then ALL the matter that originated from the big bang would need to recombine. If there is any loss of mass in one cycle there will be a cumulative loss of mass over many cycles. Therefore the universe would shrink with each successive bounce until it did not have enough mass to initiate a new cycle. If we look at a multiverse representation there needs to be a way that mass loss from bubble universes can be explained. There are a whole lot more issues to consider. I went through this thought process a few years ago.

Why would there be any loss of mass in a bounce scenario?

What would drive this bounce scenario from a crunch into re-expanding.

Also... what negates the possibility of a universe producing more mass within its cycle?
Title: Re: What was before the big bang?
Post by: jeffreyH on 11/09/2015 23:22:43
As I said. There are a whole lot of issues to consider.
Title: Re: What was before the big bang?
Post by: Bill S on 12/09/2015 21:52:27
Quote from: Timey
Ok, so if everything did pass from one cycle of the universe into the next, this argument is null and void?

 If everything passes from one cycle to the next, this must include time; yes?
Title: Re: What was before the big bang?
Post by: Bill S on 12/09/2015 22:41:56
Quote from: Timey
IF a "perfect vacuum' where to exist, and I expected to find the energy to pop a particle out of this perfect vacuum - that I would be forced to look at the Casimir Effect.


 By “a perfect vacuum”, do you mean “absolutely nothing”?  If so, where would the particle come from; where would the energy come from, and where would you get the waves for the Casimir Effect?
Title: Re: What was before the big bang?
Post by: timey on 13/09/2015 00:16:26
Goodness Bill, you don't ask for much do you? :)

Obviously, I just dunno!  If I did I'd be shouting from the rooftops.  I really do not understand enough about this area of quantum mechanics to even begin to guess.

Yes... I do mean that this proposed 'perfect vacuum' is entirely empty.

As a logical enterprise, if we 'can' consider this perfect vacuum, ie: the nothing before creation, to be a physical reality, we have to consider that this nothing is going to be unlike anything we have ever seen.
Again, in as much as the Big Bang theory states the Big Bang as a special circumstance, is there anything special about this nothing that we can identify?
The first thing that occurs, and here I am using muti-verse or multiple verses as an offset again, is that if there were truly nothing at-all, then this situation in itself may constitute some kind of physical reaction.
If this strange thought were to hold any merit at-all, then stripping a normal vacuum down to its bare essentials and beyond, in conjunction with an analysis of the time scale factor 'might' be of significance.
Again, seriously, I'm not nearly enough well read in this area of QM regarding the configuration of states, nor the potential, or indeed non potential of virtual particles to comment as to any detail.

Maybe someone else has some suggestion...
Title: Re: What was before the big bang?
Post by: Bill S on 13/09/2015 16:16:23
Quote
As a logical enterprise, if we 'can' consider this perfect vacuum, ie: the nothing before creation, to be a physical reality…..

Considering “nothing” in this way seems to be quite common in scientific circles.  My favourite quote from Lawrence Krauss is: “By nothing, I do not mean nothing…..”


I have yet to find a way round the idea that if there was ever nothing, there would still be nothing now.  There are lots of “universe from nothing” ideas, but the nothings always seem distinctly somethingy.   
Title: Re: What was before the big bang?
Post by: timey on 13/09/2015 17:35:09
Well yes Bill, so it would seem!  The point I'm trying to make is:

That if everything is physically absent, then nothing is the physical result.  This defining nothing as being something.

What is wrong with viewing nothing as being 'something'?  It 'might' prove beneficial to do so on an experimental basis... In any case, and especially if one were to be interested in a moment of creation that has its beginning in the microscopic region.  Which would actually seem to me to be the most logical scenario for the first instances of creation, when considering the concept of a cyclic universe.

If there wasn't ever nothing Bill, how do you account for progression?  Are you saying that there was something before the moment of the creation of our universe?  And if so, are you saying that the something before the moment of creation of our universe had a something that was before its moment of creation, and so on?  Was there ever not a 'something' for a universe to be created in?  What is it that creates that something?

Looking at nothing as being that something is but a more simplistic approach.
Title: Re: What was before the big bang?
Post by: Bill S on 13/09/2015 21:40:24
I’ve just returned from a dog walk, about an hour which includes some thinking time.  What was I trying to think about? – Nothing!

Two things stood out:
1. We have few, if any, words with which to talk about “nothing”.  Our language is geared to talking about something.  With due respect to your question:  “What is wrong with viewing nothing as being 'something'?”  It makes less sense than saying: “What is wrong with viewing black as being white?”  It might be possible to make a case for either, but unless we are consistent in our use of language/terminology we tend to go round in non-productive circles.

2. “Nothing” cannot exist.  There is quite a lot more to be said about that, but I just wanted to run the thought past you before saying anything else.
Title: Re: What was before the big bang?
Post by: timey on 13/09/2015 22:57:37
Ah now see, this is what I like about you Bill, you have a knack for progressive thought structure... :)

In that you are making a case for nothing not existing, obviously you are not convinced by my argument: 'if everything is physically absent, then nothing is the physical result'... This based upon the fact that it 'is' the existence of everything that we are attempting to understand within the question.

Ok, you say that stating that nothing is something, is in as much as stating black is white.  In this case, unlike our poor 'nothing', both of the properties, black and white, are already determined as being something, so the analogy is lacking. But in that we are assuming a change from one state to another, if we are to make a parallel...
Taking black as being the state before creation and white as being the created state of everything, we can see that all we have to do is add x amount of white to the black state and it will become white.
Taking this analogy, if we can determine nothing as a state of reality, and then we add x amount of everything, x being the bits of everything necessary to account for creation, then nothing becomes everything in the same way that black will become white.
Title: Re: What was before the big bang?
Post by: Mordeth on 14/09/2015 00:49:22
Still talking about nothing, eh?  It cannot be defined.  You may as well ask if the number 37 is a Republican.  If "nothing" existed ontologically, then it would be something.  Therefore, "something" is the ontological default.  Can you perceive nothing perfectly?  No, because you always perceive yourself. The closest you can come is a deep loss of consciousness. When you awake, it appears you have experienced nothing, not even time. But even this is not true nothing.
 
As for something coming from "nothing", please review this paper:

http://arxiv.org/abs/1306.3232

In particular:

“Although we have analyzed only one version of the Emergent Universe, we would argue that our analysis is pointing to a more general problem: it is very difficult to devise a system – especially a quantum one – that does nothing “forever,” then evolves. A truly stationary or periodic quantum state, which would last forever, would never evolve, whereas one with any instability will not endure for an indefinite time.”
Title: Re: What was before the big bang?
Post by: timey on 14/09/2015 02:32:29
In our human perception of nothing, nothing is already defined as a description of an absence, or a mathematical concept.  As a meditative state of consciousness?  I don't even want to go there at-all, this incorporates a different branch of physics, interesting, but not relevant here.

You seem to have forgotten that earlier in the thread we already covered the concept of divorcing infinity from the time aspect, whereas for the case of an infinite state of nothing, we found that there was no need to add time back in.  Nothing has no time, therefore nothing, if it were to exist as a state in QM, would not be 'everlasting' in its description of infinity.  In fact the possibility exists that nothing, if we 'could' parallel its time line in relation to a time aspect, might only have been in existence as a fleeting phenomenon, with time and everything else emerging smoothly as a continuum of a natural progression from nothing.
A state of the lowest entropy really does need to be set at 0 doesn't it?  If not, then what are the parameters for the instance of the state of lowest entropy?

You say that nothing, in the context that we are using it, cannot be defined.  I beg to differ.  Anything can be defined.  Sometimes, in fact for the most part, this only requires that one change ones perspective to the problem.

Both you and Bill are saying that nothing cannot be a physical reality because there is nothing there.  I turn the matter upside down and state that because everything being absent is a physical reality, that nothing is the physical result of the physical reality of everything being absent.
Just like with algebra, because I have absolutely no doubts about the fact that everything is a physical reality, this being my determined parameter, the fact that I can physically absent everything from the equation determines the physical result of this action being that we are physically left with nothing.  That is logic, and I don't think my logic is flawed.  If you think it is can you show me where?
Title: Re: What was before the big bang?
Post by: jeffreyH on 14/09/2015 13:24:49
In our human perception of nothing, nothing is already defined as a description of an absence, or a mathematical concept.  As a meditative state of consciousness?  I don't even want to go there at-all, this incorporates a different branch of physics, interesting, but not relevant here.

You seem to have forgotten that earlier in the thread we already covered the concept of divorcing infinity from the time aspect, whereas for the case of an infinite state of nothing, we found that there was no need to add time back in.  Nothing has no time, therefore nothing, if it were to exist as a state in QM, would not be 'everlasting' in its description of infinity.  In fact the possibility exists that nothing, if we 'could' parallel its time line in relation to a time aspect, might only have been in existence as a fleeting phenomenon, with time and everything else emerging smoothly as a continuum of a natural progression from nothing.
A state of the lowest entropy really does need to be set at 0 doesn't it?  If not, then what are the parameters for the instance of the state of lowest entropy?

You say that nothing, in the context that we are using it, cannot be defined.  I beg to differ.  Anything can be defined.  Sometimes, in fact for the most part, this only requires that one change ones perspective to the problem.

Both you and Bill are saying that nothing cannot be a physical reality because there is nothing there.  I turn the matter upside down and state that because everything being absent is a physical reality, that nothing is the physical result of the physical reality of everything being absent.
Just like with algebra, because I have absolutely no doubts about the fact that everything is a physical reality, this being my determined parameter, the fact that I can physically absent everything from the equation determines the physical result of this action being that we are physically left with nothing.  That is logic, and I don't think my logic is flawed.  If you think it is can you show me where?

If we have two values of energy where A is positive and B is negative then we can have a situation where A + B = 0. You then have a mathematical nothing and yet the energy has not gone away.
Title: Re: What was before the big bang?
Post by: ProjectSailor on 14/09/2015 13:55:27
Apart from if you talk absolute (heading towards an absolute absence of energy) where you cannot have negative energy.

then A + B would never equal absolute zero. If you can have negative energy in the absolute scale it would be like having negative mass
Title: Re: What was before the big bang?
Post by: jeffreyH on 14/09/2015 18:51:14
Apart from if you talk absolute (heading towards an absolute absence of energy) where you cannot have negative energy.

then A + B would never equal absolute zero. If you can have negative energy in the absolute scale it would be like having negative mass

Not if we are talking purely in terms of kinetic energy. The kinetic energy in the direction of a gravitational field (free fall) can be considered to be negative.
Title: Re: What was before the big bang?
Post by: Bill S on 14/09/2015 20:46:35
Quote from: Timey
In this case, unlike our poor 'nothing', both of the properties, black and white, are already determined as being something, so the analogy is lacking.

That’s why I said   “It makes less sense than saying: “What is wrong with viewing black as being white?”

Quote from: Timey
  Which would actually seem to me to be the most logical scenario for the first instances of creation, when considering the concept of a cyclic universe.

I’m a little confused; are we talking about a universe from nothing, or cyclic universes?
Title: Re: What was before the big bang?
Post by: Bill S on 14/09/2015 21:15:24
Discussions about nothing tend to go round in circles and get nowhere in particular.  I suspect that one reason for this is precisely because it is nothing.  “Nothing” is not really defined in physics for the simple reason that it is not a physical entity.  “Nothing” is a mental concept that rational beings have devised in order to try to talk about the absence of anything/everything.  It is reasonable to think of the physical world as being present even when there is no intelligence to perceive it; but can we say the same about nothing?

Scientists tend to say that they accept things that science can study and define.  By this token, “nothing” is not a subject for science. 

Perhaps the best we can say is that there is nothing in science that shows us a way in which something can emerge from absolutely nothing; so it is reasonable to argue that there can never have been nothing, otherwise there would be nothing now.   The only way in which there could be nothing in the future would be if everything annihilated itself completely.  The accepted conservation laws indicate that this is not possible.  We know that there is something now, so, rationally, “nothing” has no past and no future. It’s present exists only in our minds.

(Edit) Reading through that, it sounds dogmatic, it's not intended to be; it's just out there for discussion.
Title: Re: What was before the big bang?
Post by: timey on 15/09/2015 11:14:08
Saying that nothing, in this context, cannot be defined as a physical reality, because the fact of nothing defines that there can be nothing there - this is a circular argument.
Algebra has its basis in the fact that one can use a parameter that is already defined as an offset to defining the parameter that you are trying to define.  I think that the logical argument that I have used in this instance holds up...

However, I am not going to try further to change your opinion Bill.  One either can see the logic of what I am saying or they can't.  I recognise your argument, I just think mine gets round it...

Scientists tend to say that they accept things that science can study and define.  By this token, “nothing” is not a subject for science. 

Here you are very wrong.  Nothing in the form of a 0 has been the source of much scientific interest.  Since it's discovery as a concept, 0 has been opening up new doors in both mathematics and physics to great importance.
It is a much more difficult proposition to determine the physics of how a universe that doesn't start from nothing began.  In as much as the conservation law negates everything turning back into nothing, the second law tells us that a state must tend to disorder.  These difficulties being the very reasons why it is that so many of the theories state the universe as having started from nothing, including the most widely accepted theory, the Big Bang.

You say you are confused.  ""Am I talking about a universe out of nothing or a cyclic universe?""
What I am doing is looking at the possibility of a cyclic universe that finds its moment of creation in the microscopic region out of nothing.  That each cycle of the universe is larger in size than the last.
This scenario upholds both the second law and the conservation law.

Apart from if you talk absolute (heading towards an absolute absence of energy) where you cannot have negative energy.

then A + B would never equal absolute zero. If you can have negative energy in the absolute scale it would be like having negative mass
o

Not if we are talking purely in terms of kinetic energy. The kinetic energy in the direction of a gravitational field (free fall) can be considered to be negative.

Jeff, that is really interesting!
If we have a 0 state that can hold kinetic energy in the negative, could quantum fluctuations arise under those circumstances?  Do quantum fluctuations carry mass?  If the answer to these questions is yes, I can see the possibility that perhaps the time aspect that we need for these fluctuations to occur in 'could' perhaps be related to mass ""through"" kinetic energy... ??? 
Title: Re: What was before the big bang?
Post by: Bill S on 15/09/2015 17:29:46
Quote from: Timey
However, I am not going to try further to change your opinion Bill

If you can show me how something could emerge from absolutely nothing, I would change my position in an instant, and thank you for doing so.

Quote from: Timey
Nothing in the form of a 0 has been the source of much scientific interest.

That’s true, but “0” is a mathematical concept, it is not “nothing”.  We might look into an “empty” room and say: “There’s nothing in here”.  Obviously, that could be challenged, but none but the most pedantic would do that in  normal circumstances.

The same sort of thing happens in discussions about infinity.  It is often argued that Cantor showed that infinities are amenable to mathematics.  These are mathematical infinities; “absolute infinity” remains unassailed.

Quote from: Timey
In as much as the conservation law negates everything turning back into nothing, the second law tells us that a state must tend to disorder.

Disorder, yes; annihilation, no. 

Quote from: Timey
  These difficulties being the very reasons why it is that so many of the theories state the universe as having started from nothing, including the most widely accepted theory, the Big Bang.

In what way does the Big Bang Theory say that the Universe started from nothing?  To make that claim, the BBT would have to be extrapolated back beyond the BB.

Quote from: Timey
What I am doing is looking at the possibility of a cyclic universe that finds its moment of creation in the microscopic region out of nothing.  That each cycle of the universe is larger in size than the last.
This scenario upholds both the second law and the conservation law.

“…finds its moment of creation….”   Fine sounding words, Timey, but what do they actually mean?  How can nothing find anything in order to become something?

What do you mean by creation?  I believe the present Pope has just acknowledged the BB as the moment of Creation (not that he is the first Pope to do that). I doubt that is what you are talking about, but I must not jump to conclusions.

 “….each cycle of the universe is larger in size than the last.”  Are you saying that more matter/energy is being created with each successive cycle, or simply that the universe expands further each time?
Title: Re: What was before the big bang?
Post by: timey on 15/09/2015 19:08:35
Quote from: Timey
Nothing in the form of a 0 has been the source of much scientific interest.

That’s true, but “0” is a mathematical concept, it is not “nothing”.  We might look into an “empty” rooim and say: “There’s nothing in here”.  Obviously, that could be challenged, but none but the most pedantic would do that in  normal circumstances.

Mathematics and concepts are interchangeable.  One is describing the other and visa versa.
If the universe is a physical reality, which we know it is and have described it as being so mathematically to some great degree, then if we absent everything physical out of it, then everything is physically absent and nothing is the physical result.  This is not some clever word play.  This can be described as a mathematical equation as well as a concept.
Yes, for sure, I agree with you that we are left with a nothing that appears a bit somethingy.  Good, that's the whole point.  Now we have a basis for everything emerging from this nothing that has remained a bit somethingy, on the basis that this nothing is not so nothingy as one might have imagined.  We can in fact actually work with this kind of nothing, whereas the nothing you describe is, I agree, completely useless.  Because we are looking for a description of where and how everything came to be created, to use the reality of everything to determine the reality of nothing is logical.

Quote from: Timey
In as much as the conservation law negates everything turning back into nothing, the second law tells us that a state must tend to disorder.

Disorder, yes; annihilation, no.

I don't understand where annihilation  comes into it.  You'll have to explain where you are coming from with that one.

In what way does the Big Bang Theory say that the Universe started from nothing?  To make that claim, the BBT would have to be extrapolated back beyond the BB.

The Big Bang theory states that everything originated from a point.  A point being a geometrical reality of 0 space and content.

What do you mean by creation?  I believe the present Pope has just acknowledged the BB as the moment of Creation (not that he is the first Pope to do that). I doubt that is what you are talking about, but I must not jump to conclusions.

Yes, I am talking about the first moment of creation.  I'm looking for the 'physics' of it.  Religion is a separate issue in my book, although religious people may say whatever they feel relevant concerning physics, no problem.

“….each cycle of the universe is larger in size than the last.”  Are you saying that more matter/energy is being created with each successive cycle, or simply that the universe expands further each time?

I'm looking at the possibility of more mass/energy being produced during a cycle that increases the size each cycle.  The initial cycles of the universe being of the microscopic, experiencing growth in size and duration, until the dimensions and duration reach the universe that we observe today.

(Edit: I'm sorry, I can't seem to make that multi quote work properly Bill, you'll just have to manage :). )

(Edit 2: think I've sorted it now :)).  )

(Edit 3: I give up !!! Arghh...

(Edit 4: Cracked it! :)
Title: Re: What was before the big bang?
Post by: Bill S on 15/09/2015 21:33:53
It took me some time to work out the multiple quotes.  Even now, if I don’t do it for a while I have to work it out again.

Quote from: Timey
Mathematics and concepts are interchangeable.  One is describing the other and visa versa.

Just because A describes B doesn’t mean that A is B, or that they are interchangeable.

Quote from: Timey
if we absent everything physical out of it

Point of interest: If you absent everything physical, what are you left with?

Quote from: Timey
This can be described as a mathematical equation as well as a concept.

John Barrow has an interesting comment on mathematical existence.
 
“Gradually mathematicians lighted upon a new concept of existence.  Mathematical ‘existence’ meant only logical self-consistency and this neither required nor needed physical existence to complete it.  If a mathematician could write down a set of non-contradictory axioms and rules for deducing true statements from them, then those statements would be said to ‘exist’.” 

Quote from: Timey
I agree with you that we are left with a nothing that appears a bit somethingy.  Good, that's the whole point.  Now we have a basis for everything emerging from this nothing that has remained a bit somethingy, on the basis that this nothing is not so nothingy as one might have imagined

Perhaps I am misinterpreting this, but to me it says: Something can come from nothing, as long as the nothing is really something.

Do we have a referee?  [:D]
Title: Re: What was before the big bang?
Post by: timey on 15/09/2015 22:07:32
Just because A describes B doesn’t mean that A is B, or that they are interchangeable.

If A minus A = B, then of course A is not B.  To initiate an interchange between a state of B and a state of A, we must then find another factor, let's say C, to equate with.

Point of interest: If you absent everything physical, what are you left with?

You are physically left with nothing.

John Barrow has an interesting comment on mathematical existence.
 
“Gradually mathematicians lighted upon a new concept of existence.  Mathematical ‘existence’ meant only logical self-consistency and this neither required nor needed physical existence to complete it.  If a mathematician could write down a set of non-contradictory axioms and rules for deducing true statements from them, then those statements would be said to ‘exist’.”

Yes, which goes to show the importance of defining ones terms, in the instances of both mathematics and logic.

Quote from: Timey
I agree with you that we are left with a nothing that appears a bit somethingy.  Good, that's the whole point.  Now we have a basis for everything emerging from this nothing that has remained a bit somethingy, on the basis that this nothing is not so nothingy as one might have imagined

Perhaps I am misinterpreting this, but to me it says: Something can come from nothing, as long as the nothing is really something.

Do we have a referee?  [:D]

(chuckle)... No need for a referee!  No... you are not misinterpreting this.  The basis of requiring a nothing that 'is' a something has been the whole point of my venture. :)
Title: Re: What was before the big bang?
Post by: Bill S on 15/09/2015 22:41:04
Quote from: Timey
Yes, I am talking about the first moment of creation.  I'm looking for the 'physics' of it.  Religion is a separate issue in my book, although religious people may say whatever they feel relevant concerning physics, no problem.

Religion aside, doesn’t creation require a creator?  Would that be someone/something outside the nothing from which everything was being created?

Quote from: Timey
I'm looking at the possibility of more mass/energy being produced during a cycle that increases the size each cycle.  The initial cycles of the universe being of the microscopic, experiencing growth in size and duration, until the dimensions and duration reach the universe that we observe today.

Would I be right in thinking that this would involve a continuous creation of new matter/energy; similar to the steady state theory, but a bit less “steady”, in that it would come in cycles?

Quote from: Timey
No need for a referee!  No... you are not misinterpreting this.  The basis of requiring a nothing that 'is' a something has been the whole point of my venture.

As John McEnroe used to say: "You cannot be serious!!!!!"
Title: Re: What was before the big bang?
Post by: timey on 15/09/2015 22:59:11
Religion aside, doesn’t creation require a creator?  Would that be someone/something outside the nothing from which everything was being created?

Now you are bordering on philosophy... I am looking for a physical reaction that initiates the creation of the universe,

Would I be right in thinking that this would involve a continuous creation of new matter/energy; similar to the steady state theory, but a bit less “steady”, in that it would come in cycles?

I anticipate an interplay between clumped mass and the black hole phenomenon that produces more particles.

Quote from: Timey
No need for a referee!  No... you are not misinterpreting this.  The basis of requiring a nothing that 'is' a something has been the whole point of my venture.

As John McEnroe used to say: "You cannot be serious!!!!!"

I am in fact being deadly serious...
Title: Re: What was before the big bang?
Post by: Mordeth on 16/09/2015 02:08:20
Quote from: timey

You say that nothing, in the context that we are using it, cannot be defined.  I beg to differ.  Anything can be defined.  Sometimes, in fact for the most part, this only requires that one change ones perspective to the problem.



Regarding nothing, there is nothing to talk about, as I have already laboriously explained.  You can't even conceive it.  You trick your brain into thinking you can, but in fact you cannot.  So you fool yourself.  A dangerous precedent.

And you say "anything can be defined".  You are flat wrong.

Please solve the following equation: 2÷(3-x) where x = 3. 

You will find this to be undefined. Also, a point in geometry is undefined.  There are also algebraic functions which are undefined, such as a singularity.

A verticle line itself has an undefined slope.

In quantum mechanics, if you know the momentum of a test particle, its position is undefined. And vice versa. This result has nothing to do with inadequacies in the measuring instruments timey.  It is a fundamental and intimate property of particles and waves in the subatomic world.

  I can go on and on if you like.  There are many things that are undefined, and not because we lack information, but rather it is intrinsic.  Like absolute nothingness.  Meaningless questions with equally meaningless answers, using undefined terms to explain unobservable events which formulate impossible predictions that lead to untestable conclusions.  Woe is us.
Title: Re: What was before the big bang?
Post by: timey on 16/09/2015 02:33:36
Oh dear Mordeth...  Shall we all just give up the ghost and live with the inadequacies?

I'm not the only physics enthusiast who wishes to push the boundaries.

I'm not the only person who believes that 0 or nothing can be classed as something.

I'm also not the only person to believe that time and the way we perceive it may lead to answers that could reveal a theory of everything.

I am the person who has come up with an alternative time theory.  I do not wish to talk about this idea I have on 'this' thread.
What I am trying to explore here is the moment of creation, which to all intents and purposes, the evidence points to this scenario being started from nothing.

Therefore...an exploration into the potential of nothing is a logical approach, not saying that it will prove fruitful, but that 'is' what I am doing.
Title: Re: What was before the big bang?
Post by: Bill S on 16/09/2015 15:25:42
Quote from: Timey
Quote from: Bill
Religion aside, doesn’t creation require a creator?  Would that be someone/something outside the nothing from which everything was being created?
Now you are bordering on philosophy... I am looking for a physical reaction that initiates the creation of the universe,

That doesn't answer the question, Timey.  Something must initiate this creation.  What I'm trying to find out is if that something is external or internal with respect to the "nothing" from which the "creation" emerges.
Title: Re: What was before the big bang?
Post by: alancalverd on 16/09/2015 15:41:55
C Bruce Rodgers asked the Naked Scientists:
   
Theologians have an explanation for the pre-bang state of the universe, but is there any scientific speculation on the source of the source?  Even a cosmic yo-yo started from something.

What do you think?

Theologians have an explanation for everything, but it is never predictive of anything testable and therefore of no interest whatever.

The test of a "pre big bang theory" will be that it predicts something like the observable universe and is consistent with whatever happens tomorrow.
Title: Re: What was before the big bang?
Post by: Bill S on 16/09/2015 15:45:02
Quote from: Timey
Quote from: Bill
Would I be right in thinking that this would involve a continuous creation of new matter/energy; similar to the steady state theory, but a bit less “steady”, in that it would come in cycles?
I anticipate an interplay between clumped mass and the black hole phenomenon that produces more particles.

Now you’ve lost me; what interplay is that, and how does it produce more particles?
Quote from: Timey
No need for a referee!  No... you are not misinterpreting this.  The basis of requiring a nothing that 'is' a something has been the whole point of my venture.

I really think we need a third party opinion here, ideally from someone with expertise.

You are saying that nothing is something.

I maintain that that is a contradiction in terms.

Is there anybody out there who can find us some common ground, or even tell us both we are nuts, and why?
Title: Re: What was before the big bang?
Post by: timey on 16/09/2015 17:23:16
Ok, Bill... I will try to answer your questions, but first I want to explain to you about my use of the terminology 'progressive thought structures'.  We are discussing an area of physics that is not as yet defined.  To initiate progressive thought it is necessary to look at any situation from an alternative perspective.  This is all I am doing here.  I am using logic to define a situation in order that it may be viewed from an alternative perspective.  Given that we can view this path of logic as a progressive thought structure, we are now free to follow the path of logic further.
This does not necessarily mean that we are going to get anywhere truly significant with it, although we may find that to view the situation from an alternative perspective could afford us some smaller realisation along the way.  I often find this to be the case!
So please know Bill, and Mordeth, that I am not here saying, "this is the way it is", I am here saying, "hey, this is another way of looking at nothing, this being that nothing 'is' something"...

Now if we take what Alan has said:

The test of a "pre big bang theory" will be that it predicts something like the observable universe and is consistent with whatever happens tomorrow.

In relation to what Jeff's post is relating:

If we have two values of energy where A is positive and B is negative then we can have a situation where A + B = 0. You then have a mathematical nothing and yet the energy has not gone away.

Apart from if you talk absolute (heading towards an absolute absence of energy) where you cannot have negative energy.

then A + B would never equal absolute zero. If you can have negative energy in the absolute scale it would be like having negative mass

Not if we are talking purely in terms of kinetic energy. The kinetic energy in the direction of a gravitational field (free fall) can be considered to be negative.

Jeff, that is really interesting!
If we have a 0 state that can hold kinetic energy in the negative, could quantum fluctuations arise under those circumstances?  Do quantum fluctuations carry mass?  If the answer to these questions is yes, I can see the possibility that perhaps the time aspect that we need for these fluctuations to occur in 'could' perhaps be related to mass ""through"" kinetic energy... ???

Here we can see that by defining nothing as a physical reality, be this correct or not, we have opened up a discussion about the concept of a moment of creation.  In this discussion, we see that it is necessary in nothing for time to not exist, and to then start existing in order for anything to occur.  If there can be merit in the idea's above, then the phenomenon of time being perhaps related to mass, and therefore to gravity, via kinetic energy is an idea that we can relate back to our state of everything today and experiment with.

Quote from: Timey
Quote from: Bill
Would I be right in thinking that this would involve a continuous creation of new matter/energy; similar to the steady state theory, but a bit less “steady”, in that it would come in cycles?
I anticipate an interplay between clumped mass and the black hole phenomenon that produces more particles.

Now you’ve lost me; what interplay is that, and how does it produce more particles?

As I said, this is an 'anticipated' interplay.  Basically this means I don't know :D... I observe matter in my garden multiplying.  Sacks of the stuff must be removed every year in order to maintain clear space.  Perhaps particles can multiply???

I really think we need a third party opinion here, ideally from someone with expertise.

You are saying that nothing is something.

I maintain that that is a contradiction in terms.

Is there anybody out there who can find us some common ground, or even tell us both we are nuts, and why?

I'm good with that :) ...

I am saying that if everything is physically absent then nothing is the physical result.  This rendering nothing as a physical reality.

Bill says that nothing cannot be a physical reality, that this is a contradiction in terms.

If we could have an expert opinion at-all please?
Title: Re: What was before the big bang?
Post by: Bill S on 16/09/2015 19:29:57
Quote from: Timey
Bill says that nothing cannot be a physical reality, that this is a contradiction in terms.

Just to clarify: in your scheme of things does "physical reality" = "something"?
Title: Re: What was before the big bang?
Post by: timey on 16/09/2015 20:06:47
Yes, to be clear, I am proposing that the "physical reality" of nothing, determined by the physical absence of everything = something.
Title: Re: What was before the big bang?
Post by: Bill S on 16/09/2015 22:54:40
Quote from: Timey
I observe matter in my garden multiplying.  Sacks of the stuff must be removed every year in order to maintain clear space.  Perhaps particles can multiply???

Particles can multiply in the same way matter accumulates in your garden; ie by the natural process of the motion of material from one place to another; or if your garden is like mine; by passers-by throwing stuff in.  I doubt that any of this stuff comes from directly nothing.  [:)]
Title: Re: What was before the big bang?
Post by: timey on 17/09/2015 09:39:07
Well Bill, I have heard some pretty way out stuff, whereas the world of quantum paralleled to the macro world that we live in might well have an empty beer can or other such rubbish appear in your garden as if from nothing... :D

But seriously, if you can remember that I did also state this:

As I said, this is an 'anticipated' interplay.  Basically this means I don't know :D...

Of course matter build up in the garden is particles being displaced from one place to another.  We do not observe that particles multiply/breed.  If this multiplication of particles happens at-all, then it would probably be inside a black hole.  This however is just a 'nutty' idea of my own.  The only relevance it bears upon this conversation is due to my 'looking' at a creation moment in the microscopic region.  And this only because I want to look at the first cycles of the universe consisting of a minimal amount of particles, with each cycle becoming bigger in size/amount of particles, and longer in duration than the last.

This doesn't mean that looking at the possibility of a creation moment in the microscopic region wouldn't have any potential relevance to any other theory though...
Title: Re: What was before the big bang?
Post by: timey on 17/09/2015 15:41:44
As a footnote to the post above, I think it worth mentioning Bill, that as to my multiplication of particles notion that increases a universes size each cycle...

IF there was any chance that we 'could' bring forth a particle from quantum fluctuations that 'can' occur out of this 0 state, then it could be said that a state that is close to 0, where quantum fluctuations are occurring, can produce more particles.  Therefore we have a cause for a universe's cycle getting bigger in size/amount of particles.  Any part of the universe that is close to a 0 state will be producing more particles...

Just another perspective :)
Title: Re: What was before the big bang?
Post by: timey on 17/09/2015 17:29:11
...and to take this path of logic further,  IF we were to consider that a 0 state 'could' arise quantum fluctuations that 'can' produce a particle/particles...  Then we can now 'perhaps' dispense with the cyclic universe notion in favour of an eternal cosmos scenario that holds it's moment of creation in the microscopic, and has been expanding ever since.  A Big Bang scenario turned Little Bang if you like.
This scenario would uphold both the second law and the conservation of energy law.  It would explain expansion, and accelerated expansion and would be suggestive of this expansion continuing to infinity.

Again, how the time aspect might emerge in the scenario of this moment of creation in the microscopic, and what rate it is occurring at, would, I believe, be incredibly relevant...
Title: Re: What was before the big bang?
Post by: Bill S on 17/09/2015 22:22:11
Let’s see if I am beginning to understand your idea. 

Nothing was eternal into the past.  At some point, something, possibly a particle, appeared in this nothing.  The particle had energy which caused it to “explode” into a number of particles.  These particles constituted the universe.  They expanded until something, possibly gravity, pulled them back together into a small crunch, which initiated a new expansion.  The second expansion followed the same pattern as the first, but with additional matter/energy.  Cycles have continued, on increasing scale, until our Universe emerged.  This cyclic process may continue for ever.

How’s that for a first attempt?
Title: Re: What was before the big bang?
Post by: Mordeth on 18/09/2015 01:12:48
Quote from: Bill S
Is there anybody out there who can find us some common ground, or even tell us both we are nuts, and why?

He is trying to create a logical bridge between "nothing" and "something". Unfortunately, no logical bridge exists.  So, what is being asked is an impossible explanatory demand. Namely,  to determine the reason for the existence of something (everything) without using an existential premise. It is an explanatory trap. So one tries to define the indefinable and then rationalize the belief with the logical fallacy of affirming the consequent.  ie.,

If there is nothing then there is something.
There is something, therefore there is nothing.

You are both arguing over the very nonexistence of reality itself.  This "nothing" cannot be explained, imagined, calculated or defined.

Infinite regress is also a very real problem in attempting to understand the origin of everything, and I have painstakingly explained this.  Furthermore, there is no event prior to the Big Bang that is observable and this will probably always be the case.  So how do you plan to test these grand conclusions?  Do you think you can logically deduce the most important question that ever was asked?

Title: Re: What was before the big bang?
Post by: Bill S on 18/09/2015 14:56:07
Quote from: Mordeth
If there is nothing then there is something.
There is something, therefore there is nothing.

That is precisely what I am not saying.  My position would be better expressed as something like this.

If there is nothing, there cannot be something.
Manifestly, there is something, therefore there can never have been nothing.

That is an oversimplification, but gives the general idea.

In continuing the discussion, what I am trying to do is understand Timey’s line of reasoning.
Title: Re: What was before the big bang?
Post by: timey on 18/09/2015 18:43:09
Let’s see if I am beginning to understand your idea. 

Nothing was eternal into the past.  At some point, something, possibly a particle, appeared in this nothing.  The particle had energy which caused it to “explode” into a number of particles.  These particles constituted the universe.  They expanded until something, possibly gravity, pulled them back together into a small crunch, which initiated a new expansion.  The second expansion followed the same pattern as the first, but with additional matter/energy.  Cycles have continued, on increasing scale, until our Universe emerged.  This cyclic process may continue for ever.

How’s that for a first attempt?

Hi Bill... Almost :)

A particle is produced by quantum fluctuations that emerge from nothing.  More particles arise from further fluctuations that are rendered more possible by the existence of this particle.  These particles clump together, form a black hole, the black hole jets these particles into the next cycle.  More particles gives rise to more likely conditions for more fluctuations to emerge more particles, particles clump, form black hole, etc.

Or ... The fluctuations that emerged the first particles were 'special' and the black hole produces more particles, by particles that have been primed for reproduction by extreme compression.

Post 183 and 184, this path of logic runs past the same scenario of creation moment, but without the cyclic universe, black hole beginning and ending cycle notion.  This bearing more resemblance to the universes 'observed' expansion and accelerated expansion notion, this observation being based on the supposition of the causation of redshift.  Both dark matter and dark energy would have explaination.

Thanks Bill for wanting to understand my line of reasoning.  Appreciated!

Mordeth, I am in fact a 'she' and am now concentrating on preparing an answer to your post...an answer that will propose exactly why it is that I feel that logic 'can' prevail and that there lies the possibility that quantum fluctuations can arise from nothing and how particles may emerge from these conditions.

But... In the meantime.   If you are both proposing that there cannot be a reality of nothing, then what exactly are you proposing did come before the moment of creation/Big Bang?
Title: Re: What was before the big bang?
Post by: timey on 18/09/2015 21:39:14
https://en.m.wikipedia.org/wiki/Infinite_regress

Infinite regress:
If proposition 1 = that nothing is a reality.  Proposition 1 relies on proposition 2 = everything is a reality.  Proposition 3 relies on proposition 2 = in that proposition 2 minus proposition 2 = proposition 1... Proposition 4 = a progression from proposition 1 into proposition 2 (this relies on propositions 1, 2 and 3 and is yet to be defined). Proposition n - 1 = a progression of everything minus the reality of nothing (infinite nothing perhaps, but not nothing infinitely due to no time scale) and proposition n = everything progressing infinitely.

Is this an infinite regress?  (Not sure if I am applying the symbols correctly).  A virtuous circle perhaps?  ... I personally do not view this as an illogical concept.  It reads more like  an algebraic equation in my eyes, but perhaps it's just me.

In any case I am certainly not the first person in the world to consider nothing as a potential reality that everything else can emerge from.  It really does surprise me that we are having such trouble getting past first base here!  I would have thought that the attempts that quantum physicists have made in their explorations into a vacuum state speak clearly of this type of investigation being oriented to a creation moment as well as searching for a quantum unification with gravity.

Truly, I think its time I stopped beating around the bush and give it to you straight.

Purely from the information given in the following 3 links, by adding one additional concept I am going to tell you a possibility that I think 'may' have the potential to lead to 'the' theory of everything mentioned in these links.

If you both read these inks in full, please note direct evidence of physicist considering nothing to be something.

https://en.m.wikipedia.org/wiki/Vacuum_state

https://en.m.wikipedia.org/wiki/Cosmological_constant

https://en.m.wikipedia.org/wiki/Virtual_particle

Quote:
"According to Astrid Lambrecht (2002): "When one empties out a space of all matter and lowers the temperature to absolute zero, one produces in a Gedankenexperiment the quantum vacuum state."[1]"
Unquote

Quote:
"According to quantum mechanics, the vacuum state is not truly empty but instead contains fleeting electromagnetic waves and particles that pop into and out of existence.[3][4][5]"
Unquote

Quote:
"In many situations, the vacuum state can be defined to have zero energy, although the actual situation is considerably more subtle. The vacuum state is associated with a zero-point energy, and this zero-point energy has measurable effects. In the laboratory, it may be detected as the Casimir effect. In physical cosmology, the energy of the cosmological vacuum appears as the cosmological constant. In fact, the energy of a cubic centimeter of empty space has been calculated figuratively to be one trillionth of an erg (or 0.6 eV).[8] An outstanding requirement imposed on a potential Theory of Everything is that the energy of the quantum vacuum state must explain the physically observed cosmological constant."
Unquote:

Quote:
"The presence of virtual particles can be rigorously based upon the non-commutation of the quantized electromagnetic fields. Non-commutation means that although the average values of the fields vanish in a quantum vacuum, their variances do not.[15] The term "vacuum fluctuations" refers to the variance of the field strength in the minimal energy state,[16] and is described picturesquely as evidence of "virtual particles".[17]

It is sometimes attempted to provide an intuitive picture of virtual particles based upon the Heisenberg energy-time uncertainty principle:

(with ΔE and Δt being the energy and time variations respectively; ΔE is the accuracy in the measurement of energy and Δt is the time taken in the measurement, and ħ is the Planck constant divided by 2π) arguing along the lines that the short lifetime of virtual particles allows the "borrowing" of large energies from the vacuum and thus permits particle generation for short times.[18]

Although the phenomenon of virtual particles is accepted, this interpretation of the energy-time uncertainty relation is not universal.[19][20] One issue is the use of an uncertainty relation limiting measurement accuracy as though a time uncertainty Δt determines a "budget" for borrowing energy ΔE. Another issue is the meaning of "time" in this relation, because energy and time (unlike position q and momentum p, for example) do not satisfy a canonical commutation relation (such as [q, p] = i ħ).[21] Various schemes have been advanced to construct an observable that has some kind of time interpretation, and yet does satisfy a canonical commutation relation with energy.[22][23] The very many approaches to the energy-time uncertainty principle are a long and continuing subject.[23]"
Unquote

Quote:
"A virtual particle does not necessarily appear to carry the same mass as the corresponding real particle. This is because it appears as "short-lived" and "transient", so that the uncertainty principle allows it to appear not to conserve energy and momentum. The longer a virtual particle appears to "live", the closer its characteristics come to those of an actual particle.
Virtual particles appear in many processes, including particle scattering and Casimir forces. "
Unquote:

Quote:
"Many physicists believe that, because of its intrinsically perturbative character, the concept of virtual particles is often confusing and misleading, and is thus best avoided.[4][5]"
Unquote:

Physicists have been juggling these concepts around for years.  The suggestion I make has never been made before by anyone else.  It has not been considered.

GR describes time dilation perfectly. We can run GPS, mobile phones, set your clock by it.  There is no disputing this... But what sort of time dilation is GR describing?
I propose that GR is describing a mass near mass time dilation effect, and that proper locational gravitational time dilation has been completely overlooked as a result of GR's assertions.

I propose that locational gravitational time dilation is as widely variant in its scale as the strength of a gravity field is. (I can explain this further but for now if you will accept this experimentally as the premiss)

Any mathematical structure that is based on a time measurement in relation to a momentum or a length needs to take into account the rate time is occurring at for the subject matter being measured.

In the case of a "perfect" vacuum state, (this being a one time occurrence) time would be set at zero.  For 'whatever reason' it may have cause to occur, a quantum fluctuation (Casimir effect) would have to initiate time as it emerges.  Time is now set at notch 1.  It is occurring really very slowly indeed. Therefore giving rise to more potential for other quantum fluctuations to occur, producing virtual particles that will, in this rate of slow time, not be "fleeting"!

Quote:
"Quantum field theory   Edit
See also: Vacuum catastrophe
List of unsolved problems in physics
Why can't the zero-point energy of the vacuum be interpreted as a cosmological constant? What causes the discrepancies?
A major outstanding problem is that most quantum field theories predict a huge value for the quantum vacuum. A common assumption is that the quantum vacuum is equivalent to the cosmological constant. Although no theory exists that supports this assumption, arguments can be made in its favor.[14]

Such arguments are usually based on dimensional analysis and effective field theory. If the universe is described by an effective local quantum field theory down to the Planck scale, then we would expect a cosmological constant of the order of (it didn't print the maths here). As noted above, the measured cosmological constant is smaller than this by a factor of 10−120. This discrepancy has been called "the worst theoretical prediction in the history of physics!".[15]"
Unquote

Wouldn't this problem be helped by the fact of an early universe that occurred in much 'slower time'?

Wouldn't this problem be exaggerated by a mathematically flawed Planck scale?

Wouldn't the concept of quantum physics be adversely affected by a mathematically flawed Planck's h constant?

Taking this to the other extreme, we can now have a cause for the Hawking's temperature quandary.

Now, I 'may well' not be right (chuckle), but what I am suggesting presents a very simple idea as a solution to some long standing physics problems, and this idea 'is' based in logic.
Title: Re: What was before the big bang?
Post by: dlorde on 18/09/2015 23:59:29
As I understand it, the vacuum state with no particles or forces (other than virtual particles) is what is often referred to as 'nothing' by physicists. Krauss has said that when he talks of a universe from 'nothing', this is what he means - empty or 'raw' spacetime. He doesn't mean nothing as in the complete absence of spacetime or anything else. Empty spacetime isn't the complete absence of anything; the quantum fields are all present, randomly oscillating around zero due to quantum uncertainty, but the Higgs field is non-zero (from Sean Carroll's 'The Particle at the End of the Universe').

It seems to me that suggesting that the complete absence of anything physical is itself physical, is analogous to claiming that not collecting stamps is a hobby and not pulling a rabbit out of a hat is a magic trick. Semantic legedermain.
Title: Re: What was before the big bang?
Post by: timey on 19/09/2015 00:10:53
Clearly you have not taken on board the concept of reducing a vacuum state to absolute-ness.  ie: : A 'perfect' vacuum state...
Title: Re: What was before the big bang?
Post by: Bill S on 19/09/2015 00:13:32
Timey, it's going to take me a few hits to work through your last couple of posts, but one thing strikes me straight away.

Quote
A particle is produced by quantum fluctuations that emerge from nothing.

A quantum fluctuation is a transient variation in the level of energy at a given point.  It emerges from the vacuum energy, not from "nothing".
Title: Re: What was before the big bang?
Post by: dlorde on 19/09/2015 00:28:33
Clearly you have not taken on board the concept of reducing a vacuum state to absolute-ness.  ie: : A 'perfect' vacuum state...
You mean 'free space'? That's what I was talking about.

Quote from: Wikipedia
Physicists often discuss ideal test results that would occur in a perfect vacuum, which they sometimes simply call "vacuum" or free space...
[Vacuum (https://en.wikipedia.org/wiki/Vacuum)]

At university we used to discuss crackpot inventions, like the 'Hackenthorpe Knife', which was so sharp it would slice through its own blade if left facing up, or the 'Hackenthorpe Vacuum', made by taking an ordinary vacuum and sucking all the vacuum out. They were absurdist student jokes. A box with literally 'nothing inside' would be completely flat, with nothing between its sides - i.e., the surfaces would be touching.
Title: Re: What was before the big bang?
Post by: timey on 19/09/2015 00:36:43
So...dlorde.  You are disputing the Big Bang theory, everything did not emerge from a point, please elaborate!
Title: Re: What was before the big bang?
Post by: timey on 19/09/2015 00:38:29
Timey, it's going to take me a few hits to work through your last couple of posts, but one thing strikes me straight away.

Quote
A particle is produced by quantum fluctuations that emerge from nothing.

A quantum fluctuation is a transient variation in the level of energy at a given point.  It emerges from the vacuum energy, not from "nothing".

Yes Bill.  If you remove all matter from a situation, you will be left with a vacuum state in which quantum fluctuations occur.  Remove the quantum fluctuations and you might be left with a 'perfect' vacuum state.

We can even describe this state as a zero geometry, zero content point.  But this state IS still something.  This  perfect vacuum state has the potential for quantum fluctuations to arise.  Why?  Because it is the perfect vacuum state and being as we are saying this is the beginning of our universe, it is subject to the second law.  A state of order must alway tend to disorder.  We know that if we remove all matter from a space that we will be left with a vacuum state that has quantum fluctuations.  Therefore to add matter back into a vacuum state we can see that it must require the vacuum state to have quantum fluctuations... Surely?  That would be logical, right?

Also, you are saying that a quantum fluctuation is 'transient'.  Agreed, but if the rate of time is occurring much more slowly than the time we are measuring it with, perhaps a quantum fluctuation is not quite so fleetingly transient as 'we' observe.  Not forgetting that any experiment in a lab would not reflect the rate time would be occurring at for a quantum fluctuation that was not occurring in the presence of any other related mass...
Title: Re: What was before the big bang?
Post by: Mordeth on 19/09/2015 05:44:48
Quote from: timey
Mordeth, I am in fact a she...

My apologies.

Quote from: timey
But... In the meantime.   If you are both proposing that there cannot be a reality of nothing, then what exactly are you proposing did come before the moment of creation/Big Bang?

I am not proposing anything.  I have said no less than 20 times that I do not know, nor does science. All we can do is guess. 
Title: Re: What was before the big bang?
Post by: timey on 19/09/2015 10:08:19
Quote from: timey
Mordeth, I am in fact a she...
My apologies.

No problem, the user name is not indicative of gender.  I perhaps should have called myself Mrs.timey.

Quote from: timey
But... In the meantime.   If you are both proposing that there cannot be a reality of nothing, then what exactly are you proposing did come before the moment of creation/Big Bang?

I am not proposing anything.  I have said no less than 20 times that I do not know, nor does science. All we can do is guess.

I am not in disagreement with you Mordeth.  Any inroads that I am 'attempting' to make here into 'unknown' territories, are based on guesswork.  However this guesswork is 'informed' guesswork that is based in logic and my line of inquiry introduces a 'new concept' to the equation.

As Alan Calvard said earlier this thread:
"The test of a "pre big bang theory" will be that it predicts something like the observable universe and is consistent with whatever happens tomorrow."

IF what I am suggesting concerning time and its emergence in the initial moment of creation, from not occurring at-all - to occurring at an extremely slow rate that increases its speed as more mass is introduced into the universe... IF this scenario is viable, then this pre-creation theory IS testable in our universe today!  And would also be predictive of what our universe can expect of its tomorrow's!
Title: Re: What was before the big bang?
Post by: Bill S on 19/09/2015 23:10:41
Quote from: Timey
Therefore to add matter back into a vacuum state we can see that it must require the vacuum state to have quantum fluctuations... Surely?  That would be logical, right?


I would agree with that, as long as you are not saying that this vacuum, which has energy and fluctuations, is nothing.  Unless you are using "nothing" in the way that (e.g.) Krauss uses it, in which case, by his own admission it is not nothing.   
Title: Re: What was before the big bang?
Post by: Bill S on 20/09/2015 00:09:54
Since we are dealing with distinctly non-mainstream ideas; the problem of infinite regression in relation to the Universe is not so difficult to overcome, without getting into theology.  The trouble is that the concept requires fully accepting that infinity is not a number, and eternity is not time.  It is not difficult to get scientific people to accept this, but if you try to go a step further and ask them to agree that there can be no change in infinity, and that there cannot be an infinite succession, and you hit a brick wall.   

It is important to stress that I am not attempting to be dogmatic here, I am not even saying that this is what I believe, I am merely putting forward some of the ideas that have come into my head in the course of thinking about the possible origin of the Universe, and in particular about infinity.  I gladly accept that the ideas are there to be knocked down, and in fact welcome that as part of my own learning process. 

    I invite you to consider the possibility that the cosmos is infinite. (Whenever I use the word “infinite” I include “eternal” within the term).  I am, here, picking up the idea, that appears in some scientific circles, that the four dimensions of spacetime that we experience are only a shadow of a higher-dimensional reality that is beyond our reach.  I am suggesting that the cosmos has infinite dimensions, or perhaps just one infinite dimension, which we cannot experience.  Like zero, whatever you divide, or multiply infinity by, it remains unchanged.  However, unlike zero, infinity should remain unchanged if you try to add something to it, or subtract something from it, because, if it is truly infinite, whatever you “add” will already be part of it, and whatever you try to subtract will still remain part of it, otherwise, it would no longer be infinite.  Even when trying to explain this idea, we run into problems with terminology.  Strictly, I should not talk about “parts” of infinity; surely, it has no parts; that is of the essence of infinity.  Leaving aside the mathematical “infinities” that almost inevitably find their way into this kind of discussion; it feels as though no part of a true infinity should be thought of as being finite, because a true infinity cannot be sub-divided.  In eternity, time should receive the same treatment, it too is eternal; of course this is also a contradiction in terms; what I should really say is that time, as we understand it does not exist.  There is no passage of time, in the cosmos, nor do we, in our Universe, pass through time.  We live in an eternal, unchanging now, of which we can experience only a shadow. 
Title: Re: What was before the big bang?
Post by: jeffreyH on 20/09/2015 00:14:56
What happens if the expression is infinity minus infinity or infinity divided by infinity?
Title: Re: What was before the big bang?
Post by: timey on 20/09/2015 00:43:07
Quote from: Timey
Therefore to add matter back into a vacuum state we can see that it must require the vacuum state to have quantum fluctuations... Surely?  That would be logical, right?


I would agree with that, as long as you are not saying that this vacuum, which has energy and fluctuations, is nothing.  Unless you are using "nothing" in the way that (e.g.) Krauss uses it, in which case, by his own admission it is not nothing.

Ok Bill

Now take the vacuum state with its quantum fluctuations and remove them.  What are we left with?  Empty space.  Then take away the geometry of space.  We have a point.  Now take away time.  What we are now left with is something that has nothing of our universe in it.

This is the 'nothing' that I refer to.  It exists as 'something' only because we have subtracted everything from it.

Now all that we need to do is work out which order these parameters that we have removed are to be added back, in order for our universe to evolve.
Title: Re: What was before the big bang?
Post by: timey on 20/09/2015 01:22:32
I do not understand your visualisation of time Bill.  Time is a phenomenon.  I would even go so far as to say time is a force.  It does something.  Without time there can be no motion.   We know that the rate of time is a variable.  We observe there to be a change in the rate of time due to changes in a gravity field.  This is indicative of the phenomenon of time being part and parcel of the structure of our universe.  Our measurement of time, and how we then perceive it is a completely separate issue.

Ok, well I've looked up definitions of the words infinite, infinitely and infinity...

These words describe something that is immeasurably large, immeasurably small, immeasurably long, etc.  I actually come to see no need for the use of these words in the context of this subject matter.

In the case of nothing, nothing is just something we must add everything into for everything to exist.  In the case of everything we can say that it possibly goes on forever and possibly gets bigger.  And that these particular parameters of everything are immeasurable within our "current" understanding.

The Big Bang theory states that everything emerged from a point.  The Big Bang is a mainstream physics theory. On the basis that emerging everything from this point in the time scale given by the Big Bang theory is 'troublesome' - we are deviating from mainstream physics only in that we are looking at a creation moment that occurs in the microscopic region and evolves from the microscopic into our universe today 'slowly'.

Now does anyone at-all want to talk about how time, space and a Casimir fluctuation 'might' arise from a zero geometry, zero content, point?  Because for me, this is the most pertinent part of the discussion!
Title: Re: What was before the big bang?
Post by: Mordeth on 20/09/2015 02:45:43
Since we are dealing with distinctly non-mainstream ideas; the problem of infinite regression in relation to the Universe is not so difficult to overcome, without getting into theology.  The trouble is that the concept requires fully accepting that infinity is not a number, and eternity is not time.  It is not difficult to get scientific people to accept this, but if you try to go a step further and ask them to agree that there can be no change in infinity, and that there cannot be an infinite succession, and you hit a brick wall.   

It is important to stress that I am not attempting to be dogmatic here, I am not even saying that this is what I believe, I am merely putting forward some of the ideas that have come into my head in the course of thinking about the possible origin of the Universe, and in particular about infinity.  I gladly accept that the ideas are there to be knocked down, and in fact welcome that as part of my own learning process. 

    I invite you to consider the possibility that the cosmos is infinite. (Whenever I use the word “infinite” I include “eternal” within the term).  I am, here, picking up the idea, that appears in some scientific circles, that the four dimensions of spacetime that we experience are only a shadow of a higher-dimensional reality that is beyond our reach.  I am suggesting that the cosmos has infinite dimensions, or perhaps just one infinite dimension, which we cannot experience.  Like zero, whatever you divide, or multiply infinity by, it remains unchanged.  However, unlike zero, infinity should remain unchanged if you try to add something to it, or subtract something from it, because, if it is truly infinite, whatever you “add” will already be part of it, and whatever you try to subtract will still remain part of it, otherwise, it would no longer be infinite.  Even when trying to explain this idea, we run into problems with terminology.  Strictly, I should not talk about “parts” of infinity; surely, it has no parts; that is of the essence of infinity.  Leaving aside the mathematical “infinities” that almost inevitably find their way into this kind of discussion; it feels as though no part of a true infinity should be thought of as being finite, because a true infinity cannot be sub-divided.  In eternity, time should receive the same treatment, it too is eternal; of course this is also a contradiction in terms; what I should really say is that time, as we understand it does not exist.  There is no passage of time, in the cosmos, nor do we, in our Universe, pass through time.  We live in an eternal, unchanging now, of which we can experience only a shadow.

Hi Bill,

Please incorporate the 2nd Law of Thermodynamics into your "theory".  Are you saying all macroscopic behavior is reversible?  It is therefore time asymmetric?  Do you feel there can be a universal state of maximum entropy where complete equilibrium exists?  If the cosmos is eternal, why are we not in this state now?  Why did we evolve from a clearly low entropy state in the past? 
Title: Re: What was before the big bang?
Post by: dlorde on 20/09/2015 12:13:42
So...dlorde.  You are disputing the Big Bang theory, everything did not emerge from a point, please elaborate!
I'm not disputing the standard model of cosmology (i.e. the current consensus), that the universe expanded very rapidly from a very dense, very hot state, and that while the observable universe would have been sub-atomic - perhaps a few planck lengths - in scale prior to expansion (inflation), the scale of the whole universe at that time is unknown and might be infinite (it's certainly much bigger than the observable universe). It isn't known what the state of the universe was prior to that - GR predicts a singularity, which is pretty unpopular because the physics that gets us that far back fails there, and it doesn't account for the quantum effects that are expected to have predominated at that time. There seems to be some support for the idea that a 'bounce' would occur, rather than a singularity - that the BB was a sort of rebound.
Title: Re: What was before the big bang?
Post by: dlorde on 20/09/2015 12:33:39
...Like zero, whatever you divide, or multiply infinity by, it remains unchanged.  However, unlike zero, infinity should remain unchanged if you try to add something to it, or subtract something from it, because, if it is truly infinite, whatever you “add” will already be part of it, and whatever you try to subtract will still remain part of it, otherwise, it would no longer be infinite.  Even when trying to explain this idea, we run into problems with terminology.  Strictly, I should not talk about “parts” of infinity; surely, it has no parts; that is of the essence of infinity.  Leaving aside the mathematical “infinities” that almost inevitably find their way into this kind of discussion; it feels as though no part of a true infinity should be thought of as being finite, because a true infinity cannot be sub-divided.
Mathematical infinities do differ from what you describe here; is this description of physical infinity your own construction, or do you have a source for it?

If you have a source, I'd love to see the reference. If not, do you have some rationale for the properties of your description of physical infinity, or is it just what 'feels right'?
Title: Re: What was before the big bang?
Post by: Bill S on 21/09/2015 15:19:57
Quote from: Jeffrey
What happens if the expression is infinity minus infinity or infinity divided by infinity?

If you give me a practical example of either of those; then perhaps we can work from there. You know what I'm like with maths.  [:(]
Title: Re: What was before the big bang?
Post by: Bill S on 21/09/2015 16:06:29
Quote from: Timey
Now take the vacuum state with its quantum fluctuations and remove them.  What are we left with?  Empty space.  Then take away the geometry of space.  We have a point. 

Sorry, Timey, I can’t see the point. [:)]  Where did you find it?

Quote from: Timey
  Now take away time.  What we are now left with is something that has nothing of our universe in it.

Only if you are saying that nothing is “something”, which, obviously you are.

Quote from: Timey
This is the 'nothing' that I refer to.  It exists as 'something' only because we have subtracted everything from it.

This makes sense only if you can stand outside your “nothing” and manipulate it.  If that’s the case, you have not reached “nothing”.

Quote from: Timey
Now all that we need to do is work out which order these parameters that we have removed are to be added back, in order for our universe to evolve.

If there really was nothing, who/what would add these parameters back in?

Duty calls; more later.
Title: Re: What was before the big bang?
Post by: Bill S on 21/09/2015 17:36:45
Quote from: Timey
I do not understand your visualisation of time Bill.

#199 contains only the briefest outline of nearly 60 years of thinking and talking about infinity, nothing and time.  The baldest essentials are that the cosmos (=everything that could ever be) is infinite and unchanging.  Our Universe is “part” of this (I would need to go into more detail than is appropriate here to explain the use of “part”). 

Quote from: Timey
Time is a phenomenon.  I would even go so far as to say time is a force.  It does something.  Without time there can be no motion

Time is a measure of change; motion is an aspect of change.  Change is an “illusion” that lets us make sense of our Universe.

Quote from: Timey
Now does anyone at-all want to talk about how time, space and a Casimir fluctuation 'might' arise from a zero geometry, zero content, point?  Because for me, this is the most pertinent part of the discussion!

I hope someone comes in on this, I think it would be a fascinating discussion, but I can’t help with it because the idea of something emerging from nothing is a non-starter for me.  It’s a bit like trying to answer the question: “How would I feel about this if I didn’t exist?”
Title: Re: What was before the big bang?
Post by: Bill S on 21/09/2015 17:51:11
Quote from: dlorde
If you have a source, I'd love to see the reference. If not, do you have some rationale for the properties of your description of physical infinity, or is it just what 'feels right'?

As I said above, it is the result of years of struggling, in the course of which I have been labelled as all kinds of a crackpot.  I felt like the voice of one crying in the wilderness until I read Julian Barbour's " The End of Time".  This is about the only direction in which I could point you for anything approaching a scientific view with any similarity to mine.  Could JB be a crackpot as well; If so, I'm in fairly good company. [:)]

Title: Re: What was before the big bang?
Post by: Bill S on 21/09/2015 18:16:33
Quote from: Mordeth
Are you saying all macroscopic behavior is reversible?  It is therefore time asymmetric?

I thought time asymmetry involved non-reversibility.  Have I got it the wrong way round?

Quote from: Mordeth
Do you feel there can be a universal state of maximum entropy where complete equilibrium exists?  If the cosmos is eternal, why are we not in this state now?  Why did we evolve from a clearly low entropy state in the past?

If the concept of entropy applies to the infinite cosmos, the state would seem logically to be that of maximum entropy; but entropy may be only a feature of our experience of the Universe.

"Why" questions do tend to be philosophical.  If we are not in that state now, I have no idea why that would be.

My question would be: How do you know we are not in that eternal/unchanging cosmos, and that our perceptions of time, change etc are not 3+1 dimensional "shadows" of a higher dimensional reality?  At best, that sounds like philosophy, and at worst, sci. fi. but the thinking behind it underlies that most fundamental question: "How can we be here?".

 

 
Title: Re: What was before the big bang?
Post by: dlorde on 22/09/2015 14:18:33
... Could JB be a crackpot as well; If so, I'm in fairly good company. [:)]
Thanks, I'll have a browse of his stuff. I don't think going against prevailing opinion necessarily makes one a crackpot, and it's a rare crackpot who's taken seriously enough to generate controversy in the field.
Title: Re: What was before the big bang?
Post by: jeffreyH on 22/09/2015 21:21:08
If time does not exist then the universe is made up of an infinite number of connected states. The only relevant consideration is then the changes in those states. These connections would then explain entanglement but not the so called spooky action at a distance or quantum teleportation. Light is then just the state that that everything else relates to. If at the quantum level things happen in discreet jumps then time is completely removed from the equation. These jumps are then the actual changes in the states of the system.
Title: Re: What was before the big bang?
Post by: Bill S on 22/09/2015 22:22:30
Quote from: Jeffrey
If time does not exist then the universe is made up of an infinite number of connected states.

I think I’m going to have to post the whole “Infinite Cosmos” idea in order to deal with things like infinite numbers and connected states.  The trouble is, it’s a bit long, and I think there’s a “law” that says the number of people who read a post is inversely proportional to the length of the post.

Quote from: Jeffrey
These connections would then explain entanglement but not the so called spooky action at a distance or quantum teleportation

Thanks for pointing that out; I think the I C idea would offer an explanation for both.  I must give that some thought.
Title: Re: What was before the big bang?
Post by: Bill S on 22/09/2015 22:37:55
Dlorde, I think David Bohm's work might be worth investigating as well.  Michael Talbot's "The Holographic Universe", if I remember rightly, touches on this, but also contains a lot of weird stuff.   [:)]
Title: Re: What was before the big bang?
Post by: dlorde on 22/09/2015 23:41:01
Dlorde, I think David Bohm's work might be worth investigating as well.  Michael Talbot's "The Holographic Universe", if I remember rightly, touches on this, but also contains a lot of weird stuff.   [:)]
I've read Bohm's 'The Undivided Universe' and 'Wholeness and the Implicate Order', and it's pretty boggling - I'm not really sure whether I understand what he was on about...
Title: Re: What was before the big bang?
Post by: timey on 23/09/2015 20:14:21
Quote from: Timey
I do not understand your visualisation of time Bill.

#199 contains only the briefest outline of nearly 60 years of thinking and talking about infinity, nothing and time.  The baldest essentials are that the cosmos (=everything that could ever be) is infinite and unchanging.  Our Universe is “part” of this (I would need to go into more detail than is appropriate here to explain the use of “part”). 

Quote from: Timey
Time is a phenomenon.  I would even go so far as to say time is a force.  It does something.  Without time there can be no motion

Time is a measure of change; motion is an aspect of change.  Change is an “illusion” that lets us make sense of our Universe.

Quote from: Timey
Now does anyone at-all want to talk about how time, space and a Casimir fluctuation 'might' arise from a zero geometry, zero content, point?  Because for me, this is the most pertinent part of the discussion!

I hope someone comes in on this, I think it would be a fascinating discussion, but I can’t help with it because the idea of something emerging from nothing is a non-starter for me.  It’s a bit like trying to answer the question: “How would I feel about this if I didn’t exist?”

Yes, a discussion concerning the emergence of time, space and a quantum fluctuation in "slow' time would be nice, but unlikely!  I'm not sure 'anyone' truly understands the concept - that gravitational time dilation, as I am describing it, would make an enormous difference to quantum mechanics!!!

I do not understand why you would think, that how you or anyone else 'feels' about a situation would have any bearing upon the physics of that situation.  This is like saying "A tree does not fall unless someone is there to witness it"

Without the concept of everything, nothing cannot exist.  Without the concept of nothing, the concept of everything cannot exist.  The two are inseparable, therefore nothing does exist, but only as a something that everything has been subtracted from.  Like I 'keep' saying, nothing 'is' something!  Logically speaking, this affords the concept that everything can then be added back, and that there will be 'something' about nothing that can initiate a 'physical' reaction of quantum mechanics that 'will' initiate time, geometrical space and a quantum fluctuation to occur from 'this' nothing.  This being a 'potential' possibility in a much 'slower' occurring rate of time.

My ideas concerning time are born from reading Lee Smolin's book "The Trouble with Physics", and I have been formulating this idea for 5 years.  I've given it a lot of thought!  Lee actually presents 'everyone's' ideas including Julian Barbour's... He covers Big Bang, multiverse, eternal cosmos, cyclic universe, steady state, standard model, quantum, quantum gravity quest, GR, SR, Maxwell's equations, redshift, conservation law, second law, how we perceive time, etc, etc.

He explains in great detail 'exactly' which of these concepts are proven and where they are not.  He covers where these concepts fall short of describing our universe as a 'complete' system, why it has been impossible to unify the most relevant and observably consistent theories with each other, and what physicists have been doing and thinking about with regards to furthering the quest for unification of these concepts.

My theory of time solves 'all' of these problems.  However, I now feel it unlikely that anyone 'here' actually wishes to consider the universe experimentally under this premiss.  Although, to say so, you all seem quite happy to consider the universe under much more complicated, less viable, and less testable premiss, that require additional qualities that are 'not' observed... so I will now leave you to it... In any case, it's been great meeting everyone. Thanks for the conversation.  All the best.
Vikki
Title: Re: What was before the big bang?
Post by: Bill S on 23/09/2015 21:38:56
Quote from: Jeffrey
These connections would then explain entanglement but not the so called spooky action at a distance or quantum teleportation

    Can the infinite cosmos idea explain various features if quantum weirdness?  It is certainly an area worth exploring, because, although many aspects of the idea appear distinctly speculative, if it could even begin to shed light into dark corners it could have some value.

    First we should identify some of these weird features. 

    1. The double-slit experiment in which a single, unobserved, quantum particle (quon, sensu Herbert) appears to be able to pass through both slits at the same time; and, through repetition, can most certainly give rise to an interference pattern which must indicate that each quon interferes with itself.

    2.  Quantum entanglement.  How can quons remain entangled even if they are on opposite sides of the Universe?

    3.  Quantum teleportation and other “spooky action at a distance”.

In order to see how an infinite cosmos might help with these aspects of QM it will be necessary to accept that in infinity there is no time, and no change, and that our seemingly finite, changing Universe is “part” of this.  Without more background, I acknowledge that this is something of a call to faith, but the reasoning that brought me to this point is available should anyone be interested.

In a timeless, changeless, infinite realm everything that can be “is”.  Although our Universe is “embedded” in this, we are able to perceive only a shadow in which time and change are features of our perceived reality.  So, what happens in the double-slit experiment?  The major problem here is that we have only the language of time and space to try to explain it.  Try to think, not of events happening in sequence, but rather of everything being in a static state.  Even the “snapshots” of Barbour’s timeless realm will not do.  The quon is at the emitter, at both slits, at the screen and anywhere else it could possibly be, statically and eternally.  Even this is an oversimplification, because (again, more background needed) in this infinite realm everything is the entirety of the realm. 

The last point, above, leads us into a consideration of quantum entanglement.  The universality to which I refer might be considered as the “conclusion” of David Bohm’s reasoning about the “implicate order”.  The quon at one side of the Universe is the quon at the other side of the Universe, and is everything in between.  The entanglement we perceive equates to an interpretation of Bohm’s “explicate order”.  It is the 3+1 dimensional shadow of the underlying infinite reality.

Viewed in this context, spooky action at a distance is no longer either “spooky” nor is it “action at a distance”. Quantum mechanics is a window into the infinite, through which we are just learning to look.  A measurement is simply the translation of infinite quantum reality, which we cannot see, into our limited perception of reality.  Is it surprising that it looks weird?

How does this idea affect our world and our physics?  It doesn’t, any more than working out, or deciding on, some particular interpretation of QM would make any difference to the technological wonders that can be achieved using it.  The most recent discoveries in particle physics are likely to have more influence on our world and our science; but, surely, anything that involves even a small step towards understanding is worth considering. 
 
Title: Re: What was before the big bang?
Post by: Bill S on 23/09/2015 21:44:55
Quote from: Timey
so I will now leave you to it

I'm sorry you feel you have to leave.  Hopefully that would be only this thread?

There are at least a couple of things in your last post I want to come back to when I have a few moments, so please don't go too far away.  [:)]
Title: Re: What was before the big bang?
Post by: jeffreyH on 24/09/2015 00:27:32
If every particle retains some aspect of every entanglement it has undergone that would explain some things. However it still does not explain the detection of spin in one particle determining the spin of its entangled partner. Even time reversal doesn't make it better as it is distance and not time that makes the difference. An even weirder thing is that the particles may have experienced different rates of time on their separate journeys so simultaneity cannot be maintained due to relativistic effects. The measurements may violate causality. Cause may actually follow effect under some special conditions. If we view the universe with no time then we have to eliminate distance too in order to resolve this. So that all particles are actually very near to each other and our perception of dimensions is the illusion. Of course it may well be something else entirely.
Title: Re: What was before the big bang?
Post by: Bill S on 24/09/2015 14:23:56
Quote from: Timey
Yes, a discussion concerning the emergence of time, space and a quantum fluctuation in "slow' time would be nice, but unlikely!  I'm not sure 'anyone' truly understands the concept - that gravitational time dilation, as I am describing it, would make an enormous difference to quantum mechanics!!!

I seem always to be rushing, so I might have missed something.  I have a particular interest in time, so would be glad to know more about your “slow time”.

Quote from: Timey
I do not understand why you would think, that how you or anyone else 'feels' about a situation would have any bearing upon the physics of that situation.

I didn’t make my point clear here.  It was not about feelings; I was drawing a comparison between saying: “How can  nothing be something” and “How could I do anything if I didn’t exist”.

Quote from: Timey
 
Without the concept of everything, nothing cannot exist.  Without the concept of nothing, the concept of everything cannot exist.  The two are inseparable, therefore nothing does exist, but only as a something that everything has been subtracted from.  Like I 'keep' saying, nothing 'is' something!  Logically speaking, this affords the concept that everything can then be added back, and that there will be 'something' about nothing that can initiate a 'physical' reaction of quantum mechanics that 'will' initiate time, geometrical space and a quantum fluctuation to occur from 'this' nothing.  This being a 'potential' possibility in a much 'slower' occurring rate of time.

For the sake of discussion, could we agree to differ on the something/nothing terminology?  What I would really like to understand is what difference the slowing of time would make to the scenario.

It’s a long time since I read "The Trouble with Physics".  Goodness knows if/when I would find time to read it again. A few tips as to where to look for the bit that triggered your thoughts about time would be much appreciated.
Title: Re: What was before the big bang?
Post by: Bill S on 24/09/2015 15:06:27
Quote from: Jeffrey
If every particle retains some aspect of every entanglement it has undergone that would explain some things. However it still does not explain the detection of spin in one particle determining the spin of its entangled partner. Even time reversal doesn't make it better as it is distance and not time that makes the difference. An even weirder thing is that the particles may have experienced different rates of time on their separate journeys so simultaneity cannot be maintained due to relativistic effects. The measurements may violate causality. Cause may actually follow effect under some special conditions. If we view the universe with no time then we have to eliminate distance too in order to resolve this. So that all particles are actually very near to each other and our perception of dimensions is the illusion. Of course it may well be something else entirely.

Of course, these questions have to be addressed; we do our physics in this Universe where linier time,  separation in space and causality are very relevant. It could not be otherwise.  However, none of these factors would apply in an infinite cosmos where everything just “is”, and where every “part” is the whole.   The distinction between the two might be compared to the distinction between understanding why QM is as it is and being able to use it on a practical level.
Title: Re: What was before the big bang?
Post by: timey on 24/09/2015 20:56:05
Quote from: Timey
Yes, a discussion concerning the emergence of time, space and a quantum fluctuation in "slow' time would be nice, but unlikely!  I'm not sure 'anyone' truly understands the concept - that gravitational time dilation, as I am describing it, would make an enormous difference to quantum mechanics!!!

I seem always to be rushing, so I might have missed something.  I have a particular interest in time, so would be glad to know more about your “slow time”.

Ok, Bill... I will explain again.

We think of gravitational time dilation as is described by General Relativity.
I am suggesting, on the basis of the non-unification of quantum mechanics with gravity, that this time dilation that GR describes is only a time dilation 'effect' due to a mass near mass phenomenon.  I am suggesting that GR is 'not' describing 'locational' gravitational time dilation in its considerations as is currently thought.

I am suggesting that 'locational' gravitational time dilation, as opposed to the mass near mass time dilation effect that GR describes, is as widely variant in the rate time occurs at, as a gravity field is in strength.  I am suggesting that time is stopped in a 0 gravity field and that as a gravity field strengthens, the rate time occurs at increases as per the strength of the gravity field. (I could now get into a complicated discussion concerning GR and relativistic mass in relation to gravity potential and kinetic energy, but let's just leave it at that for now, and you accept these parameters experimentally as the premiss)

Clearly anything with mass is going to be subject to this mass near mass phenomenon of time dilation that GR describes.  Light has no mass, so the effects of my 'locational gravitational time dilation will be most obviously observed where light is concerned.  (Ref: Pound Rebka experiment.) I will get back to the business of light and redshift later.

Clearly, if my 'locational' gravitational  time dilation is set at time stopped in a 0 gravity field, then... IF you would 'please' accept the premiss of my nothing that is something???  We can see that without a gravity field present, that time is now stopped.  IF we can find the 'energy' in this nothing that is 'something' to produce a Casimir fluctuation, and this fluctuation carries mass, then we can say that the 'reaction' between the energy that allows for the fluctuation and the mass of the fluctuation, and the resulting gravity field in relation to the mass of the fluctuation, that these parameters started time.  That these elements 'the fluctuation', 'the mass/gravity field' and 'time/motion emerged together, causing a geometric point to expand into geometric space.

What you have here is a creation moment in the microscopic, in 'extremely' slow occurring time.  Now, in order that you not be confused.  This does not mean that events are occurring in slow motion as relative to events occurring in the rate of time that we experience.  But, having said this, a Casimir fluctuation will exist for longer in a slower rate of time than it does in our rate of time. (There is a balancing ratio associated with SR that regulates this speed of events effect)

Virtual particles can arise from Casimir fluctuations as well as particle scattering.  In this slow time a virtual particle will exist for longer.  A virtual particle that exists for longer will take on more of the characteristics of a real particle.  I am suggesting that some of these virtual particles 'make it' and 'become' real particles.

Ok so we have a creation moment and a few real particles, what next?  How can that progress into what we see today? ... Ok, well clearly IF virtual particles can become 'real' in an almost 0 gravity field of a much slower rate of time, then we can say that more particles are produced at the edges/weaker gravity of the gravity field produced by the few particles of our microscopic universe, which are already attracting each other gravitationally into motion and clumping.  However... If we continue 'this' path of logic as is, what we will end up with is a bigger clump of matter in the middle of smaller clumps of matter.  We do not observe this to be the case in our universe.  Everything is relatively evenly spread and evenly clumped.

So... Taking the path of logic into the cyclic universe idea...  We have a microscopic universe with a few real particles that are clumping together. The spacial dimensions of this microscopic universe are dominated by the gravity field, in that where the gravity field runs to 0, time stops.  Where these few particles have now clumped together time runs a bit faster... and in between the clumped mass and the 0 gravity field - time runs marginally slower and slower as it approaches the 0 gravity field.  In this universe of such small spacial dimensions, the few particles of this microscopic universe clumping will cause gravitational reactions on scale with our universe and these clumped particles will form a black hole/black holes that merge into one black hole which jets out all of the particles, into a sea of particles. This has formed the beginning of the next cycle.  The scattering of particles has initiated more virtual particles to become real in the 'slower' time of this sea of particles.  The particles start clumping together.  Where particles have clumped, time runs quicker.  This follows the pattern until black holes are formed, that merge together into one, to jet out all the particles to form the beginning of the next cycle.  Each cycle becoming bigger in amount of particles, spacial size and duration of cycle.

IF you can accept the above as a premiss, then it becomes necessary to look at how we might 'view' events occurring within a rate of time that is occurring at extremely different rates to that of our own rate of time.  This requires looking at phenomenon that are subject to different strength of gravity fields.

First let's look at the world of quantum.  We have already accepted the premiss of particles emerging in extremely slow time in relation to an extremely weak gravity field.  What would be the status of a particles gravity field in relation to a mass near mass phenomenon?  Would it experience a rate of time as experienced by the mass of earth and measured by a cessium atom in relation to the motions of our solar system, as in how we measure all events?  Or will it be operating its existence within the bounds of a rate of time dictated by 'its' mass and 'gravity field' in relation to the greater mass of earth?

Looking at Planck's h constant.  Planck used a time measurement to derive this constant.  In his quest he noticed that there are frequencies that seem to be 'disallowed'.  Quanta do not operate on these frequencies, hence the gaps between the quantum leaps.  I am suggesting that it would be impossible for us to view all of the events of the world of quanta, IF the world of quanta were operating within a different rate of time.  If, say for instance, the gaps between the quantum leaps were to constitute 20% of the entirety of the range, then we would only be viewing 80% of the quanta's time scale, and that the quanta's rate of time would be occurring 20% slower, (faster would also apply) than our own rate of time.   What I am suggesting 'could' bring the world of quantum mechanics out from behind the uncertainty principle and more in line with classical physics.  No more spooky action, no more probability.

Moving to the other end of the scale... Black holes.  Hawking's temperature quandary has truly puzzled physicists, because as a black hole gets bigger, the temperature drops inversely proportional to the increase in its mass.  This is the direct opposite of what we experience in our scale of the universe concerning the addition of energy/mass normally being associated by an increase in temperature cubed proportionally to the mass of any addition.   Under the premiss of time set stopped in a 0 gravity field, the rate of time for the black hole occurs very fast.   We cannot see much of the events of a black hole.  Where time gets so fast at the event horizon, not even a glimmer of light can be viewed of its  time in relation to the rate of ours.  We do detect a temperature though!  As the black hole takes on more mass/gets bigger, it's rate of time runs faster, and as a result we view less of its temperature.  The black holes temperature is in fact rising with its intake of extra mass/energy, but its gravity field is also gaining in strength, causing time to occur faster, causing us to view a lesser percentage of its heat.  (A clever mathematician 'could ' work out by how much a gravity field increases the rate of time by calculating these parameters against each other!)

Now we come to redshift...  Time in the spaces between clumped mass will be running much slower than it will for the clumped mass.   This means that light is travelling through areas of slow time.  It will take the light longer to travel these distances of slow time because of distance in time, and not due to distance in geometry.  It takes light longer to cross the same distance in slower time.  Consequently, we can see that everything is not as far apart as we currently believe.

By making an addition to the equivalence principle stating that the speed of light is only constant to the ratio of the length of a second, we can see that in the case of the gravity field associated with the singular black hole that spells the end of a universes cycle and the beginning of the new, that we will have an escalated (vastly escalated in relation to a universe as big as our current one) speed of light, which when plugged into the equation, e=mc2, should produce enough energy to account for the inflationary period (black holes jets) of every new cycle.

This notion adheres to the conservation law and to the second law.   It gives explanation for star displacement during eclipse.  It gives reason for bullet cluster observations.  It gives reason for galaxy spiral.  It solves long term physics conundrums and presents the universe as a complete and working system, giving cause and viable conditions for the beginning and end of each cycle of the universe.

This theory requires no unobserved additional qualities for the system to work.

This theory can be very simply tested by placing two identical precision atomic clocks at locations that are of the exact same elevation, but of know significant difference in density.  See which one runs faster.
Title: Re: What was before the big bang?
Post by: Bill S on 24/09/2015 22:10:17
Thanks Timey, I’m impressed!  I’ve read that post once, but it deserves more, so I’m going to return to it.  You must have devoted a lot of time and effort to this, and have obviously looked at the problem from various perspectives.  That, I believe, deserves recognition, whether one agrees with everything, or not.

You know that my position is that nothing cannot be something, but I promise to do my best to put that on hold while I look more closely at your ideas.   

I’m glad you’ve not vanished.  BTW, is it OK to call you Vikki, or do you prefer to stick with Timey?
Title: Re: What was before the big bang?
Post by: jeffreyH on 25/09/2015 19:27:11
OK then what you are saying is that you have a coordinate space that proper space is mapped on to. Then in the voids the proper space swaps in magnitude with the coordinate space. Correct me if I am wrong here. So that at some cutoff energy gravity no longer contains the expansion of proper space. Since proper space is expanding time starts to slow.
Title: Re: What was before the big bang?
Post by: timey on 25/09/2015 19:36:35
Thanks Timey, I’m impressed!  I’ve read that post once, but it deserves more, so I’m going to return to it.  You must have devoted a lot of time and effort to this, and have obviously looked at the problem from various perspectives.  That, I believe, deserves recognition, whether one agrees with everything, or not.

You know that my position is that nothing cannot be something, but I promise to do my best to put that on hold while I look more closely at your ideas.   

I’m glad you’ve not vanished.  BTW, is it OK to call you Vikki, or do you prefer to stick with Timey?

Thanks Bill...!!!  For better or worse, yes, I have spent a lot of time thinking about this idea.  It's just another perspective, I'd be one very lucky ducky if I am actually right in any of it though.  All I can say is that it 'is' based in logical thought process and 'should' be mathematically calculable.  Just can't do the maths myself and this is the reason I am here on the forum.  Trying to find someone interested in helping me. :D

I know you are unconvinced by a nothing that is a something.  I'm grasping at straws here myself tbh Bill.  All I can say is that it seems to me a more viable prospect than trying to bring forth a Big Bang out of a who knows what...  But I think that a 'proper' particle physicist may well perhaps find some value in the idea of calculating quantum with a variable time aspect.

By all means call me Vikki.  The only reason I signed on as timey is due to this forum not having accepting any part of my real name when I registered. This probably being due to my email address of my name having been repeatedly hacked and all of my contacts list harassed.  I did, in my early days of having this idea, back when I really couldn't explain it in any kind of terms understandable to anyone but me, email quite a few prominent physicists my idea....lol!  This followed by a series of hacked emails, my name has no doubt been thoroughly marked by all and sundry as 'spam'... :)
Title: Re: What was before the big bang?
Post by: timey on 25/09/2015 20:45:32
OK then what you are saying is that you have a coordinate space that proper space is mapped on to. Then in the voids the proper space swaps in magnitude with the coordinate space. Correct me if I am wrong here. So that at some cutoff energy gravity no longer contains the expansion of proper space. Since proper space is expanding time starts to slow.

No.  I'm suggesting that the universe is not expanding.  That 'space' is not expanding.  That redshift is due to light travelling through slower time.  It takes the light longer and longer to travel the same lengths of distance.

The only expansion of space that happens in my model is the initial inflation period, this having been caused by the jets of the singular black hole that spells the end of the last universe.  All of the matter from within the last universe is contained in this singular black hole and it jets it all out in particle form as a sea.  A sea of particles that start clumping together.  Space is formed by particles vacating the area as they clump together.   GR predicts a mess of black holes.  In my model the universe is not expanding, or not like we currently think it is anyway. It's spacial dimensions actually reduce overall as matter further clumps.  The mess of black holes will not be far flung as per expanding universe and will merge into one, a singular black hole.

It is hard for me to describe exactly how the alterations to GR might work Jeff, because I do not truly understand the exact and precise ratio of balancing factors between elevation, kinetic energy, gravity potential and relativistic mass.  I don't do maths, remember?  All I can say is that these considerations may not be describing what time dilation is doing in the 'location' of interest, but could be describing a mass near mass phenomenon of mass in that 'location' in relation to earth.  That my idea of locational gravitational time dilation will be most significantly notable in the behaviour of massless light, and therefore I suspect that the rate that time occurs at will be linked to the strength of a gravity field, but that the changes in the rate of time, in respect to changes in the gravity field, will be to the tune of the metric.  The metric then being 'distance in time' not 'distance in geometry'.

I suspect that the space time matrix might still work, but that a time matrix would have to be added to the time aspect of this space time matrix in order for it to do so.

GR will not break down in a black hole under this premiss.
Title: Re: What was before the big bang?
Post by: puppypower on 25/09/2015 21:36:27
One thing that appears to left out of discussions of GR and black holes is gravity is more than GR, since gravity will also generate pressure which can cause phase changes of matter. For example, in the core of the sun, the pressure due to gravity, helps to induce fusion, which releases high frequency energy. As such, although the core of the sun is at the bottom of the space-time well and time runs slower due to GR, the fastest physical expressions of time also appear; gamma frequencies become the ticks of the local matter/energy clock. GR deals with space-time but pressure deals with the frequency of matter/energy; matter-time. 

If you compare space/distance to pressure, in the core of the sun the space of space-time is most contracted, while physical space between matter is also most contracted due to pressure. These go in the same direction. But time of space-time goes in the opposite direction of matter-time.

In a black hole, GR may be approaching the zero limit of space-time, while  the space due to pressure may be tiny, but matter-time is extremely fast. The matter clock is ticking as fast as it can go.
Title: Re: What was before the big bang?
Post by: Bill S on 25/09/2015 21:37:08
 I’ve been trying to get my head round what it would mean to slow time.  Would it be possible for time to slow without slowing the changes it was measuring?  Is time something that can change independently?

If I have two motors and run them both at 2,000 rpm; there would seem to be two ways in which I could double the rpm.

1. I could accelerate one motor to 4,000 rpm.

2. I could redefine time.  A second is defined as 9,192,631,770 (let’s call this 9.2x10^9) transitions of a caesium atom.  If, for the second motor only, I redefine a second as 1.84x10^10 transitions, I have both motors running at 4,000 rpm.  What have I achieved?   

OK, I can’t do that, but suppose the “gnomes” who make these decisions had defined a second as 1.84x10^10 transitions, or as 9,192,631,771 transitions; that would make a difference to how we use time for measuring change; but would it make any difference to time as such? 
 
Title: Re: What was before the big bang?
Post by: timey on 25/09/2015 22:36:59
Ok Bill

If you have 2 cars, both cars are travelling at 40 mph.  (This being mph earth hours.)  The first car is experiencing 'earth' time.  The second car is experiencing the slower rate of time.

Both cars travel for 10 miles.

The second car will feel like it has travelled a longer distance.
Title: Re: What was before the big bang?
Post by: Bill S on 25/09/2015 23:53:59
  Surely neither car will experience time as being anything other than “normal”.

Alice goes on a space flight during which she experiences one year.  When she is reunited with Bob, he has experienced ten years.  I.e. Alice has experienced slower time than has Bob.  If more things can happen in slower time, wouldn’t Alice  have consumed more supplies than Bob?
Title: Re: What was before the big bang?
Post by: jeffreyH on 26/09/2015 02:38:38
Then you are wrong timey.I have done the maths.

Consider this. Proper time can be given by:

05ea0cfe527a648de01b4418178ac3bc.gif where s is the distance of 1 metre. This is the proper time of free fall. If we calculate g at various radial distances from the surface of the earth we can plot this progression. The graph is attached. This is linear with an unchanging gradient. If you were right the plot would be a curve.
Title: Re: What was before the big bang?
Post by: timey on 26/09/2015 10:30:38
  Surely neither car will experience time as being anything other than “normal”.

Alice goes on a space flight during which she experiences one year.  When she is reunited with Bob, he has experienced ten years.  I.e. Alice has experienced slower time than has Bob.  If more things can happen in slower time, wouldn’t Alice  have consumed more supplies than Bob?

Yes, in the context of 'experiencing' time Bill, you are correct.  Both cars will experience the time passed as being the same from their own point of view. But the reality of how much 'distance' has been travelled will actually be different in the same time scale.

Both Alice and Bob go off on a space trip at 4000 mph, 'earth' hours.  Bob's time is occurring 10% slower than Alice's.  They both travel for 1 hour.

Alice has covered 4000 miles.
Bob has only covered 3600 miles.

(Clearly I am suggesting that time will get marginally slower and slower in weaker and weaker gravity fields, so this analogy is just to show 'a' slow time in relation to distance - and is not synonymous of how I'm suggesting slow time works in space.)
Title: Re: What was before the big bang?
Post by: timey on 26/09/2015 11:32:08
Then you are wrong timey.I have done the maths.

Consider this. Proper time can be given by:

05ea0cfe527a648de01b4418178ac3bc.gif where s is the distance of 1 metre. This is the proper time of free fall. If we calculate g at various radial distances from the surface of the earth we can plot this progression. The graph is attached. This is linear with an unchanging gradient. If you were right the plot would be a curve.

Jeff, I'm not sure that a description of proper time in free fall can apply.

The terminology 'free fall' indicates that we are measuring the time for 'mass' that is in free fall 'through' space.  This constitutes a mass near mass phenomenon.  All GR considerations will apply under these circumstances.

I am measuring what time is doing in the 'space' that mass is free falling through, not the 'time' for the 'mass' that is free falling through this 'space'.

Essentially this means that I am creating an observer independent reference frame for time that can be calculated in respect to the gravity field. (Hence the suggestion of a time matrix being added to the time aspect of the space time matrix)

In proper terms, both Alice and Bobs space craft will be subject to mass near mass phenomenon time dilation as they leave earth as per GR.  This effect will escalate up to a point, but as the craft move away from the influence of earths gravity field and travel into deep space, this minute escalation in the crafts rate of time will reverse and they will become subject to slower and slower rates of time.

Ok, so neither Alice or Bob realise that time goes slower and slower as one moves out into deep space.  They both have exactly 40 000 miles to travel and they are both travelling at 4000 mph 'earth hours'.  Neck and neck, its a race to the finish line. Bob experiences engine trouble and his craft is now only travelling at 2000 mph.

During the journey the rate of time in deep space decreases steadily up to the point of our imaginary ribbon finish line by 50%.  The equivalence principle now states that the speed of light is only constant to the ratio of the length of a second and mass cannot exceed the speed of light.  In fact, as Alice's speed of 4000 mph 'earth hours' moves into slower time, this speed is becoming a higher percentage in relation to the decreasing upper speed limit of the speed of light 'allowed' by this slower rate of time.  Alice's journey is now experiencing a greater degree of SR time slowing effects.

Bob was tempted to turn around and go home.  What chance did he have of winning the race now?  But being the good sport that he is and not wanting to leave Alice in a 1 horse race, he continues.  Maybe Alice might encounter engine troubles too!

Bob's speed of 2000 mph 'earth hours' does not amount to as much of a percentage of the upper speed limit afforded by the slower rates of time he is travelling through.  The SR effects are not time slowing his journey as much as they are time slowing Alice's.

Under this remit Jeff, can Bob get to the set finish line of distance 'before' Alice?
Title: Re: What was before the big bang?
Post by: alancalverd on 26/09/2015 13:09:22

Both Alice and Bob go off on a space trip at 4000 mph, 'earth' hours. 
4000 mph relative to what?
Quote
Bob's time is occurring 10% slower than Alice's.  They both travel for 1 hour.
measured on whose clock?


[/quote]
Title: Re: What was before the big bang?
Post by: timey on 26/09/2015 14:06:54

Both Alice and Bob go off on a space trip at 4000 mph, 'earth' hours. 
4000 mph relative to what?
Quote
Bob's time is occurring 10% slower than Alice's.  They both travel for 1 hour.
measured on whose clock?

Ok Alan.  On the basis that this particular consideration was just to give Bill a feel for slow time in relation to distance travelled and not depicting a suggested reality.

The answer to your questions are 4000 mph 'earth hours' relative to the distance travelled...

Bobs clock would be ticking slower than Alice's, measured relative to the strength of his locational gravity field, in relation to his own associated mass, and regulated by his speed of travel.

These questions are much more 'fun' when answered in relation to the considerations in Post 233, which I am suggesting do constitute a 'potential' reality.
Title: Re: What was before the big bang?
Post by: jeffreyH on 26/09/2015 16:32:26
That unchanging gradient will go all the way to infinity due to the inverse square nature of gravity. It is now up to YOU to show mathematically that this is wrong. No more word play.
Title: Re: What was before the big bang?
Post by: timey on 26/09/2015 18:33:56
Why would I need to prove that gradient to be wrong in order to prove that time stops in a zero gravity field?

Check list:
1) The gradient that you have calculated is for an object in free fall, am I right?
2) This gradient follows the inverse square law, right?
3) You have calculated this gradient according to a method associated with 'proper time' and this method is synonymous to GR, right?

If you can answer these questions, I still won't be able to do the maths to determine how the gradient of my locational gravitational time dilation is linked to the gradient of a gravity field that is subject to the inverse square law, but I think I 'could' make a very good attempt at explaining the method by which 'you' could do it Jeff. :D
Title: Re: What was before the big bang?
Post by: jeffreyH on 26/09/2015 18:46:14
I already know it is wrong. You are never free from the influence of gravity. What I did suggest is a cutoff energy which you objected to because it too went against you ideas.
Title: Re: What was before the big bang?
Post by: Bill S on 26/09/2015 20:15:43
Vikki, are you sure you are not a scientist?  "This is the answer to your question; whatever your question might have been" is a common response from scientists.  [:)]

The question was: 

Quote
If more things can happen in slower time, wouldn’t Alice  have consumed more supplies than Bob?

This is something I found quite helpful; even the equations were not too frightening. 

http://home.earthlink.net/~owl232/twinparadox.pdf)

 
Title: Re: What was before the big bang?
Post by: timey on 26/09/2015 20:41:33
I already know it is wrong. You are never free from the influence of gravity. What I did suggest is a cutoff energy which you objected to because it too went against you ideas.

Well actually Jeff, I had no idea what you meant by a cut off point.  If you could elaborate perhaps I might understand.  It had no relation to what I meant, (which is what you were asking about) or not as I understand it anyway... although I can accept that I haven't perhaps understood it, and therefore am open to doing so.

I do not understand why you would think that my ideas incorporate being free from the influence of gravity.  Time is time.  Gravity is gravity.  They are linked via gravitational time dilation.  We know this already.  I am just suggesting that how we think they are linked is perhaps wrong.  That this observation we make of time dilation is occurring for different reasons than we think.  That actual 'locational' time dilation is a much wider variant on a scale that echoes the strength of a gravity field, and is linked to the strength of a gravity field, but please realise that this locational gravitational time dilation does 'not' change the rate of time as per the inverse square law as gravity changes in strength as per radius.  This being very obviously 'not' a possibility when considering the behaviour of temperature in the black hole. (this being under the remit of time stopped in a 0 gravity field.)
Title: Re: What was before the big bang?
Post by: timey on 26/09/2015 20:48:05
Vikki, are you sure you are not a scientist?  "This is the answer to your question; whatever your question might have been" is a common response from scientists.  [:)]

The question was: 

Quote
If more things can happen in slower time, wouldn’t Alice  have consumed more supplies than Bob?

This is something I found quite helpful; even the equations were not too frightening. 

http://home.earthlink.net/~owl232/twinparadox.pdf)

Actually Bill, I didn't really think it an important factor to the discussion.  Everyone has talked the *rse off the twin paradox.  Why would we wish to go down that road?

The pertinent part of this discussion, regarding slower time due to locational gravitational time dilation, lies in the factor of distance travelled!

If you travel the same distance in slower time it will take you longer.  If you didn't know you were travelling in slower time you would think you had travelled a longer distance.
Title: Re: What was before the big bang?
Post by: Bill S on 27/09/2015 00:05:43
Quote from: Vikki
Actually Bill, I didn't really think it an important factor to the discussion.  Everyone has talked the *rse off the twin paradox.  Why would we wish to go down that road?

Perhaps because it is relevant to whether or not more things can happen if time is slower. 
I carefully avoided suggesting that Alice and Bob were twins.  [:)]

Quote from: Vikki
The pertinent part of this discussion, regarding slower time due to locational gravitational time dilation, lies in the factor of distance travelled!
Is that distance travelled per unit time?

Quote from: Vikki
I am suggesting that 'locational' gravitational time dilation, as opposed to the mass near mass time dilation effect that GR describes, is as widely variant in the rate time occurs at, as a gravity field is in strength.

I’m still not clear on the distinction between “locational gravity” and “mass near mass gravity”.


Quote from: Vikki
If you travel the same distance in slower time it will take you longer.

Now you've really lost me.  I thought you could achieve more in slower time.   
Title: Re: What was before the big bang?
Post by: jeffreyH on 27/09/2015 00:14:29
I already know it is wrong. You are never free from the influence of gravity. What I did suggest is a cutoff energy which you objected to because it too went against you ideas.

Well actually Jeff, I had no idea what you meant by a cut off point.  If you could elaborate perhaps I might understand.  It had no relation to what I meant, (which is what you were asking about) or not as I understand it anyway... although I can accept that I haven't perhaps understood it, and therefore am open to doing so.

I do not understand why you would think that my ideas incorporate being free from the influence of gravity.  Time is time.  Gravity is gravity.  They are linked via gravitational time dilation.  We know this already.  I am just suggesting that how we think they are linked is perhaps wrong.  That this observation we make of time dilation is occurring for different reasons than we think.  That actual 'locational' time dilation is a much wider variant on a scale that echoes the strength of a gravity field, and is linked to the strength of a gravity field, but please realise that this locational gravitational time dilation does 'not' change the rate of time as per the inverse square law as gravity changes in strength as per radius.  This being very obviously 'not' a possibility when considering the behaviour of temperature in the black hole. (this being under the remit of time stopped in a 0 gravity field.)

That is just a lot of words put together in a misleading fashion to confuse the issue. When describing theories science should be precise and unambiguous. Everyone should be able to read and understand what is meant. You dodge around the issues. You need to learn physics. By that I mean the mathematics as well as the vague concepts that you seem to have acquired. I am quite fond of puzzles. Science is full of them. They usually arise from unexpected results of observations. They are not just conjured up in the minds of scientists. Explaining them then leads to new theories. You are trying to rewrite physics to suit your ideas. Not a good idea if you want to be taken seriously.
Title: Re: What was before the big bang?
Post by: timey on 27/09/2015 00:51:26
Quote from: Vikki
Actually Bill, I didn't really think it an important factor to the discussion.  Everyone has talked the *rse off the twin paradox.  Why would we wish to go down that road?

Perhaps because it is relevant to whether or not more things can happen if time is slower. 
I carefully avoided suggesting that Alice and Bob were twins.  [:)]

Of course more things can happen if time goes slower.  The twin paradox is more concerning SR effects of slowing due to speed of travel. Edit: humour noted :)

Quote from: Vikki
The pertinent part of this discussion, regarding slower time due to locational gravitational time dilation, lies in the factor of distance travelled!
Is that distance travelled per unit time?

Overall distance travelled, per speed, per unit time.

I’m still not clear on the distinction between “locational gravity” and “mass near mass gravity”.

Imagine a clock placed in a location at a radius in elevation to earth.  That clock will tell you what time is doing in relation to that clock.  Now take the clock out of the equation and tell me what time dilation is doing in the location that the clock was in.

I am suggesting that the time dilation registered by the clock in that location is caused by the clock and it's related mass in relation to earth's mass.
I am suggesting that if we take away the clock and it's related mass from the location, the time dilation at this location will be subject to the gravity field of that radius.
I am suggesting that locational gravitational time dilation is set with time stopped in a 0 gravity field.

Quote from: Vikki
If you travel the same distance in slower time it will take you longer.

Now you've really lost me.  I thought you could achieve more in slower time.

It's a simple equation.  Mph.  If the hour is a longer hour, it will take longer to travel a mile.  We could drop the speed factor in relation to the equation to the same effect.  A slower speed of motion or a slower speed of hour will result in it taking longer to travel a mile.

Now then.  If you didn't know you were travelling through slower time, and you have observed that your speed has remained constant, you will then believe that you have travelled a further distance than a mile.
Title: Re: What was before the big bang?
Post by: timey on 27/09/2015 01:02:41
I already know it is wrong. You are never free from the influence of gravity. What I did suggest is a cutoff energy which you objected to because it too went against you ideas.

Well actually Jeff, I had no idea what you meant by a cut off point.  If you could elaborate perhaps I might understand.  It had no relation to what I meant, (which is what you were asking about) or not as I understand it anyway... although I can accept that I haven't perhaps understood it, and therefore am open to doing so.

I do not understand why you would think that my ideas incorporate being free from the influence of gravity.  Time is time.  Gravity is gravity.  They are linked via gravitational time dilation.  We know this already.  I am just suggesting that how we think they are linked is perhaps wrong.  That this observation we make of time dilation is occurring for different reasons than we think.  That actual 'locational' time dilation is a much wider variant on a scale that echoes the strength of a gravity field, and is linked to the strength of a gravity field, but please realise that this locational gravitational time dilation does 'not' change the rate of time as per the inverse square law as gravity changes in strength as per radius.  This being very obviously 'not' a possibility when considering the behaviour of temperature in the black hole. (this being under the remit of time stopped in a 0 gravity field.)

That is just a lot of words put together in a misleading fashion to confuse the issue. When describing theories science should be precise and unambiguous. Everyone should be able to read and understand what is meant. You dodge around the issues. You need to learn physics. By that I mean the mathematics as well as the vague concepts that you seem to have acquired. I am quite fond of puzzles. Science is full of them. They usually arise from unexpected results of observations. They are not just conjured up in the minds of scientists. Explaining them then leads to new theories. You are trying to rewrite physics to suit your ideas. Not a good idea if you want to be taken seriously.

LOL!!!
Title: Re: What was before the big bang?
Post by: jeffreyH on 27/09/2015 02:11:07
Concise response and summing up your viewpoint admirably.
Title: Re: What was before the big bang?
Post by: timey on 27/09/2015 02:22:15
...and there speaks the man who was telling lightarrow only 4 or so weeks ago how he had just come to grips with the double light slit experiment and was moving on to complex conjugates, was it?  My photographic memory fails me slightly.

Oh, yes... and I nearly fell off my chair!

Can I think of one time when you have even attempted to answer 'any' question I have posed to you in response to your responses to my posts.

No... no I can't!  I rest my case...  Lol indeed!
Title: Re: What was before the big bang?
Post by: jeffreyH on 27/09/2015 02:46:10
Do you even know what a complex conjugate is?
Title: Re: What was before the big bang?
Post by: mathew_orman on 27/09/2015 09:34:10
Local collapse of planets is a natural phenomena and happens continuously... At some point resulting sun explodes due to extreme gravity...
Title: Re: What was before the big bang?
Post by: timey on 27/09/2015 11:07:01
Do you even know what a complex conjugate is?

It surprises me Jeff, that in this day and age you would ask anyone who can clearly read, and who is obviously connected to the internet, a question consisting of the words "do you even know what a ? ...is?"

https://en.m.wikipedia.org/wiki/Complex_conjugate

http://www.mathwords.com/c/complex_conjugate.htm

http://www.mathcentre.ac.uk/resources/sigma%20complex%20number%20leaflets/sigma-complex6-2009-1.pdf

Need I continue?

Now Jeff, do you even know what this post is?  I'll give you a clue.... Common sense?  Or sarcasm?
Title: Re: What was before the big bang?
Post by: jeffreyH on 27/09/2015 16:32:10
So I attack your theory and you attack me personally. It says a lot about you. If you could defend your theory I suppose you would. Wouldn't you?
Title: Re: What was before the big bang?
Post by: timey on 27/09/2015 17:15:05
I wasn't aware that you had attacked my theory.  You made some calculations concerning free fall and pronounced me wrong about an idea that cannot be calculated by proper time in free fall.   I mention this fact and explain why, you tell me to go learn physics and then question my knowledge about an aspect of quantum mechanics, as if one has to be part of a secret society to know about such a concept, when in fact the info is readily available to anyone who can read and who is equipped with an internet connection.  I point this out to you, and now I am attacking you personally?

You 'are' a strange kettle of fish Jeff, to be sure!  ... It remains a mystery to me exactly where you are coming from, but really, if it's not informative... or funny... then it's just 'pants' in my opinion!
Title: Re: What was before the big bang?
Post by: unstman on 27/09/2015 17:32:11
I would say the question goes into the same category as ' I can prove there is another Universe once you survive going through a blackhole '  without actually having any evidence to substantiate such a statement. What was before the Big Bang? Anything your mind wants to put there, as it is as of no real relevance to what there is now, and, in all honesty, I do not think the question can be answered.
Title: Re: What was before the big bang?
Post by: jeffreyH on 27/09/2015 18:09:37
If you are not at infinity, which is impossible in itself, then you are always in freefall with respect to a mass. It may be of an infinitesimally small magnitude but it is still there. Why don't you get this? You are trying to reverse the accepted view of time dilation AND saying expansion of the universe is wrong. Show the evidence instead of assuming a passive-aggressive attitude.
Title: Re: What was before the big bang?
Post by: timey on 27/09/2015 19:42:57
What part of observer independent do you miss-understand Jeff?

I want to measure the time dilation that (I am suggesting ) is occurring in the 'space' that one is free falling through in respect to mass.

When you understand this, then you will understand that the calculation that you put forward cannot apply as a measurement of what is occurring in the locational space that one is falling through, and is only describing what is occurring for oneself, free falling with respect to mass.

I am not here to 'prove' anything Jeff.  As I keep stating, I am here seeking mathematical help regarding my idea... An idea that I am perfectly well aware goes against the grain of accepted physics.  However, the idea adds up to the same observable universe, doing the same observed and proven physical things, only without introducing any additional unobserved qualities into the equation and giving a viable means of a cyclic universe that finds its energy for the initiation and end of each cycle within the system.  On this basis alone, this idea is worthy of consideration and can be tested in physical terms very simply, as I have indicated previously.

At no point have I stated that my theory is the 'correct theory', only that my theory is an interesting idea, which it is.  I only become bolshy with you when you speak to me as if I'm stupid Jeff.  To say so, I really don't understand what your problem is!
Title: Re: What was before the big bang?
Post by: Bill S on 27/09/2015 21:51:01
Is the idea of free fall causing a problem?

Richard Wolfson uses the term free float rather than free fall, on the basis that although it is easy to identify “free fall” as falling in the case of a parachutist dropping from a plane, in that time before the parachute opens.  It is less easy to see it as falling in the case, for example, of a satellite orbiting the Earth, or the Earth orbiting the sun, and particularly difficult in the case of the moon.  (BTW: “supermoon" + eclipse tonight)

  It is equally difficult to identify the motion of stars around the centre of a galaxy, or the motion of the Universe as a whole, in the event that it should be rotating, as falling.  Wolfson uses the term “free-float”, which has the same meaning as free fall, but creates a more easily visualised image, especially on a large scale. 
Title: Re: What was before the big bang?
Post by: Bill S on 27/09/2015 22:05:34
Quote from: Vikki
I want to measure the time dilation that (I am suggesting) is occurring in the 'space' that one is free falling through in respect to mass.

Let’s see if I’m getting anywhere near grasping this. 

There is a mass “A” free falling through space.
You don’t want to measure the time dilation in the F of R of “A”.
You want to measure the time dilation in the surrounding space.
When you say “in respect to mass”, do you mean “A”, or some more generalised mass?
Title: Re: What was before the big bang?
Post by: timey on 27/09/2015 22:21:41
Is the idea of free fall causing a problem?

Lol Bill :) Erm, nope.  I just don't see empty space as being 'in' free fall, is all.

As I'm trying, when looking at the idea of slower time in space, to measure what the 'potential' time dilation for empty space, as per that empty spaces gravity field, is, I don't think that a measurement of a time scale for an object in free fall is going to be applicable.

The idea of free fall itself is entirely unproblematic.
Title: Re: What was before the big bang?
Post by: Bill S on 27/09/2015 22:29:04
Quote from: Unstman
What was before the Big Bang? Anything your mind wants to put there, as it is as of no real relevance to what there is now, and, in all honesty, I do not think the question can be answered.

I think there are two questions that frequently become confused:
1. What was before the Big Bang?  I think you have the right answer to that.
2. Was there anything before the Big Bang?  This would seem to be very relevant to what there is now, because if the answer were “no”, there would be nothing now, which is not exactly the situation.
Title: Re: What was before the big bang?
Post by: unstman on 27/09/2015 23:24:25
I asked the question ' If Dark Matter/Energy makes up 96% of what the Universe is, is it possible that Dark Matter/Energy existed before the Big Bang, and it was Dark Energy/Matter which acted as a catalyst for the Big Bang to take place in the first place?

It could also be, possibly, interpreted as the basis for multi-Universes to exist, but with different physical properties or even the same, but they never meet. I was watching an Horizon programme where the creation, if memory serves me well, of matter was the result of 2 Universes colliding for a brief moment in time?   
Title: Re: What was before the big bang?
Post by: jeffreyH on 27/09/2015 23:44:15
OK timey. You are also saying that galaxies are not as far away as we think. Correct me if I'm wrong. Therefore the universe is not as old as we think. Am I summing it up correctly? That is what this implies. So by how much are we out on the age of the universe? Currently the age of the universe is thought to lie somewhere between 13 and 14 billion years. I think you need to put a definite figure on this so that it can be checked mathematically. That shouldn't be difficult. Even just a guesstimate will do.
Title: Re: What was before the big bang?
Post by: Bill S on 27/09/2015 23:46:38
Quote from: Unstman
It could also be, possibly, interpreted as the basis for multi-Universes to exist, but with different physical properties or even the same, but they never meet. I was watching an Horizon programme where the creation, if memory serves me well, of matter was the result of 2 Universes colliding for a brief moment in time?

Horizon comes up with some interesting stuff, but slips easily into “fairytale physics”.  I suppose the idea of the Big Bang being the result of a collision between two universes is not so far removed from the idea of two branes colliding; but where do fairytales end and physics begin? 
Title: Re: What was before the big bang?
Post by: jeffreyH on 27/09/2015 23:48:21
You are also saying that time slows down but you travel further. So that as time slows velocity increases. So without any impetus vast distances can be crossed with ease. That is what you said. So we can have free energy from nowhere in deep space.
Title: Re: What was before the big bang?
Post by: jeffreyH on 28/09/2015 00:08:25
OK timey since you posted a link to complex conjugates then tell me what is significant about a number that is its own complex conjugate. Look it up if necessary.
Title: Re: What was before the big bang?
Post by: Bill S on 28/09/2015 00:46:19
You could be on to a loser there Jeffrey,  [:)]

As a non-mathematician the answer meant nothing to me - in fact it seemed self contradictory - but I found an answer in less than two mins.
Title: Re: What was before the big bang?
Post by: jeffreyH on 28/09/2015 00:51:00
Yes but timey might actually learn something mathematical.
Title: Re: What was before the big bang?
Post by: timey on 28/09/2015 01:21:38
OK timey. You are also saying that galaxies are not as far away as we think. Correct me if I'm wrong. Therefore the universe is not as old as we think. Am I summing it up correctly? That is what this implies. So by how much are we out on the age of the universe? Currently the age of the universe is thought to lie somewhere between 13 and 14 billion years. I think you need to put a definite figure on this so that it can be checked mathematically. That shouldn't be difficult. Even just a guesstimate will do.

If the universe started in slow time then the universe as measured by years in our quicker time, would be inadequately measured.  The universe would be a lot 'older' than we think.

To calculate by how much older the universe is it would be necessary to know by what ratio time increases in relation to an increasing gravity field.  This being the aspect of mathematics that I have come to this forum for help with.  Therefore your demanding some mathematical guesstimate of an age for the universe is both unfair and unrealistic.

You are also saying that time slows down but you travel further. So that as time slows velocity increases. So without any impetus vast distances can be crossed with ease. That is what you said. So we can have free energy from nowhere in deep space.

Nope.  If time is going slower, velocity does not change under these circumstances, if you didn't know you were travelling through slower time you would think that the distance travelled is longer when it isn't.
I'm saying that the slowing effects of SR time dilation with respect to rate of motion can be controlled via speed of motion when travelling through slower time.

OK timey since you posted a link to complex conjugates then tell me what is significant about a number that is its own complex conjugate. Look it up if necessary.

I really do 'not' appreciate your tone here ... I have repeatedly told you that I do not do maths.  My education ended abruptly at age 11, my maths ed finished at long division.  It's not like I just flunked it Jeff.  If I'd had any training in algebra, I've no doubt I'd be damn good at it.  I didn't, and so  I'm here for help!!!  (Not riddles thank you)  I don't know the answer to your question... However from reading the blurb, it does occur to me that 'nothing' in relation to 'everything' could almost be considered as a pair of complex conjugates!

Yes but timey might actually learn something mathematical.

Yes but... the maths timey wants to know about are specific and specified, and do not incorporate my answering you regarding non-related complex conjugate questions in order to prove myself to you... ;)
Title: Re: What was before the big bang?
Post by: Bill S on 28/09/2015 04:22:51
4.20am here. Just been watching the eclipse.  Worth staying up for.

Vikki, please don't let #257 slip away unanswered; I'm really trying to get my head round this.
Title: Re: What was before the big bang?
Post by: timey on 28/09/2015 10:43:46
Quote from: Vikki
I want to measure the time dilation that (I am suggesting) is occurring in the 'space' that one is free falling through in respect to mass.

Let’s see if I’m getting anywhere near grasping this. 

There is a mass “A” free falling through space.
You don’t want to measure the time dilation in the F of R of “A”.
You want to measure the time dilation in the surrounding space.
When you say “in respect to mass”, do you mean “A”, or some more generalised mass?

Jeff was talking about a measurement for free fall in relation to mass.  I assumed he meant a planetary body of mass.  I want to measure the time dilation for the space the object is falling through, in respect to the gravity field of the planetary body of mass that the object is falling towards, and not the time dilation measured 'for' the object that is in free fall.

Eclipse of the moon?  It was a very clear night last night where I am in UK, so it would have been well viewed from here, but it was after 4 in the morning when you posted.  My curiosity is aroused!  :)  Where in the world are 'you' Bill?
Title: Re: What was before the big bang?
Post by: Bill S on 28/09/2015 14:56:15
Quote from: Vikki
Where in the world are 'you' Bill?

UK. I'm a Cornishman, stuck in Essex.  [:(]
Title: Re: What was before the big bang?
Post by: Bill S on 28/09/2015 15:36:33
Quote from: Vikki
Jeff was talking about a measurement for free fall in relation to mass.  I assumed he meant a planetary body of mass.  I want to measure the time dilation for the space the object is falling through, in respect to the gravity field of the planetary body of mass that the object is falling towards, and not the time dilation measured 'for' the object that is in free fall.

If the free falling mass is a planet orbiting its star, it will be subject to time dilation due to its speed relative to the star.  You are not looking for that?

It will also be subject to time dilation due to the gravity of the star.  You are not looking for that?

The space through the planet is travelling will be subject to time dilation due to the gravity of the star.  This is what you are interested in?

So, is the question you are asking something like this: If a clock is placed in deep space where gravitational influences are minimal; and what forces there are will probably be balanced; how will that clock’s record of the rate of the passage of time compare with that of a clock placed in the gravitational field of the star which your planet is orbiting?
Title: Re: What was before the big bang?
Post by: timey on 28/09/2015 16:30:02
A Cornish man in Essex aye... You were up late!

We'll yes, clearly the rate of time in accordance with the gravity field for a planet, or star, and the planet in relation to the star, and the speed that both planet and star are travelling through space at, are of great interest... The fact that the rate of time for the space that the planet and star are moving into, and out of, will change as the gravity field moves... is also of interest.

But the question is... With time stopped in a 0 gravity field, how can we link the changes in the rate of time to the changes in the gravity field?  ... ie: By how much does the rate of time change per change in the strength of a gravity field under the remit of time stopped in a 0 gravity field?

I believe that the answer to this question lies in calculating the mass of a black hole against the temperature that we would normally expect to find in e=mc2.  We log by what percentage there is a difference, and take this as the percentage by which the black holes time is running faster.  We then look at the temperature that we observe of a black hole and how that temperature drops inversely proportional to additional mass.  We then calculate by how much of a percentage the temperature should be rising with additional mass in relation to by what percentage the black holes temperature appears to drop.   On the basis that one already has earths length of second and earths mass as parameters, it should be possible to work out by how much an increasing gravity field steadily increases the rate of time from these considerations.

As soon as you put a clock anywhere, it has associated mass.  If that mass is under the influence of a greater mass, it will experience a GR related time dilation.
Title: Re: What was before the big bang?
Post by: Bill S on 28/09/2015 19:41:05
Thanks for that Vikki.  One of the problems with a thread like this that progresses quite swiftly is that things can get left behind.  For this reason, I am going to go back to some points from your post #222 before looking at later stuff.

I need to look more closely at the idea that time stops in zero gravity.  Does this apply to any zero gravity situation?  For example: The gravity at the international space station is about 89% what it is on earth. However the special path the ISS takes makes conditions there almost zero gravity.  Is time 1% slower than on Earth, or is it almost at a standstill?
Title: Re: What was before the big bang?
Post by: timey on 28/09/2015 20:39:50
Ok Bill, the ISS is running under specific conditions, we'll get to that but first...

In a next to nothing gravity field you mention time being 1% slower.  This gives the wrong impression.  If all of the mass of the universe were in one clump, then we could say that time was running at 100% of that universes potential.  At 1% in a near 0 gravity field, time would be running 99% slower than its full potential.  Quite how the range of the rate of time spreads over the gravity fields in between is the question.

The ISS's rate of time is relative to the locational gravity field in respect to its own associated mass in relation to the mass of earth and regulated by the rate of its momentum.  It's zero gravity condition is relative to its situation of free fall, being as its momentum is at escape velocity, and in 'orbit' around the earth.
Title: Re: What was before the big bang?
Post by: dlorde on 28/09/2015 21:07:40
.. The gravity at the international space station is about 89% what it is on earth. However the special path the ISS takes makes conditions there almost zero gravity.
Objects in orbit aren't in zero gravity, they're in free fall - falling under the influence of non-zero gravity. The ISS crew are in the Earth's gravitational field, but don't feel that gravity because their surroundings (the ISS) are also falling at the same rate. The same applies to the occupants of a free-falling lift (elevator).
Title: Re: What was before the big bang?
Post by: Bill S on 28/09/2015 21:26:30
Quote from: Vikki
  It's zero gravity condition is relative to its situation of free fall, being as its momentum is at escape velocity, and in 'orbit' around the earth.

What is your understanding of "escape velocity".  Mine is that it is the speed and direction I would have to give to a projectile at the Earth's surface, if I wanted it to "escape" into space without any further power or expenditure of energy, after launch.   I can't equate this with your saying that the momentum of the ISS is at escape velocity.
Title: Re: What was before the big bang?
Post by: Bill S on 28/09/2015 21:30:37
Thanks Dlorde, that was, sort of, what I meant when I said "the special path the ISS takes makes conditions there almost zero gravity", but you post was much clearer.
Title: Re: What was before the big bang?
Post by: jeffreyH on 28/09/2015 21:49:37
An object cannot orbit at escape velocity. That is why it is called escape velocity.
Title: Re: What was before the big bang?
Post by: timey on 28/09/2015 22:18:34
Ok, ALERT , terminology problem!!!

An object that is in orbit has to remain at a certain velocity to remain in orbit.  That is the momentum I mean.  I understand that escape velocity consists of an equal of gravity potential and kinetic energy.

Back to the interesting bits now?
Title: Re: What was before the big bang?
Post by: jeffreyH on 28/09/2015 22:36:59
Ok, ALERT , terminology problem!!!

An object that is in orbit has to remain at a certain velocity to remain in orbit.  That is the momentum I mean.  I understand that escape velocity consists of an equal of gravity potential and kinetic energy.

Back to the interesting bits now?

OK. Say we have velocity v that a mass m is moving at. Then momentum is m times v. Just to clarify. To get kinetic energy the expression is (1/2) times m times v squared. That is (1/2)mv^2. The integral of momentum with respect to velocity is therefore kinetic energy. ead3aa6f1893b109c9a3382acdbce041.gif
Title: Re: What was before the big bang?
Post by: timey on 28/09/2015 22:57:04
Ok, I can follow that Jeff, but to understand its relevance I need you to put it into some kind of context.
Title: Re: What was before the big bang?
Post by: jeffreyH on 28/09/2015 23:02:37
Make sure there is not a wall in front of you if you have a lot of kinetic energy.
Title: Re: What was before the big bang?
Post by: timey on 28/09/2015 23:41:31
In context with regards to time dilation, or what came before the Big Bang
(You are most welcome to post down on 'On the Even Lighter Side' if you like ;). )
Title: Re: What was before the big bang?
Post by: jeffreyH on 28/09/2015 23:43:56
Well if you hit the wall there will definitely be a big bang.
Title: Re: What was before the big bang?
Post by: jeffreyH on 29/09/2015 00:16:45
Take kinetic energy. You could state it as ceada00a18afd3dee056485a9dd7913b.gif to get an increase in relativistic mass. However is this true of gravitation?

Proper time a83208d837add4e22657d3b695d473f5.gif

Proper distance 21cf75c0d7ee6f726bbf90fe6988d0fa.gif

So instantaneous velocity cbb60a3ae491c9139fd7456d1285ee42.gif

Then 6328c9c6620736856d32eef5150bf27b.gif

The kinetic energy will be 76b6251dd1d810313bde18dd5a3e43ca.gif

Since all particles experience the same effect then it may be unwise to apply relativistic mass in this case. Otherwise there would be infinite amounts of mass at the event horizon of a black hole.

Is that enough context?
Title: Re: What was before the big bang?
Post by: jeffreyH on 29/09/2015 00:42:09
This may also be useful.

http://www.schoolphysics.co.uk/age16-19/Mechanics/Gravitation/text/Kinetic_energy_in_orbit/index.html (http://www.schoolphysics.co.uk/age16-19/Mechanics/Gravitation/text/Kinetic_energy_in_orbit/index.html)
Title: Re: What was before the big bang?
Post by: timey on 29/09/2015 01:03:12
Si si señor, sin problemo,..

Orbit, escape velocity, relativistic mass... Not part of the point of the conversation, not sure why you think it relevant or why that link is helpful

Bored now!

Adios mi amigo...
Title: Re: What was before the big bang?
Post by: jeffreyH on 29/09/2015 01:06:28
Because ultimately there needs to be a repulsive force to initiate a big bang. That may well be gravity itself.
Title: Re: What was before the big bang?
Post by: timey on 29/09/2015 01:33:46
Cool, cool.  Now that you have made me aware that you have initiated a change of subject matter to the conversation, and are not in fact referring to any part of what Bill and I were discussing.  Well, you know, to say so, it's helpful and indicative of the relevance of the context.  You might have said so in the first place and saved us some time. (Excuse the pun)

Sounds like an interesting idea... I'll leave you to it!
Title: Re: What was before the big bang?
Post by: Bill S on 29/09/2015 04:09:43
Back to #222, if that's OK.

You say that at zero gravity time stops..  Does everything stop with it?  If so, how does time start again? 
If not, how can change occur without time in which to accomplish that change?
Title: Re: What was before the big bang?
Post by: PmbPhy on 29/09/2015 04:37:42
Quote from: Bill S
You say that at zero gravity time stops..
Who made this assertion? Its wrong. If that's someone's theory then it's not supposed to be posted here. That's what the New Theories forum. One mistake is that it makes no sense to refer to zero gravity since the term gravity is vague. It's either the gravitational force or the gravitational potential, not simply "gravity."

The rate at which a clock ticks depends on the difference in the gravitational potential between the clock and the observer. For that reason it cannot be said that zero gravity time stops. If you're far away from any gravitational source such as in-between star systems then the gravitational potential is zero there as is the gravitational force.
Title: Re: What was before the big bang?
Post by: timey on 29/09/2015 11:45:58
Back to #222, if that's OK.

You say that at zero gravity time stops..  Does everything stop with it?  If so, how does time start again? 
If not, how can change occur without time in which to accomplish that change?

Regarding your question and post: 222, I've sent you a pm.
Title: Re: What was before the big bang?
Post by: PmbPhy on 29/09/2015 13:06:43
Quote from: Aquarius
The big bang never happened.
While that may be true you act like you know that as a fact. What's your argument for your assertion?

Alan Guth has something to say on this subject. See: http://www.newenglandphysics.org/common_misconceptions/DSC_0005.MOV
Title: Re: What was before the big bang?
Post by: Bill S on 29/09/2015 18:17:10
Quote from: Pete
Who made this assertion? Its wrong. If that's someone's theory then it's not supposed to be posted here. That's what the New Theories forum.

Pete, I included the reference to #222 so that others would have the context for my question.

I can understand that experts like you might find this sort of thread a bit taxing on the patience, but the value for us hitch-hikers is that it can sharpen our understanding.  When meeting a non-standard idea we have to ask ourselves such questions as: Is this rubbish, or is there some underlying science?  Or: Confronted with this, what basic knowledge do I need to counter it? 

As far as the “New Theories” comment is concerned; you are probably right, but it does emerge quite naturally from the OP, which in itself, is inclined to invite speculation.

Is it a new theory?  The idea of time stopping is certainly not new.  A quick look at the threads in this forum would support that.  The zero gravity idea is not found as commonly, but is out there.

I think it was you who said recently that for an idea to be classed as a scientific theory it had to be supported by rigorous maths.  Vikki (Timey) has pointed to the lack of mathematical backing in her ideas, and said that having the ideas subjected to mathematical scrutiny is part of her reason for coming to TNS.

On the basis of this, it’s probably fair to say that her idea is neither new, nor a theory.

You have the knowledge, Pete, and usually show the patience necessary to help amateurs; why not give #222, which seems to be a brief overview of Vikki’s thoughts, a “professional once-over”?  Who knows, it might even bring her back to the forum.
Title: Re: What was before the big bang?
Post by: Davidlawrencekellam on 30/09/2015 13:04:32
Good morning:  Well, what was before the Big Bang? But first, we need to examine what happened prior to the Large Raipd Expansion which we call the Big Bang.  The singularity?  The singularity translated means "we don't know anything and we can't explain it" so we call it the singularity.  What existed prior to the big bang was what I label a "plasma" that consisted of anti-matter and was composed of massive amounts and had it's own particular characteristics which I will go into at a later time as I am under the gun this morning for time.  In order to be brief I will be explaining the following in subsequent posts.

1,  There was no singularity  2. The big bang was a rapid expansion/conversion of anti matter to matter, not all of which converted-this will be linked to the expanding universe. 3. The universe will not continue to expand as anti matter is a weaker (although more abundant) force than gravity and it has a shorter existence in its strongest form. 4. Not all anti matter was converted at the big bang event, which will explain why portions of the universe will be expanding at different rates. 5.Black holes do indeed allow for escape as we should all now know, and in the final stages of super massive black holes (well into the future as none exist currently) they create anti-matter. Think about it, we are trying to get anti-matter from matter now aren't we? 6. Nature and the natural order of things, once discovered, make logical sense.  Look at the mysteries of history and you will find that once they are discovered or understood, at the end of the day thay make perfect sense. There is much more to the above however  I am obligated to appear at another place.    Dave Lawrence Kellam
Title: Re: What was before the big bang?
Post by: Bill S on 30/09/2015 14:52:43
Hi David, welcome.

It seems you have some interesting and revolutionary ideas.  Although, obviously, they are linked to this thread, you might be wise to take them straight into "New Theories" to avoid criticism.